You are on page 1of 570

• Question 1:

• A patient with a known hypertrophic cardiomyopathy and


dynamic left ventricular outflow tract obstruction is intubated
for community-acquired pneumonia. Urine output is minimal,
with no response to a fluid bolus. The patient is tachycardic
with HR of 120/min and BP of 118/74 mm Hg. Which of the
following therapies is most appropriate as part of the
treatment?

A)Furosemide
B)Enalaprilat
C)Metoprolol
D)Nitroglycerin
E)Nicardipine
• Correct Answer: C
• Rationale
Therapy for hypertrophic cardiomyopathy is directed at the
dynamic left ventricular outflow tract obstruction. The
obstruction causes an increase in left ventricular systolic
pressure, which leads to a complex interplay of
abnormalities that decrease cardiac output. In the intensive
care setting, this condition often deteriorates with volume
depletion, and with the institution of inotropic agents. In
that circumstance, the infusion of fluids and the
discontinuation of inotropic agents is the initial therapy. A
beta-blocker should also be added; however, if hypotension
is present, a vasoconstrictor such as phenylephrine should
be administered first. Acute onset of atrial fibrillation may
result in severe hemodynamic compromise due to the loss
of atrial contractions. Prompt cardioversion should occur in
this circumstance.
• The first-line approach to the relief of symptoms
is to block the effects of catecholamines that
exacerbate the outflow tract obstruction, and to
slow that heart rate to enhance diastolic feeling.
Beta-blockers are generally the initial choice to
accomplish these goals. Verapamil, the calcium
channel blocker, can also be used. Sudden death
has been reported in patients with severe
pulmonary hypertension and severe outflow
obstruction who are given verapamil. This drug
should be given with caution in patients with this
combination of findings. Nitroglycerin would
decrease cardiac filling and is problematic.
• Question 2:
• A 5-day-old infant presents to the emergency
department with RR of 84/min, oxygen saturation of
88%, HR of 170/min, and BP of 55/20 mm Hg. Physical
examination findings are notable for severe retractions;
cool, mottled extremities with weak peripheral pulses;
and lethargy. After intubation, establishment of
peripheral IV access, and an initial bolus of 20 mL/kg of
normal saline, the patient has not improved. Laboratory
values are as follows: sodium, 144 mEq/L; potassium,
5.0 mEq/L; chloride, 105 mEq/L; carbon dioxide, 13
mEq/L; blood urea nitrogen, 35 mg/dL; creatinine, 1.2
mg/dL; glucose, 105 mg/dL; ionized calcium, 4.7 mg/dL;
lactate, 7 mmol/L.
In addition to providing broad-spectrum antibiotics
and fluid, the most appropriate next step in
resuscitation is:

A)Dopamine to start at 5 µg/kg/min

B)Prostaglandin infusion

C)Milrinone

D)Calcium gluconate, 100-mg/kg bolus

E)Cardiology consultation
• Correct Answer: B
• Rationale
Neonatal shock has a broad differential diagnosis. Infection,
congenital heart disease, arrhythmia, and inborn errors of
metabolism can all present with shock. Much as with pediatric
and adult shock patients, the initial minutes of the resuscitation
are spent establishing means to stabilize the patient and
supporting intravascular volume. For the neonate with evidence
of poor cardiac output, ductal dependent congenital heart
disease must be considered. Current recommendations include
the initiation of prostaglandin infusion to maintain ductal patency
until the diagnosis of congenital heart disease can be excluded. If
the patient does not improve with fluids and prostaglandins, a
dopamine infusion is the next step. While neonates are more
dependent on extracellular calcium for myocardial contractility
than adults, this child has a normal ionized calcium level and
would not benefit from higher levels. Pulmonary hypertension is
frequently seen in septic neonates and can be treated with
inhaled nitric oxide, but this is not the next appropriate step for
this patient.
• Question 3:
• Which of the following measures would result in an
immediate increase in right ventricular stroke volume?

• A)Exhalation during positive pressure ventilation

• B)Relief of inferior vena cava compression

• C)Relief of intra-abdominal pressure

• D)Sustained right lateral decubitus positioning

• E)Sustaining a Valsalva maneuver while sitting


• Correct Answer: A
• Rationale
It has long been recognized that positive airway pressure in
lung inflation can have distinct effects on heart loading
conditions and performance. Other direct and indirect
effects on the heart have been suspected as well. Sustained
increased intrathoracic pressure produces a net decrease in
venous return and a decrease in stroke volume and cardiac
output. This clinical effect needs to be differentiated from
the dynamic immediate effects of intrathoracic pressure
variations. Lung inflation with positive airway pressure may
have dynamic effects on myocardial contractile status that
can have rapid onset, even during a single breath.
Positive-pressure inspiration, inferior vena cava
constriction, and release of abdominal compression have
all been reported to decrease right ventricular (RV)
inflow. RV inflow is increased with positive pressure
expiration, the release of inspiratory hold, the
constriction of the inferior vena cava, and abdominal
compression. Decrease of the RV end flow during
positive pressure inspiration and vena cava constriction
transiently decreases RV end-diastolic volume and
increases the transseptal pressure gradient, causing the
septum to shift to the right.
With left ventricular (LV) end-diastolic volume, the result is no
change in the anterior/posterior diameter and a stroke volume
increase; when RV inflow increases, the opposite occurs.
Therefore, during the expiratory phase of mechanical ventilation,
and when the inspiratory hold and the inferior vena cava
constriction are released, or during abdominal compression, the
transseptal pressure gradient decreases, causing the septum to
shift to the left. LV end-diastolic pressure and stroke volume
decrease as a result.
Mechanical ventilation can affect cardiac function
through a number of mechanisms. Direct ventricular
interaction (change in the volume of one ventricle
causing a simultaneous and opposite change in the
volume of the other ventricle) is one of the mechanisms
by which positive end-expiratory pressure may decrease
the left ventricular end-diastolic volume in output during
mechanical ventilation. These effects are dependent on
the presence of pericardial constraint. A sudden decrease
in right ventricular end-diastolic volume is associated
with a simultaneous increase in left ventricular end-
diastolic volume, whereas a sudden increase in right
ventricular end-diastolic volume is associated with a
decrease in left ventricular end-diastolic volume.
• Question 4:
• A 40-year-old man with hypertension is admitted to the ICU for
3 days with right leg swelling. He has also noted pain and a
bluish discoloration. The patient had a previous left-leg, below-
the-knee amputation due to a motor vehicle accident and
venous thrombosis of the left leg, with the placement of an
inferior vena cava filter. BP is 102/52 mm Hg, HR is 80/min, RR is
28/min, and temperature is 35.2°C (95.3°F). The right leg is
noted to be swollen to the inguinal ligament with bluish
discoloration most prominent in the ankle, foot, and distal thigh.
Femoral and pedal pulses are palpable but decreased.
Laboratory results include the following: WBCs, 23,500/µL;
hemoglobin, 16.4 g/dL; prothrombin time, 41.3 seconds;
activated partial thromboplastin time, 200 seconds; D-dimer,
positive; sodium, 139 mmol/L; potassium, 3.5 mmol/L;
bicarbonate, 16 mmol/L; chloride, 102 mmol/L; glucose, 107
mg/dL; blood urea nitrogen, 26 mg/dL; and creatinine, 4.1
mg/dL.
Which of the following is the most important
the apeuti i te e tio fo this patie t’s o ditio ?

A)Catheter-directed thrombolysis
B)Operative venous thrombectomy
C)Right leg above-knee amputation
D)Broad-spectrum antimicrobials
E)IV heparin-based anticoagulation
• Correct Answer: A
• Rationale
This patient has phlegmasia cerulea dolens, an infrequent
but severe manifestation of venous thrombosis. This
condition can result in venous gangrene, arterial
compromise, loss of limb, and even death. The most
appropriate intervention to treat the underlying condition is
catheter-directed thrombolysis to rapidly remove the
thrombus and restore venous drainage. There are no well-
designed studies evaluating this approach, and it should be
reserved for limb salvage after an assessment of the risk-
benefit ratio, as compared with routine anticoagulation.
Operative thrombectomy is an alternative intervention, but
because of its high mortality rate, it is usually used after the
failure of anticoagulation and thrombolytic therapy.
Although anticoagulation is indicated, heparin will not
alleviate the venous obstruction rapidly enough. While
amputation may be needed when other interventions fail, it
is reasonable to utilize other interventions to salvage the
limb prior to considering amputation. Although this patient
has an elevated white blood cell count and hypothermia,
these manifestations are most likely secondary to the tissue
ischemia and subsequent inflammatory response, rather
than an established infection. It is reasonable to obtain
culture data and consider antibiotic therapy, but these
interventions would not address the underlying cause. In
some case reports, phlegmasia cerulea dolens has been
associated with prior placement of inferior vena cava filters.
It may occur in lower as well as upper extremities.
With mild manifestations and subocclusive thrombosis,
systemic anticoagulation may be sufficient. With occlusive
thrombosis and vascular compromise, catheter-directed
thrombolysis would be the treatment of choice. In some
cases, catheter access to the thrombosis may be impossible
and a thrombectomy would be indicated. Fasciotomy may be
required after reestablishment of the flow and stabilization
of the patient to relieve the elevated compartment
pressures.
• Question 5:
• A 36-year-old, gravida 1, para 1 woman is admitted to the
ICU for the diagnosis of severe preeclampsia with a
persistent BP of 165/112 mm Hg, 4+ proteinuria, and
decreased urinary output (< 30 mL/h). Which of the
following is the best initial agent for controlling her
hypertension?

A)Valsartan
B)Enalapril
C)Furosemide
D)Nitroprusside
E)Labetalol
• Correct Answer: E
• Rationale
Labetalol is the currently recommended drug of choice for
hypertension in the pregnant patient. All antihypertensive
agents can cross the placenta. Angiotensin receptor
blockers such as valsartan and angiotensin-converting
enzyme inhibitors such as enalapril are known to be
harmful to the fetus and should not be used whenever
possible. Nitroprusside has the potential to develop toxic
metabolites over time and with high doses and should not
be the initial agent selected absent a life-threatening
malignant hypertensive crisis. Furosemide can be used for
management of hypertension and is thought to be safe for
the fetus.
The risk in using a diuretic in a pregnant patient is
volume depletion; for this reason, furosemide
should be used with caution. Labetalol can be
used in the pregnant patient and, since it has
both alpha-adrenergic and beta-adrenergic
blocking properties, it may preserve placental
blood flow better than other beta-blockers.
While not listed as a choice, calcium channel
blockers such as nifedipine have also been used
to manage hypertension in pregnancy, as has
magnesium sulfate.
• Question 6:
• The morning after an open cholecystectomy, a 49-
year-old woman develops nausea and vomiting. On
examination, her abdomen is tympanitic and
distended. She has diffuse mild pain with palpation.
Radiograph of the abdomen shows a normal gas
pattern in the small bowel and distended right and
transverse colon. Rectal enemas result in a slight
decrease in abdominal distension. However, the
patie t’s ausea a d o iti g pe sists. Afte se e al
doses of a a tie eti , the patie t’s h th st ip
shows QT prolongation and then torsade de pointes.
She is successfully resuscitated with magnesium
sulfate.
• Which of the following medications was most likely
used to treat her nausea and vomiting?

A)Metoclopramide

B)Domperidone

C)Droperidol

D)Ondansetron

E)Hydrocortisone
• Correct Answer: C
• Rationale
A number of drugs can lead to QT prolongation and torsade de
pointes. Droperidol, a butyrophenone derivative, is an
antiemetic that has the potential of prolonging the QT interval.
Fortunately, it rarely produces this phenomenon at
recommended doses. Antiemetics such as ondansetron
(serotonin antagonists), metoclopramide (antidopaminergic
and antiserotonergic), and dronabinol (cannabinoid derivative),
are not known to cause QT prolongation. Phenothiazines used
for nausea and vomiting may also cause QT prolongation but
are not among the answer choices. Targeted drug therapy for
nausea and vomiting can improve the success of relieving
symptoms. In addition, being aware of associated adverse drug
reactions can help decrease the drug-related complications
from the drug itself or through drug-drug interactions.
.
Postoperative nausea and vomiting are often
multifactorial in origin. Drugs, physical stimuli, or
emotional stress can cause the release of
neurotransmitters that stimulate serotonergic (5-HT3),
dopaminergic (D2), histaminergic (H1), and muscarinic (M1)
receptors. The receptor stimulation in the chemoreceptor
trigger zone, gastrointestinal tract, vestibular apparatus,
pharynx, or cerebral cortex triggers neurogenic signals to
be sent to the vomiting center in the brainstem. The
vomiting center, rather than a discrete area, is more of a
neural network comprising the chemoreceptor trigger
zone, area postrema, and nucleus tractus solitarius.
Vagal afferent signals through the nodose ganglion
and the nucleus tractus solitarius mediate nausea
that arises from gastric irritants; gastric, small
intestinal, colonic, or bile duct distension; and
inflammation or ischemia of bowel, liver, pancreas,
and peritoneum.
Phenothiazines and butyrophenones act on D2, H1,
and M1 receptors. Benzamides, such as
metoclopramide and domperidone, affect 5-HT3 and
5-HT4 receptors; scopolamine is an M1-receptor
antagonist; and diphenhydramine and cyclizine are
H1 antagonists. Specific 5-HT3-receptor antagonists,
such as ondansetron and granisetron, are the most
recently developed class of antiemetics.
• Question 7:

• A 4-month-old boy is admitted to the ICU following repair of


tetralogy of Fallot. Initially, temperature is 38.2°C (100.7°F), HR
is 130/min, and BP is 85/45 mm Hg. He is being supported
hemodynamically with an epinephrine infusion at 0.05
µg/kg/min and a milrinone infusion at 0.5 µg/kg/min. His
initial central venous pressure is 14 mm Hg. Shortly after
arrival, his BP decreases to 75/42 mm Hg, his HR increases to
148/min, and his central venous pressure increases to 16 mm
Hg. Systemic venous oxygen saturation at this time is 72%. His
rhythm is shown in the Figure.
Which of the following is the most important next step
in the management of this patient?
A)Cooling the infant to less than 37°C (98.5°F)
B)Increasing the epinephrine infusion
C)IV amiodarone therapy
D)IV lidocaine therapy
E)IV magnesium therapy
• Correct Answer: A
• Rationale
The infant in the vignette has junctional ectopic
tachycardia (JET). No p waves are visible on the ECG
tracing. Further, the increase in central venous pressure
and decrease in blood pressure commonly occur as a
result of the atrioventricular dyssynchrony that occurs
with JET. The pathophysiology of postoperative JET is most
likely secondary to trauma, infiltrative hemorrhage, or
inflammation of the conduction tissue. Patients
undergoing repair of tetralogy of Fallot have been found to
be especially at risk for developing JET because of the
marked inflammation that occurs following infundibular
resection during repair of tetralogy of Fallot. Further, fever
has been shown to be one of the more potent stimuli of
JET.
• Treatment depends on the degree of hemodynamic
instability. For the patient in the vignette, blood
pressure is still within normal limits for age, and
systemic oxygen delivery is adequate. Thus,
establishing normothermia and time may be all that
are needed to convert to sinus rhythm. Therefore,
option A is a reasonable first step. Excessive
catecholamine use can also trigger JET, though this
patient is only receiving low-dose epinephrine, his
blood pressure is on the low end of the normal
range, and thus he may not tolerate weaning in the
immediate postoperative period. Therefore, option
B would be incorrect as the next step in the
management.
• If the patient were more unstable with a higher heart rate
and lower blood pressure, IV amiodarone therapy (option
C) could be implemented. An IV bolus of 1-5 mg/kg
followed by a continuous infusion of 5-10 µg/kg/min is the
traditional protocol. In the patient in the vignette,
however, amiodarone is not yet necessary. Lidocaine only
affects conduction of ventricular tissue and thus would not
be helpful for JET. If hypomagnesemia was present in the
patient, IV magnesium would be helpful. Empiric
magnesium therapy, however, is not recommended in
non–life threatening situations, and thus option E is
incorrect.

Other therapies that could also be helpful in this patient


include temporary cardiac pacing and fluid administration.
• Question 8:
• A patient has an in-hospital cardiac arrest. Which of the
following variables has poor predictive value in estimating the
likelihood of survival to hospital discharge?

A)Coronary perfusion pressure during closed chest


compressions
B)End-tidal carbon dioxide level during closed chest
compressions
C)Poor neurologic status immediately after return of
spontaneous circulation
D)Failure of return of spontaneous circulation after 30 minutes
of CPR
E)Metabolic acidosis resolution upon return of spontaneous
circulation
• Correct Answer: C
• Rationale
Although chances of surviving to hospital discharge may
be low in many cases of in-hospital cardiac arrest, the
chances of survival to discharge are dismal in a few
scenarios. Prolonged (>30 min) resuscitation efforts (as in
option D) probably confer no benefit if the return of
spontaneous circulation has not been achieved. Failure to
increase the end-tidal carbon dioxide pressure to exceed 8
mm Hg indicates that a markedly low cardiac output is
present, and cardiopulmonary resuscitation is not likely to
be effective. In a study placing central monitors in subjects
receiving chest compressions, a failure to augment
coronary artery perfusion pressure to greater than 15 mm
Hg identified subjects that could not be resuscitated.
Neurologic findings, including pupillary response,
reflexes, and motor activity, may change over the
first 72 hours after return of spontaneous
circulation. However, at the end of 72 hours, the
neurologic findings are highly predictive of
neurologic outcome. If the initial rhythm of the
cardiac arrest was ventricular fibrillation
(possibly with other initial rhythms of cardiac
arrest), and the return of spontaneous circulation
can be achieved, therapeutic hypothermia
should be initiated immediately in order to
minimize neurologic injury and give the patient
the best chance for a better neurologic
outcome.
• Question 9:
• A 24-year-old woman undergoes spinal anesthesia for
fractional dilation and curettage in the lithotomy position. She
arrives in the recovery room resting comfortably on 40%
oxygen via face mask. She suddenly becomes agitated,
diaphoretic, and cyanotic, and develops altered mental status.
She is promptly intubated and manually ventilated with 100%
oxygen. Physical examination reveals a temperature of 37.8°C
(100°F), BP of 80/60 mm Hg, HR of 44/min, and RR of 22/min
with bilateral basilar rales and a churning murmur over her left
sternal border. Neurologic examination is remarkable for no
response to sternal rub with pinpoint pupils and left lateral
gaze preference. Arterial blood gases on 100% oxygen reveal
pH of 7.42, PaCO2 of 28 mm Hg, and PaO2 of 68 mm Hg. Which
of the following would be the most appropriate immediate
course of action?
A)Left lateral decubitus Trendelenburg position

B)Insertion of a central venous catheter

C)Atropine, 0.5 mg IV push

D)Immediate brain CT with contrast

E)Lorazepam, 2 mg IV push
• Correct Answer: A
• Rationale
The sudden occurrence of unexplained
cardiopulmonary dysfunction with neurologic
findings during or soon after an
obstetric/gynecologic surgical procedure should
suggest the possibility of venous air embolism
leading to paradoxic arterial embolism. Whenever a
surgical wound disrupts veins, creating a blood-air
interface that lies above the level of the right atrium,
there is a potential for negative intravascular
pressure to create venous air emboli. Venous air
embolism may occur during or after a cesarean
section, orogenital sex, normal labor with placenta
previa, or illegal abortion.
• The primary pathophysiologic event in venous
embolism is intense vasoconstriction of the
pulmonary circulation, which results in
ventilation/perfusion mismatch leading to hypoxia,
interstitial pulmonary edema, and systemic
hypotension leading to reduced cardiac output as
pulmonary vascular resistance increases. Paradoxic
arterial air embolization may further complicate the
course of a patient who has a patent foramen ovale.
As venous emboli increase right atrial pressure, a
patent foramen ovale may open, thereby creating a
right-to-left shunt by which air reaches the arterial
circulation. The precordial ill heel u u is
characteristic of venous air embolism.
The appropriate management is to place the patient in the
left lateral decubitus head-down position to decrease the
venous air leaving the pulmonary outflow tract. Oxygen
(100%) should be administered to allow reabsorption of
nitrogen. If a central venous catheter is in place, aspiration
of air can be attempted. Other supportive measures, such
as mechanical ventilation, are utilized as indicated.
Volume replacement is unlikely to correct the
hemodynamics in this patient and may increase right-
sided pressure and lead to further paradoxic arterial
emboli. Atropine will also have no benefit in this patient.
Head CT would delay appropriate management, and
systemic heparinization is not indicated because the
clinical situation does not suggest pulmonary embolism.
Use of hyperbaric oxygenation is controversial.
• Question 10:
• A 6-month-old with complex congenital heart disease is 4 hours
postbypass when she develops pulmonary hypertension.
Despite adjustments in ventilator, fluids, and acid-base status,
the pulmonary hypertension persists and the baby is started on
inhaled nitric oxide. Which of the following is the mechanism of
action of inhaled nitric oxide in the therapy of pulmonary
hypertension in this child?

A)Increase in cyclic guanosine monophosphate (cGMP) through


activation of guanylate cyclase
B)Increase in cGMP through inhibition of phosphodiesterase-5
C)Increase in cyclic adenosine monophosphate (cAMP) through
inhibition of phosphodiesterase-3
D)Decrease in cGMP through inhibition of phosphodiesterase-3
E)Decrease in cAMP through inhibition of phosphodiesterase-5
• Correct Answer: A
• Rationale
Nitric oxide interacts with guanylate cyclase, resulting in
increased intracellular cyclic guanosine monophosphate
(cGMP). Increased cGMP causes a reduction in cytosolic
calcium by sequestration in the sarcoplasmic reticulum and by
ellula e t usio . Nit i o ide used to e alled e dotheliu -
de i ed ela i g fa to .
Nitric oxide is synthesized in the vascular endothelial cell
from L-arginine by nitric oxide synthase. Its half-life is very
short because of inactivation in the bloodstream either by
binding with hemoglobin and forming methemoglobin and
nitrate (NO3–) or by binding with oxygen and forming nitrite
(NO2–).
Phosphodiesterase-5 is an enzyme that degrades cGMP in
vascular smooth muscle with a reduction in vasodilation.
Sildenafil is a phosphodiesterase-5 inhibitor, so it causes
pulmonary vasodilation through an increase in cGMP levels.
• Question 11:
• A 45-year-old woman with no significant past
history has abdominal pain and weight loss and
undergoes IV-contrast CT of the abdomen and
pelvis for evaluation. After administration of the
contrast material, the patient reports throat
tightness, diffuse itching, and light-headedness.
Physical examination reveals a diaphoretic and
anxious patient. BP is 72/48 mm Hg, HR is
124/min, and RR is 25/min. Examination of the
chest reveals diffuse wheezing. A rash is noted
o the patie t’s hest a d uppe e t e ities.
• Which of the following is the most appropriate
immediate treatment for the patie t’s hypotension?

A)Dobutamine

B)Vasopressin

C)Epinephrine

D)Hydrocortisone

E)Diphenhydramine
• Correct Answer: C
• Rationale
This patient is in anaphylactic shock, most likely as a reaction
to the contrast material used in the CT. An anaphylactic shock
is the most severe presentation of an anaphylaxis. Anaphylaxis
can be triggered by immunologic or nonimmunologic
mechanisms. Agents associated with anaphylactoid reactions
include radiocontrast dye, opiates, aspirin, and other
nonsteroidal antiinflammatory drugs. Therapy for an
anaphylactic shock includes prompt administration of IV fluids
and epinephrine. Epinephrine can be administered
subcutaneously, intramuscularly, or intravenously. In patients
with clear signs of hypoperfusion and shock, subcutaneous
administration is not recommended. For treatment of
anaphylactic shock, epinephrine should be administered
intramuscularly or intravenously. IV administration is reserved
for cases of severe hemodynamic collapse; extreme caution is
necessary in patients at risk for myocardial ischemia.
Corticosteroids have a role in the later treatment of
anaphylaxis, mostly to prevent late-phase reactions such as
intermediate airway edema and bronchoconstriction. However,
corticosteroids have no immediate effect on hemodynamic
abnormalities in anaphylactic shock. Dobutamine is an
inotrope a d ill ot help i p o e the patie t’s lood p essu e
since anaphylactic shock is a vasodilatory shock process.
• Question 12:
• A supine patient on positive end-expiratory pressure (PEEP) of
5 cm H2O has the following hemodynamics measured at end-
expiration: right atrial pressure, 7 mm Hg; pulmonary artery
occlusion pressure, 12 mm Hg; arterial pressure, 110/70 mm
Hg. One hour later you are informed of the following changes:
right atrial pressure, 16 mm Hg; pulmonary artery occlusion
pressure, 21 mm Hg; arterial pressure, 120/80 mm Hg. Which
of the following most likely explains the observed changes?

A)Increased respiratory muscle activity


B)Acute myocardial ischemia
C)Increase in PEEP to 15 cm H2O
D)Change in the level of the transducer
E)Fluid bolus of 500 mL of 0.9% normal saline
• Correct Answer: D
• Rationale
Increased expiratory muscle activity (option A) can cause
substantial elevation in juxtacardiac pressure and elevate
both the right atrial pressure and pulmonary artery
occlusion pressure (PAOP), although one would not expect
the increase in intra-arterial pressure. Right ventricular
ischemia (option B) causes an acute decrease in
ventricular compliance, resulting in an elevated right atrial
pressure that would not manifest in a sudden rise in the
PAOP unless both ventricles were involved. A positive end-
expiratory pressure (PEEP) of 15 cm H2O (>12 mm Hg)
would not raise the right atrial pressure and PAOP by 10
mm Hg. Normally, only about half of PEEP is transmitted
to the juxtacardiac space, and the fraction is even less
when lung compliance is reduced, as in acute respiratory
distress syndrome.
An increase in the PAOP equivalent to the increase in PEEP
would indicate non–zone 3 conditions. However, in this
case, the increase in PAOP is higher than the increase in
PEEP, indicating that the PAOP value does not reflect
alveolar pressure. The most likely cause of the observed
pressure changes is a change in the level of the transducer.
A change in the transducer (without releveling by opening
the leveling stopcock to room air) of 5 inches (12.5 cm)
below the previous level of measurement will result in
about a 9 mm Hg increase in all the measured
hemodynamic pressures. A 500-mL 0.9% normal saline
solution fluid bolus shuld not increase the pressures by 9
mm Hg, and if the bolus were to meet a plasma volume
deficit, one would expect to see a change in the pulse
pressure—an event not noted in this patient. Thus, the
clinician should suspect a device-related error
when all hemodynamic pressures change the same
amount.
• Question 13:
• A 58-year-old woman is in the ICU with acute
respiratory distress, bilateral rales, and a systolic
murmur. Chest radiograph shows pulmonary edema,
normal heart size, and no pleural effusions. A
pulmonary artery catheter is inserted revealing
cardiac output of 5.0 L/min (cardiac index of 2.7
L/min/m2), right atrial pressure of 16 mm Hg,
pulmonary artery pressure of 40/24 mm Hg,
pulmonary artery occlusion pressure of 28 mm Hg,
SvO2 of 66%, and SaO2 of 92%. The tracing shown in
the Figure is from the distal lumen of the pulmonary
artery catheter with arrow indicating balloon
inflation.
• Which of the following could explain the data?

A)Acute inferior myocardial infarction with papillary muscle


ischemia
B)Postinfarct ventricular septal defect
C)Acute saddle pulmonary embolism
D)Incomplete wedging of the catheter
E)Chronic pulmonary hypertension
• Correct Answer: A
• Rationale
The tracing shows an elevated pulmonary artery pressure
changing to an elevated pulmonary artery occlusion pressure
(PAOP) with a dominant and elevated v wave upon balloon
inflation. A prominent v wave may be due to papillary muscle
ischemia; however, it can also occur in the absence of mitral
regurgitation and may be seen in patients with noncompliant
left atrium or in postinfarct ventricular septal defect. However,
in the latter case, both the SvO2 and thermodilution cardiac
output typically would be higher. In both acute pulmonary
embolism and pulmonary hypertension, pulmonary vascular
resistance is increased, elevating the pulmonary artery
pressure but not the PAOP. Thus, one would expect to see a
gradient between the pulmonary artery end-diastolic pressure
and the PAOP. The pulmonary artery pressure can be
differentiated from the PAOP by comparing the timing of
pulmonary artery systole, which occurs with the ECG T wave, to
the timing of the PAOP v wave, which occurs after the T wave.
• Question 14:
• A 44-year-old man with lobar pneumonia and severe
hypoxemia requires endotracheal intubation and
mechanical ventilation. He is sedated with etomidate and
given succinylcholine for intubation. He is then sedated
with a continuous infusion of lorazepam; analgesia is
managed with morphine as needed. He is also given a
long-acting neuromuscular blocker (vecuronium) because
of dyssynchrony and high peak airway pressures. After
transfer to the ICU, the mean arterial pressure, which had
initially been mildly low (52 mm Hg) and did not rise with
plasma volume expansion, drops to 36 mm Hg; a
norepinephrine infusion is started. His mean arterial
pressure remains low (50 mm Hg) despite norepinephrine
at 0.15 µg/kg/min. Sepsis-associated relative adrenal
insufficiency is suspected, and he is administered low-dose
steroids, to which the blood pressure responds, permitting
a substantial decrease in norepinephrine.
Which of the following medications most likely has
contributed to the hypotension in this patient?

A)Etomidate

B)Succinylcholine

C)Lorazepam

D)Vecuronium

E)Morphine
• Correct Answer: A
• Rationale
The patient has sepsis and is at risk for relative
adrenal insufficiency that could be worsened or
precipitated by a medication that affects the adrenal
axis. Etomidate can inhibit adrenal steroidogenesis
and would be the most likely agent in this scenario
to precipitate or worsen adrenal dysfunction. When
intubating a patient with severe sepsis or septic
shock, it would be prudent to either (a) select an
alternate sedative agent for induction, or (b) have a
heightened awareness of the need to test the
adrenal axis (or administer steroid replacement
therapy) in the event that the patient develops
refractory hypotension. The other options are not
known to affect the adrenal axis.
• Question 15:
• A patient is brought to the emergency department in
cardiac arrest with an unknown down time; ventilation
is occurring by bag-valve mask. You perform
endotracheal intubation and use colorimetric end-tidal
carbon dioxide testing to help confirm endotracheal
tube placement. The color change is not as intense as
usual but is persistent. Breath sounds can be heard with
bagging, and no rush of air is heard over the stomach
with ventilation; chest compressions continue. You bag
ventilate the patient through the endotracheal tube 10
times, and the color change persists but remains weak.
Which of the following most likely explains the low-level
color variation?

A)Esophageal intubation with a blown cuff

B)Venous hyperoxia following cardiac arrest

C)Hypoventilation with the bag ventilation

D)Low cardiac output despite compressions

E)Overventilation associated with low carbon dioxide


levels
• Correct Answer: D
• Rationale
Qualitative capnometry is a rapid and simple
confirmatory test for endotracheal tube placement. The
chemical indicator in the device will change color
depending on the carbon dioxide concentration. A
quantitative end-tidal carbon dioxide detector is
typically not necessary. In patients with profoundly low
cardiac output, end-tidal carbon dioxide may be difficult
to detect because the lungs are not being perfused. A
profoundly low cardiac output could cause this clinical
scenario, and this may be due to ineffective chest
compressions with cardiopulmonary resuscitation,
tension pneumothorax, or some other cause of an
inadequate cardiac output during CPR.
An esophageal intubation could potentially give a positive reading
with a colorimetric indicator because of the air that could be in
the stomach when the patient is mask ventilated prior to
intubation. However, that color change should go away after
delivery of several breaths (less than 6, most commonly). Since
the low-level color change persisted, esophageal intubation is an
unlikely source.
Venous hyperoxia has been demonstrated after cardiac arrest
with the return of spontaneous circulation and is presumed to be
secondary to bioenergetic failure and defects in oxygen utilization
afte a p olo ged do ti e. This ould ha e o i pa t o
end-tidal carbon dioxide detection. Overventilation can lower
carbon dioxide levels, but reduced carbon dioxide from an
increased minute ventilation would not lead to a decreased color
change intensity at clinically achievable levels of carbon dioxide in
a patient with spontaneous perfusion.
• Question 16:
• A rapid response call is made on a 4-month-old boy with a
history of Ebstein anomaly, now status post hemi-Fontan
procedure, for tachycardia greater than 200/min that is
unresponsive to vagal maneuvers. A 12-lead ECG reveals
atrioventricular nodal reentrant tachycardia. Which of the
following is an acceptable pharmacologic option for acute
t eat e t of this patie t’s supraventricular tachycardia?

A)Adenosine
B)Amiodarone
C)Digoxin
D)Verapamil
E)Esmolol
• Correct Answer: A
• Rationale
Adenosine is the preferred pharmacotherapy for
restoration of sinus rhythm in children with
supraventricular tachycardia (SVT). Since its site of action
is the atrioventricular (AV) node, this agent is preferable to
use in a patient with AV node reentry SVT. Amiodarone
may be considered in refractory SVT. Most of the
experience using amiodarone in children is in the
treatment of postoperative junctional tachycardia. The use
of digoxin in this patient is not preferred. Wolff-Parkinson-
White s d o e is a possi ilit ased o the patie t’s
congenital heart anomaly; digoxin is contraindicated
because it may enhance the conduction over the
accessory pathway, leading to atrial flutter or fibrillation.
Verapamil is contraindicated in this age group.
• Question 17:
• A patient with severe acute pancreatitis develops
symptomatic hypotension and is treated with 2 L of
crystalloid rapidly infused via peripheral IV lines. After
the initial crystalloid resuscitation, arterial pressure rises
to 88/40 mm Hg but the patient remains oliguric; a
e t al e ous li e is pla ed. The patie t’s hest
radiograph after central venous pressure (CVP) catheter
insertion prior to your planned intubation is read as
pulmonary edema. Central venous pressure before
intubation is 10 mm Hg.
• Which of the following is the most appropriate initial course
of action to take?

A)Stop IV fluids as a CVP of 10 mm Hg is adequate.

B)Stop IV fluids and initiate a vasopressor medication.

C)Start an IV diuretic to reduce the CVP below 9 mm Hg.

D)Continue plasma volume expansion with crystalloids.

E)Obtain a CT abdomen/pelvis to evaluate for hemorrhage.


• Correct Answer: D
• Rationale
This patient with severe acute pancreatitis has
developed evidence of acute tissue hypoperfusion
(hypotension and oliguria). At this time, the etiology of
the circulatory insufficiency is not clear but is probably
related to the typical third-space fluid shift that
accompanies severe pancreatitis. Hypovolemia (due to
either capillary leak or perhaps a hemorrhagic process
in the pancreas) is possible as a source of the
hypotension. Sepsis is also a potential concern. With
signs of circulatory insufficiency, the patient needs
aggressive resuscitation. After 2 L of crystalloid, the
patient is still hypotensive and oliguric.
Although fluid resuscitation in a patient with a history of
congestive heart failure warrants some degree of
caution, this patient has signs of circulatory
insufficiency. Therefore, restrictive fluids would not be
prudent. Although the appearance of the lung fields on
the chest radiograph suggests a pulmonary edema,
given the fluid responsiveness a noncardiac pulmonary
edema is the most likely etiology. Moreover, pancreatitis
often results in splinting and reduced lung volumes
leading to the appearance of vascular crowding on chest
radiograph and may be read as consistent with
pulmonary edema. This appearance often vanishes with
positive pressure ventilation and positive end-expiratory
pressure.
The elevated central venous pressure does not change
the conclusion about volume status in this case as
several studies have shown that CVP does not help in
the prediction of fluid responsiveness. Fluid
resuscitation should be continued in this patient to
resolve the signs of circulatory insufficiency. In a patient
such as this one, adequate fluid resuscitation may result
in the need for mechanical ventilation and positive end-
expiratory pressure to maintain arterial oxygen
saturation and carbon dioxide clearance.
• Question 18:
• A 55-year-old man with a history of ischemic
cardiomyopathy presents with a 3-day history of increased
shortness of breath. Physical examination reveals
increased work of breathing. ECG shows sinus tachycardia
with no ST-segment changes. Chest radiography reveals
pulmonary edema. Results of initial troponin I testing are
negative. SpO2 is 88% on 50% FIO2. Appropriate
respiratory management includes:

A)Benzodiazepines for dyspnea


B)100% oxygen nonrebreather mask
C)Initiating noninvasive ventilation
D)Emergently intubating the patient
E)Nebulized albuterol/ipratropium
• Correct Answer: C
• Rationale
The patient is presenting with acute cardiogenic
pulmonary edema. Noninvasive ventilation in these
patients has been shown to reduce the need for
emergent intubation and may improve mortality.
Noninvasive ventilation has been shown to improve
oxygenation and ventilation, and to unload respiratory
muscles, reducing the work of breathing. Positive airway
pressure also results in increased intrathoracic pressure,
decreasing left ventricular afterload, and improving
cardiac output. Nebulized albuterol/ipratropium will not
influence cardiogenic pulmonary edema. Instead
medication therapy would appropriately include a
diuretic.
Question 19:
A 71-year-old man presents to the emergency department
with a BP of 120/80 mm Hg and reports chest discomfort.
The rhythm shown in the Figure is noted.
• What immediate intervention is most appropriate?

A)Carotid sinus massage

B)IV beta blockade bolus

C)IV amiodarone infusion

D)Electrical cardioversion

E)IV procainamide bolus


• Correct Answer: D
• Rationale
The ECG indicates ventricular tachycardia and the
patient reports chest pain. Therefore, despite a normal
blood pressure, this patient is appropriate for acute
intervention to terminate the abnormal rhythm.
Medication therapy is not appropriate as an immediate
intervention in this setting.
• Question 20:
• A 65-year-old man with a history of diabetes mellitus,
coronary artery disease, and emphysema is found
unresponsive 10 hours after an uneventful laparoscopic
cholecystectomy done at 2AM. The resuscitation team
arrives, finds him to be in pulseless electrical activity,
and promptly initiates chest compressions. He is
intubated and continuous waveform capnography
reveals an end-tidal carbon dioxide level of 8 mm Hg.
For the code team leader.
• which of the following is the best next management?

A)Assess chest compression adequacy to ensure optimal


circulation.

B)Reposition the endotracheal tube as it probably


migrated out.

C)Terminate efforts as return of spontaneous circulation is


unlikely.

D)Give additional vasopressors to augment myocardial


oxygen delivery.

E)Insert an emergency temporary transvenous pacemaker.


• Correct Answer: A
• Rationale
Quantitative waveform capnography is the continuous
measurement of end-tidal carbon dioxide. During
cardiopulmonary resuscitation, capnography can be useful in a
number of ways, and has now been incorporated into
Advanced Cardiac Life Support guidelines as of 2010 (Class 2b
recommendation). Continuous waveform capnography is now
recommended for intubated patients throughout the peri-
arrest period. During CPR, capnography is useful in evaluating
the effectiveness of chest compressions. Ongoing high-quality
chest compressions should achieve an end-tidal carbon dioxide
level in the 10-20 mm Hg range, and certainly, very low
measurements (<10 mm Hg) should signal the need for
improved quality of administered chest compressions. Animal
and human studies have shown an association between end-
tidal carbon dioxide levels and cerebral perfusion pressure
during CPR. The code team leader should immediately assess
compression depth, rate, and provider fatigue. Therefore,
option A is correct.
• As with colorimetric end-tidal carbon dioxide detection after
endotracheal intubation, waveform capnography can also be
utilized to confirm tube placement. Occasionally, small
concentrations of carbon dioxide can be detected after an
esophageal intubation, especially if bag-mask ventilation
preceding intubation insufflated the stomach with previously
exhaled air. Observation over several breaths usually
overcomes this problem. Therefore, the ongoing presence of
detectable exhaled carbon dioxide for more than 6 bagged
breaths makes esophageal placement of the endotracheal tube
unlikely. Therefore, option B is incorrect.
Additionally, quantitative waveform capnography can mark
return of spontaneous circulation. When return of
circulation occurs, there is a significant rise in end-tidal
carbon dioxide, typically into the 35-45 mm Hg range. This
reflects increase in blood flow to the lungs, and
subsequent carbon dioxide elimination. Although
prolonged cardiac arrest (and associated low end-tidal
carbon dioxide levels during CPR) makes return of
spontaneous circulation unlikely, the decision to terminate
resuscitative efforts should be individualized and take into
account the underlying acute and chronic illnesses,
prognosis, and patient wishes. Therefore, option C is
incorrect.
Finally, vasopressors cause an increase in afterload, which
increases blood pressure and myocardial blood flow during
CPR. In some patients, small decreases in end-tidal carbon
dioxide levels have been noted after vasopressor therapy.
Though the mechanism is not entirely clear, the hypothesis is
that the higher afterload leads to a transient drop in cardiac
output. However, fluctuation in end-tidal carbon dioxide levels
after vasopressor therapy is variable and unlikely to be
clinically significant. End-tidal carbon dioxide levels have never
been utilized to guide specific pharmacologic therapy during
CPR, and treatments should follow the Advanced Cardiac Life
Support algorithm. Therefore, option D is incorrect.
Question 21:
A 4-month-old girl with a history of repair of tetralogy of Fallot and
atrioventricular canal underwent a cardiac catheterization due to
failure of weaning from mechanical ventilation. Cardiac
catheterization data are shown in the Figure.
• The most likely cause for continued mechanical support is:

A)Residual ventricular septal defect

B)Right ventricular outflow tract obstruction

C)Right atrioventricular valve regurgitation

D)Left atrioventricular valve regurgitation

E)Left atrioventricular valve stenosis


• Correct Answer: A
• Rationale
The step-up in oxygen saturation from the right atrium
to the right ventricle is caused by left-to-right shunting
through a ventricular septal defect.
• Question 22:
• A patient undergoes an uncomplicated quadruple
coronary artery bypass graft using cardiopulmonary
bypass. Rewarming is uneventful and the patient is
extubated and brought to the CT ICU. He has a labile
mean arterial pressure and heart rate. You are called to
the bedside to provide management as the cardiac
surgeon is involved in another case. The chest tubes
have drained 400 mL over the past 1.5 hours. The
pulmonary artery occlusion pressure is measured at 18
mm Hg, but the cardiac index is 1.8 L/min/m2; the
patient is on no pressors.
Which of the following statements regarding hemodynamic
instability immediately following cardiac surgery utilizing
cardiopulmonary bypass is most appropriate to guide patient
management?

A)Filling pressure inaccuracy stems from dynamic changes in


myocardial compliance.
B)Urine output >1 mL/kg/h is a reliable indicator of
appropriate cardiac preload.
C)Protamine infusion after cardiopulmonary bypass does not
affect hemodynamics.
D)Pulseless electrical activity should prompt reexploration in
the operating room.
E)Hemodynamic instability is principally causally related to
postoperative hypothermia.
• Correct Answer: A
• Rationale
After exposure to extracorporeal circulation, an intense
proinflammatory response induced by the
cardiopulmonary bypass circuit can cause profound
hemodynamic derangements in the immediate
postoperative period from cardiac surgery. This may be
further complicated by the presence of myocardial
depression, which may be de novo or an exacerbation of
preexisting poor myocardial function. The
proinflammatory postcardiopulmonary bypass state can
be associated with potent peripheral vasodilation.
Use of protamine sulfate has been associated with both
immune-mediated and idiosyncratic reactions that can
manifest as severe hypotension, especially in diabetics.
Dynamic changes in myocardial compliance, especially
in the setting of hypothermia, can make estimation of
cardiac preload based on filling pressures quite
challenging. Changes in cardiac output with
intravascular volume expansion may be better at
identifying preload dependency as compared to
pressure-based assessments.
• If the patient developed a pulseless electrical activity,
cardiac tamponade could be the etiology. However,
there would be no time to bring the patient back to the
operating room, and emergent bedside resternotomy in
the ICU (in the hands of qualified personnel) would be
indicated to evacuate any clot that could be obstructing
cardiac output.

A high urine output is not necessarily a good indicator of


volume status, as a post–cardiopulmonary bypass
diuresis is common immediately after cardiac surgery,
especially if mannitol is included in the pump prime
solution.
• Question 23:
• A 67-year-old man undergoes on-pump triple coronary
artery bypass grafting with left and right internal
mammary artery grafts as well as a saphenous vein
graft. The operation is uneventful and he is extubated in
the operating room and brought to the CT ICU. Three
hours later, his mediastinal and pleural tubes begin
draining brighter blood and the rate has slowly
increased from 70 mL/h to 300 mL/h.
• Which of the following statements about postoperative
bleeding immediately after cardiac surgery is most
accurate?

A)Surgical hemorrhage is most common and should lead


to reoperation.
B)It should be treated with a bolus and infusion of an
antifibrinolytic agent.
C)Platelet dysfunction is common and related to
hypothermia or the bypass circuit.
D)More than 200 mL/h for 4 hours is an indication for
echocardiography.
E)Correcting medical causes of coagulopathy should
precede reexploration.
• Correct Answer: C
• Rationale
Concerns over postoperative bleeding are common in the
ICU after cardiac surgery. Most of the bleeding is
considered a medical (related to platelet dysfunction or
other acquired coagulopathies) rather than a surgical
(technical procedure–related) problem of hemostasis.

Dysfunction of platelets is common, and is usually the


result of hypothermia or exposure to the cardiopulmonary
bypass circuit. As systemic heparin administration is
required for cardiopulmonary bypass, a patient may need
additional dosing of protamine to correct any acquired
coagulopathy associated with heparin. Thrombocytopenia
is also common and often warrants platelet transfusion in
the ICU.
The decision to take a patient back to the operating room for
reexploration after cardiac surgery is a function of the total
output of blood from the mediastinal tubes as well as the rate
of bleeding. A generally accepted rule is that an output of 200
mL/h or more for 4 hours, or 500 mL of output in any single
hour warrants a return trip to the operating room for
reexploration rather than imaging to see if there is any need
for surgical hemostasis.
Echocardiography is often technically limited in the immediate
postoperative period. In addition, cardiac tamponade in the
postsurgical patient is a challenging diagnosis to make by
transthoracic echocardiography; visualization is enhanced by
transesophageal imaging but is not 100% sensitive or specific.
An isolated clot can occur behind the heart and cause
tamponade, and this is extremely difficult to visualize with
transthoracic echocardiography. Antifibrinolytics do have a role
in postoperative hemorrhage, but the indications are narrow
and may rely on identifying accelerated fibrinolysis with
techniques such as thromboelastography.
• Question 24:
• In a patient without cardiopulmonary disease or
arteriovenous fistula, an abnormally high oxygen
saturation of blood sampled from the distal lumen of the
pulmonary artery catheter is observed (92%). Which of the
following is the best explanation for this finding?

A)High fraction of inspired oxygen (FIO2)

B)Distal pulmonary artery catheter tip migration

C)Sepsis with decreased VO2

D)PaO2 higher than 100 mm Hg

E)Systemic hypothermia of sepsis


• Correct Answer: B
• Rationale
The distal lumen of the pulmonary artery catheter is
usually located in the pulmonary artery, and the
saturation of blood taken from this lumen represents
the mixed venous oxygen saturation (SvO2). If the
SvO2 is 92%, the corresponding arteriovenous oxygen
difference (at a normal hemoglobin level) is 1.6 mg per
100 mL. It is extremely unlikely for this to occur, even in
the presence of a high cardiac output. Clearly, high
FIO2 or PaO2 cannot explain the observed small
arteriovenous oxygen difference. If the distal lumen
(catheter tip) is in the right ventricle, the saturation of
blood is close to that in the pulmonary artery and would
not be expected to be this high unless the patient has a
left-to-right intracardiac shunt. However, this patient
has no cardiopulmonary disease.
The best explanation is a contamination of mixed
e ous lood a te ialized lood f o the
pulmonary capillaries from a catheter tip that has
ig ated fo a d a d e o e edged i a a h of
the pulmonary artery. The corresponding waveform
from the distal lumen should exhibit pulmonary artery
occlusion pressure waveform characteristics.
• Question 25:
• A 3-month-old gi l o at eeks’ gestatio is
intubated, sedated, and on milrinone, 0.7 µg/kg/min, 10
days after atrioventricular canal repair.
Echocardiography was done for progressive tachycardia
up to 180/min and symptoms of low cardiac output,
with findings of no residual atrial septal defect, small
residual ventricular septal defect, moderate tricuspid
regurgitation, mild mitral valve regurgitation, right
ventricular pressure estimate of 55 mm Hg above the V
wave, depressed right ventricular function, and
preserved left ventricular function.
• Which of the following treatments would be most
beneficial?

A)Increase the milrinone drip.

B)Start nitric oxide at 20 ppm.

C)Start maintenance digoxin.

D)Start a dopamine infusion.

E)Administer a diuretic.
• Correct Answer: B
• Rationale
The echocardiogram indicates elevated pulmonary
vascular resistance based on the tricuspid valve jet and
right ventricular dysfunction. Inhaled nitric oxide is a
specific pulmonary vasodilator that may be useful in
augmenting cardiac output.
• Question 26:
• A 45-year-old man collapses at the airport terminal and
bystander CPR is initiated. An automated defibrillator
present onsite detects ventricular fibrillation. After 2
shocks and CPR, spontaneous circulation returns. He is
intubated by emergency medical services and brought to
the emergency department. He has a pulse rate of 92/min
in sinus rhythm, and BP of 112/70 mm Hg. Spontaneous
movements are minimal. His ventilator settings are assist-
control mode (volume), set RR of 8/min, and tidal volume
of 700 mL (9 mL/kg ideal body weight). His electrolytes are
in the normal range, with serum lactate level of 1.4
mmol/L. Arterial blood gas studies show pH of 7.30,
PaCO2 of 52 mm Hg, and PaO2 of 175 mm Hg on 50% FIO2.
Chest radiography shows endotracheal tube in good
position, with no infiltrates.
• Bedside echocardiography shows an ejection fraction of 55%.
Which of the following measures is most likely to result in
improved outcome?

A)Low-tidal-volume protocol (6 mL/kg ideal body weight) and


increasing the RR to 18/min
B)Hypothermia protocol, with cooling to 32°C-34°C (89.6°F-
93.2°F), for 12-24 hours
C)Placing the patient on an IV sodium bicarbonate infusion to
correct the acidosis
D)Intraaortic balloon pump, set at 1:1 with inflation during
diastole and timing by ECG
E)Hyperbaric oxygen with serial dives to 2.5 atmospheres twice
daily for 72 hours
• Correct Answer: B
• Rationale
In patients who suffer out-of-hospital cardiac arrest and
when the initial rhythm is ventricular fibrillation, cooling
to moderate temperatures of 32°C-34°C (89.6°F-93.2°F),
for 12-24 hours has been shown to be associated with
improved neurologic outcomes and reduced mortality.
The number needed to treat is 6. In spite of these
benefits, hypothermia as a treatment modality is used
infrequently in United States ICUs. The treatment should
be initiated as early as possible, ideally within a few
hours. Multiple methods like external ice packs,
intravascular cooling devices, ice helmets, and vests are
available.
This patie t’s hest adiog aph is lea a d his PaO2 to
FIO2 ratio is 350; he does not have acute lung injury.
Low tidal volume has been shown to decrease mortality
in patients with acute lung injury (bilateral infiltrates
and PaO2 to FIO2 ratio <300). Thus, option A is incorrect.

The patient has mild respiratory acidosis, probably as a


result of his low ventilator rate. The best way to correct
this is by increasing his respiratory rate to normalize the
PaCO2 and pH. IV bicarbonate drip has not been
demonstrated to change prognosis when used to treat
metabolic acidosis—a condition that is not present in
this patient. Correction of mild acidosis has not been
shown to improve outcomes. Therefore, option C is
incorrect.
Intraaortic balloon pump (IABP) improves outcomes
when a patient is in cardiogenic shock. This patient has
an ejection fraction of 55% with a normal blood
pressure and lactate level. IABP in this patient will not
affect outcome, so option D is incorrect.

Hyperbaric oxygen therapy has not been demonstrated


to confer any survival or neurologic performance benefit
after successful resuscitation from out-of-hospital
cardiac arrest.
• Question 27:
• Which of the following is an expected (ie, normal)
change in cardiopulmonary physiology associated with
the third trimester of pregnancy?

A)Increased venous return in the supine position

B)Increased pulmonary artery occlusion pressure

C)Decreased total circulating blood volume

D)Decreased systemic vascular resistance

E)Decreased heart rate with recumbent positioning


• Correct Answer: D
• Rationale
There are a number of important physiologic (and
specifically hemodynamic) effects of pregnancy of which
the critical care clinician should be aware. These would
especially be important if a pregnant patient becomes
hemodynamically unstable and needs cardiovascular
support. Pregnancy is associated with an increase of
total circulating blood volume (as much as 1,500 mL). It
is notable that patients with preeclampsia may not have
this typical degree of increased circulating blood volume
and therefore may be at higher risks for hemodynamic
effects with blood loss associated with delivery.
The gravid uterus can decrease venous return to the
heart when the patient is in the supine or recumbent
position. Therefore, for a pregnant patient with signs of
acute circulatory insufficiency, it is important to place
the patient in the left lateral decubitus position to
facilitate venous return to the heart by displacing the
gravid uterus away from the inferior vena cava. Despite
an increase in total circulating blood volume, there is no
demonstrable increase in pulmonary capillary occlusion
pressure. The heart rate is typically mildly elevated.
Normal physiology of pregnancy will exhibit a decrease
in systemic vascular resistance.
• Question 28:
• Three intubated patients are admitted to the ICU following
exposure to an unknown gas in a bus station during a
presumed terrorist attack. Clinical findings include severe
hypoxemia with copious respiratory secretions and wheezing.
Additional findings include small pupils, bradycardia, and
diarrhea. One of the patients is able to follow commands but
appears to have significant diffuse weakness. Which of the
following interventions would be most beneficial?

A)Pyridostigmine
B)Atropine
C)Sodium nitrite/sodium thiosulfate
D)Hyperbaric oxygen
E)Pralidoxime

Correct Answer: B
• Rationale
Poisoning with nerve gases such as sarin (O-isopropyl
methylphosphonofluoridate) results in a cholinergic
syndrome suggested by the clinical findings. A cholinergic
syndrome can also result from organophosphate or
carbamate poisoning. The toxicity is caused by inhibition
of cholinesterase, with no degradation of acetylcholine at
the postsynaptic receptor. Muscarinic effects include
bronchorrhea, bradycardia, and a hypersecretory
syndrome (salivation, lacrimation, urination, defecation,
emesis). Nicotinic effects result in muscle fasciculations
and weakness, and central nervous system effects lead to
headache, confusion, and central respiratory depression.
Pupils are typically miotic. The primary concern in such
patients is hypoxic respiratory failure from bronchorrhea,
bronchospasm, and respiratory depression.
IV atropine should be administered in doses of 2-4 mg
repeated every 5 minutes until tracheobronchial
secretions are controlled. The most common cause of
death in cholinergic poisonings is asphyxiation. Continued
administration of large doses of atropine may be required
as bolus or continuous infusion. Because sarin irreversibly
binds to cholinesterase, the patient is likely to require
continuous atropine infusion in an ICU setting until the
agent is completely metabolized.
Atropine does not reverse nicotinic effects and
pralidoxime is used to reverse muscle weakness by
regeneration of acetylcholinesterase. It is administered as
a loading dose (1 to 2 g in 500 mL normal saline solution
over 30 minutes) and then as a continuous infusion at 500
mg/h.
Pyridostigmine, an anticholinesterase medication, may
precipitate a cholinergic crisis and is used for treatment of
myasthenia gravis.
Hyperbaric oxygen may be considered for patients with
severe carbon monoxide poisoning, but patients would
not typically have evidence of a hypersecretory syndrome.
Patients with carbon monoxide poisoning will often
present comatose with an anion gap metabolic acidosis,
cardiovascular instability, and increased levels of
carboxyhemoglobin.
Sodium nitrite and sodium thiosulfate are used in
antidotal treatment of cyanide poisoning. Cyanide
poisoning usually presents with more life-threatening
manifestations, such as sudden cardiovascular collapse,
seizures, hypotension, arrhythmias, and severe metabolic
acidosis.
• Question 29:
• A 75-year-old woman with diabetes mellitus and hypertension
was found by her family to be confused and brought to the
emergency department. They noted that she had been
recently diagnosed with cellulitis of her right leg after several
days of high spiking fevers. She had improved significantly after
only 3 days of dicloxacillin. Two days prior, she was eating
without difficulty. Her medications included enalapril,
dicloxacillin, and fludrocortisone.
On examination, the patient is delirious and ill-appearing, with
evidence of dehydration. Her temperature is 37.1°C (98.8°F),
BP is 150/70 mm Hg, HR is 110/min, RR is 14/min, and pulse
oximetry is 95% on room air. Her oral mucosa is dry and the
remainder of her physical examination findings are normal. Her
laboratory findings include the following: sodium, 155 mg/dL;
chloride, 120 mEq/L; potassium, 4.0 mEq/L; bicarbonate, 23
mEq/L; blood urea nitrogen, 30 mg/dL; creatinine, 1.0 mg/dL;
glucose, 200 mg/dL; urine osmolality, 800 mEq/L; urine
sodium, 20 mg/dL.
The ost likel ause of the patie t’s hypernatremia is:

A)Fludrocortisone

B)Nephrogenic diabetes insipidus

C)Hyperglycemia-induced osmotic diuresis

D)Central diabetes insipidus

E)Primary hypodipsia
• Correct Answer: E
• Rationale
Primary hypodipsia of the elderly is the only diagnosis
listed that is consistent with the laboratory values given.
The condition probably developed because of inadequate
thirst mechanism in the setting of excessive water loss
associated with her recent febrile illness. Fludrocortisone,
a mineralocorticoid, increases sodium reabsorption but
should not affect sodium concentration since
hypernatremia can only result from abnormalities in water
balance, including disorders of the thirst center,
hypothalamus (release of antidiuretic hormone), and
kidneys (responsiveness to antidiuretic hormone).
Although not maximally concentrated, the urine
osmolarity excludes diabetes insipidus of either variety as
well as an osmotic diuresis.
• Question 30:
• A patient sustains a severe traumatic brain injury in a
motor vehicle crash. There is evidence of intracranial
hypertension with an intracranial pressure of 24 mm Hg;
a mean arterial pressure of 78 mm Hg is maintained
using pressors. Serum sodium level is 155 mEq/L. Given
cerebral edema on CT, fluids consist only of 3% normal
saline and tube feeds at 50 mL/h; glucose levels are
maintained below 150 mg/dL using an insulin infusion.
On postinjury day 3, urine output increases above 300
mL/h, and diabetes insipidus is suspected.
• Correct Answer: B
• Rationale
Diabetes insipidus is a common clinical condition
encountered in ICUs, particularly in neurological or
neurotrauma ICUs. It causes severe polyuria and can be
life-threatening because of electrolyte and water loss.
Basic quick bedside testing can be lifesaving as it
prompts intervention. A urine specific gravity of 1.005
or less is considered diagnostic in the presence of severe
polyuria that does not have an alternate explanation
such as inadvertent diuretic administration, severe
hyperglycemia, etc. An alternative and more precise
laboratory test for diabetes insipidus is a urine
osmolality that is inappropriately less than 200
mOsm/kg. The concomitant serum osmolality is
generally increased in the setting of dilute urine.
• Question 31:
• A 30-year-old woman has been undergoing fertility
treatments. She has received an ovulation induction
regimen with human chorionic gonadotropin. She is
admitted to the hospital with abdominal pain and
distension, tachycardia, and respiratory difficulty. BP is
110/60 mm Hg in the supine position and 90/52 mm Hg
when she is standing. HR is 118/min while supine and
130/min while standing, RR is 24/min, and temperature
is 36.9°C (98.4°F). Physical examination reveals diffuse
abdominal tenderness, with a moderate amount of
tense ascites. Laboratory tests are remarkable for a
hematocrit of 45%, leukocyte count of 18,000/µL, and
mild elevation of liver transaminase levels. Pulse
oximetry reads at 96% while receiving room air.
• Which of the following is the most likely diagnosis for
this patient?

A)Ectopic pregnancy

B)Ruptured ovarian cyst

C)Pelvic inflammatory disease

D)Ovarian hyperstimulation syndrome

E)Acute Chiari syndrome


• Correct Answer: D
• Rationale
This patient presents with ovarian hyperstimulation
syndrome (OHSS), which is an exaggerated response to
ovulation induction therapy. OHSS is characterized by an
increase in capillary permeability resulting in a fluid shift
from the intravascular space to third space compartments.
Clinical symptoms usually include mild manifestations,
such as transient abdominal discomfort, mild nausea,
vomiting, diarrhea, and abdominal distension. Serious
illness exists when pain is accompanied by rapid weight
gain, ascites, hemodynamic instability, respiratory
difficulty, oliguria, or laboratory abnormalities. This
patie t’s e a i atio e ealed e ide e of orthostasis,
tachycardia, and hemoconcentration, which suggests
intravascular volume depletion. The best initial
management is to replace volume.
Diuretics should only be considered after adequate
intravascular volume has been restored. Intravascular
volume should be replaced before considering a
paracentesis. Paracentesis may be indicated for patients with
ascites and pain, compromised pulmonary function, or
oliguria/anuria unresponsive to fluid resuscitation. In
general, bimanual pelvic examination should be avoided due
to the risk of ovarian rupture. Although ectopic pregnancy
should be considered, the clinical circumstances are more
suggestive of OHSS. In addition, ascites is not present in
ectopic pregnancy. Bleeding into the abdomen can cause
distension but would probably result in a decrease in the
hematocrit or shock for the amount described in this
patient. Ruptured ovarian cysts can cause mild peritoneal
signs but not the hemodynamic or respiratory problems.
Severe pelvic inflammatory disease can cause signs of severe
infection but not ascites.
• Question 32:
• A 75-year-old man with a history of stage IV lung cancer
and advanced emphysema is on mechanical ventilation
after developing a postviral
multilobar Staphylococcus pneumonia. His ICU course has
been complicated by acute respiratory distress syndrome
and multiorgan failure. On hospital day 10, he remains on
maximal ventilator support, with deep sedation and
neuromuscular blockade. His mean arterial pressure is 58
mm Hg, despite norepinephrine, 0.2 µg/kg/min, and
vasopressin, 0.04 U/min. A family meeting is held to
discuss goals of care. His wife and children are in
agreement that the patient would not have wanted to be
sustained on life support indefinitely given his poor
prognosis, and ask that advanced life support be removed
and that the remainder of his care be focused on
measures that maximize comfort.
• Which of the following is the best approach in this end-of-
life scenario?

A)Discontinue neuromuscular blocking agents and


promptly remove the endotracheal tube while he is still
deeply sedated to minimize delay.
B)Titrate down rather than abruptly discontinue
vasopressors, as sudden withdrawal of hemodynamic
support may lead to undue discomfort.
C)Administer opioids using a combination of bolus doses
and infusion to treat both pain and dyspnea as life support
measures are withdrawn.
D)Use judicious doses of benzodiazepines with opioids, as
pharmacologic agents used to treat discomfort frequently
hasten death.
• Correct Answer: C
• Rationale
Opioids are the mainstay of treatment for managing pain
and dyspnea that frequently accompany the dying
process. Particularly with regard to dyspnea, opioids have
been shown to provide relief by simultaneously affecting a
number of important factors, such as work of breathing,
oxygen consumption, sensitivity to hypercapnia, anxiety
associated with breathlessness, and the central perception
of dyspnea. In order to minimize delays in symptom
control, IV bolus doses should be used concurrently when
infusion dose escalation is considered to address signs of
suffering. Therefore, option C is the best response.
Although some have suggested that neuromuscular
blocking agents may be useful to minimize agonal
respirations and allow patients to look more
"comfortable," use of such agents is not acceptable.
• Assessing the level of comfort becomes impossible once
patients are neuromuscularly blocked, and therefore
attainment of the primary goal (ie, minimizing suffering)
cannot be achieved adequately. When patients have
been receiving neuromuscular blocking agents, as in this
case, adequate time should be given to allow the drug
to wear off (or be reversed) before life support is
removed. Therefore, option A is incorrect. In special
circumstances where the neuromuscular blocking action
of the drug is particularly prolonged because of
concomitant organ failures, the burdens of continuing
life support and its impact upon the patient and family
must be considered. If withdrawal of life support is
deemed the best course of action after careful review,
and occurs before restoration of neuromuscular
function, particular attention must be paid to ensure
comfort and appropriate premedication is administered.
There are no studies that have shown that discontinuation
of vasopressors leads to undue burden or discomfort to the
patient, provided appropriate palliation is provided.
Therefore, there is no reason to taper or down-titrate such
medications at the end of life. Option B is thus incorrect.
Benzodiazepines have sedative, hypnotic, anxiolytic, and
amnestic properties and can be a useful pharmacological
adjunct at the end of life. They may have a beneficial,
synergistic effect when used in combination with opioids.
The ethical principle of double effect provides a rationale for
using sedative and analgesic agents so long as the explicit
purpose for titration is to relieve symptoms and suffering.
This applies even if provision of such medications carries the
unintended consequence of hastening death. Recent
research, however, has called into question the relevance of
such concerns in most circumstances by demonstrating little
or no correlation between doses of opioids/benzodiazepines
and survival time. Therefore, option D is incorrect.
• Question 33:
• A 45-year-old man who has progressive amyotrophic lateral
sclerosis is found unresponsive in his wheelchair outside his
home by his neighbor. The paramedics intubate the patient
and transport him to the hospital ICU. On presentation, his
temperature is 38.1°C (100.6°F), pulse rate is 125/min, RR is
35/min, and BP is 85/40 mm Hg. After the administration of
2,000 mL of IV normal saline, his BP improves to 120/85 mm
Hg. Laboratory data reveals WBC count of 17,000/µL. His chest
radiograph shows a new right lower lobe opacity. The patient is
placed on assisted mechanical ventilation with a set rate of
12/min. The patient is breathing at 16/min. His tidal volume is
initially set at 8 mL/kg, FIO2 is 1.0, and positive end-expiratory
pressure is 5 cm H2O. His arterial blood gas studies show pH of
7.35, PaCO2 of 35 mm Hg, and PaO2 of 200 mm Hg. Empiric
ceftriaxone and doxycycline are started. On examination, the
patient is unresponsive to voice despite no administration of
any sedatives. The lungs are clear except for right lower lobe
rhonchi and dullness to percussion.
The patie t’s ife a i es a d epo ts that he hus a d did ot
want to receive mechanical ventilation under any circumstances.
She provides a living will documenting his wishes and requests
that the mechanical ventilation be discontinued. Other family
members arrive and confirm her statements. The patient remains
unresponsive, but his mechanical ventilator requirement
improves. Which of the following is the most appropriate step?

A)Extubate the patient, begin bilevel positive airway pressure,


and continue antibiotics.
B)Obtain an ethics consultation.
C)Inform the wife that the current situation is not chronic
respiratory failure and continue mechanical ventilation and
antibiotics until the patient can state his preferences.
D)Provide adequate sedation and discontinue mechanical
ventilation.
• Correct Answer: D
• Rationale
Option A is incorrect. Bilevel positive airway pressure
constitutes mechanical ventilation and is contraindicated in the
unresponsive patient with the exception of a brief trial for
carbon dioxide narcosis.
Optio B is i o e t. The patie t’s ishes a e lea l stated i
his living will and his wife has requested that his autonomy be
respected.
Option C is incorrect. Pneumonia is a frequent cause of death
i a ot ophi late al s le osis a d the patie t’s p io
statements were to forgo mechanical ventilation under any
circumstances.
Option D is correct. The Cruzan v. Director of MDH decision by
the US Supreme Court allowed withdrawal of support in
settings where the patient is permanently incapacitated or in
pain, not actively suicidal, and where preventing death
requires violation of bodily integrity. These principles were
upheld in the Teri Schiavo case.
• Question 34:
• You are caring for a patient who resided in an extended
care facility due to advanced dementia, chronic
obstructive pulmonary disease, and dysarthria after a
remote stroke. He is 3 weeks status post sigmoid
colectomy and Hartmann procedure for perforated
diverticulitis. His course has been complicated by
pneumonia and ventilator-dependent respiratory failure.
He has developed multisystem organ failure and his sole
surviving relative is a widowed daughter. The daughter
relates that her father would not want to continue
receiving medical care given the poor prognosis with
multisystem organ failure, but would want to receive
comfort care. As a result, the daughter expects her father
to die but believes that he would find a comfortable death
to be an acceptable outcome.
• Which of the following ethical principles best describes
the daughte ’s i te a tio ith ou?

A)Paternalism

B)Substituted judgment

C)Nonmaleficence

D)Power of attorney

E)Legal moralism
• Correct Answer: B
• Rationale
The situation described articulates end-of-life care issues.
Clea l , gi e the patie t’s iti al ill ess o u i g i the
setting of preexisting disability from comorbid diseases, this
patient has a grim prognosis. The daughter has identified a
goal that she believes her father would set for himself if he was
able to interact with the care team and understood what the
daughte u de sta ds ega di g he fathe ’s p og osis. This
principle is termed substituted judgment and describes the
judg e t of o e that is supplied i the pla e of a othe ’s.
Implicit in this principle is the concept of beneficence—that
the judg e t is ei g ade i the patie t’s est i te est. It
also embraces the concept of nonmaleficence—that the action
or decision will not create harm. Fairness is also encompassed
in that the decision derived from substituted judgment is a fair
decision for the patient and therefore also embraces truth.
Truth is expected from the care team and the individual
rendering the substituted judgment.
Paternalism is a philosophic approach to medical decision-
making that relies on physician control instead of patient
autonomy.
Legal moralism is an ethical principle that applies legal
principles and rules as a guide for morality-based decision
making. Legal moralism is the view that the law can
legitimately be used to prohibit behaviors that conflict
ith so iet ’s olle ti e o al judg e ts e e he
those behaviors do not result in physical or psychological
ha to othe s. A o di g to this ie , a pe so ’s f eedo
can legitimately be restricted simply because it conflicts
ith so iet ’s olle ti e o alit ; thus, legal moralism
implies that it is permissible for the state (or other rule-
based agency) to use its coercive power to enforce
so iet ’s olle ti e o alit . Po e of atto e des i es
the legal odifi atio of o e i di idual’s autho it to
render a decision for another.
• Question 35:
• A 58-year-old man is doing yard work on a very hot and
humid day. He feels light-headed and nauseated and
subsequently has multiple episodes of emesis. He feels
better and lies down. He then notes chest discomfort
that continues to worsen over the next 8 hours. He
presents to the emergency department with mild
hypertension, tachycardia, tachypnea, and a
temperature of 38.3°C (100.9°F). His white blood cell
count is elevated, and his ECG is normal except for
tachycardia. Chest radiograph shows a previously
undiagnosed left-sided pleural effusion. A thoracentesis
is performed.
• Which of the following abnormal fluid analyses is most
likely to be present?

A)Elevated glucose level

B)Cholesterol crystals

C)Elevated amylase level

D)High eosinophil count

E)Viscid fluid with low pH


• Correct Answer: C
• Rationale
This patient has Boerhaave syndrome. Boerhaave
syndrome, or spontaneous esophageal rupture, is an
uncommon condition associated with a sudden increase
in intra-esophageal pressure and is classically induced
by forceful vomiting.

Esophageal perforations usually involve the left thoracic


cavity, as anatomically the distal esophagus is directly
beneath the left thoracic pleura. Plain radiography of
the chest may reveal hydrothorax, hydropneumothorax,
or mediastinal emphysema.
The pleural effusion of Boerhaave syndrome is
characterized by an increased amylase level of salivary
etiology; therefore, the fluid is thin and not viscid.
Contrast study is required to locate the exact site of the
perforation. Barium is the contrast medium of choice
because if it is aspirated, it is not inflammatory within the
airway; deliberate aspiration of thin barium was one
previously used modality to diagnose endobronchial
tumors. In contrast, water-soluble contrast is hygroscopic
and inflammatory within the upper airway. Urgent surgery
is typically required as mediastinits carries a high
morbidity and mortality rate with nonoperative
management. Successful surgeries have included primary
closure, reinforced primary closure, and wide mediastinal
drainage if presentation after perforation is delayed. Some
cases may require management with esophageal resection
and conduit interposition such as the stomach or colon.
• Question 36:
• A 58-year-old man with type 2 diabetes mellitus is
admitted with abdominal pain and distension. His BP is
85/48 mm Hg, HR is 133/min, and his temperature is
38.8°C (101.8°F). His WBC count is 27,800/µL and his
lactate level is 5.4 mmol/L. Abdominal CT shows a
dilated, thickened colon with pericolonic fat stranding.
He recently completed 6 weeks of IV antibiotics for an
incompletely healed but no longer infected foot ulcer.
He is admitted to the ICU and started on metronidazole
and cefotaxime and resuscitation is begun.
• Which of the following is the most appropriate
management for his condition?

A)IV vancomycin

B)Oral and rectal vancomycin

C)Emergent surgical consultation

D)Infectious disease consultation

E)IV nitazoxanide and probiotics


• Correct Answer: C
• Rationale
This patient is critically ill from a fulminant colitis.
Recent treatment with antibiotics strongly
suggests Clostridium difficile as the offending
organism. Metronidazole and vancomycin along with
several new agents are effective in treating C
difficile colitis, including nitazoxanide; no added
efficacy derives from adding probiotics to the
regimen. However, fulminant colitis has a reported
mortality up to 80%. Significant predictors of
mortality include vasopressor requirement, WBC
count greater than 20,000/µL, elevated lactate level,
age over 75, and delay to surgical intervention.
Standard treatment for this patient is an emergent
subtotal total colectomy (rectum is left in place)
after he is resuscitated.
Those with severe complicated (ie, fulminant) colitis
may alternatively be well treated by a novel approach
that includes laparoscopic loop ileostomy and distal
washout of the colon with 7 L of a lavage solution such
as warmed polyethylene glycol 3350/electrolyte
solution to decrease colonic bacterial and toxin burden,
followed by luminal vancomycin therapy via the
ileostomy. This approach leads to colonic preservation
and reduced mortality compared to historic controls.
• Question 37:
• A 36-year-old man is involved in a 2-car collision and rollover.
He sustains bilateral femur fractures and a traumatic brain
injury consisting of a subdural hematoma, as well as a hepatic
laceration (American Association for the Surgery of Trauma
[AAST] organ injury scale grade III). The most appropriate
nutritional prescription on postinjury day 2 is:

A)25 kcal/kg body weight (bw) total calories and 0.8 g/kg bw
protein
B)A formula providing a nonprotein calorie to nitrogen ratio of
125:1
C)30 kcal/kg bw total calories and 1.5 g/kg bw protein
D)35 kcal/kg bw total calories and 1.5 g/kg bw protein
E)30 kcal/kg bw total calories and 2.0 g/kg bw protein
• Correct Answer: E
• Rationale
This patient has suffered traumatic brain injury (TBI), a
condition known to increase metabolic rate and augment the
need for both nonprotein and protein calorie supplementation.
The TBI is further complicated by long bone fractures and solid
o ga i ju , fu the i easi g the patie t’s eed fo
nutritional support. Option A provides the baseline needs for
nutritional support of an uninjured and healthy individual.
Similarly, option B provides a nonprotein calorie to nitrogen
ratio that is between uninjured and healthy (150:1) and
seriously injured (approximately 100:1). Options C and D both
provide too little protein, although C provides the correct
amount of nonprotein calories to avoid excessive carbohydrate
and fat administration. Therefore, only answer E provides the
correct amount of both nonprotein and protein calories to a
patient with the detailed conditions.
• Question 38:
• A 27-year-old man is admitted to the ICU after an
exploratory laparotomy and a distal small bowel resection
following a gunshot wound to the abdomen. He remains
intubated. A nasojejunal feeding tube was placed at the
time of surgery. Which of the following nutritional
strategies is most appropriate in this case?

A)Hold feeding until bowel function objectively returns.


B)Begin enteral feedings 48 hours postoperatively.
C)Begin enteral feeding immediately postoperatively.
D)Start total parenteral nutrition on postoperative day 2.
E)Start immediate total parenteral nutrition plus only
trophic enteral nutrition.

Correct Answer: C
• Rationale
Early enteral feeding (within 36 hours of admission or
surgery) is associated with fewer infectious complications
and lower mortality following abdominal operations.1 "In
the ICU patient population, neither the presence nor
absence of bowel sounds nor evidence of passage of flatus
or stool is required for the initiation of enteral feedi g. 2In
the previously healthy patient, no nutrition is superior to
total parenteral nutrition (TPN) during the first 7
days.3,4After 14 days, TPN is superior to no nutrition.5 No
studies have examined the use of TPN between 7 and 14
days; therefore, TPN should probably be started after 7
days if enteral feeding is unlikely to succeed in the next 7
days. There is no role for TPN plus trophic luminal feeds in
this setting.
• Question 39:
• A 65-year-old, male smoker is in the ICU with respiratory
failure secondary to left lower lobe pneumonia. He
requires a norepinephrine drip to maintain his mean
arterial pressure and has an elevated blood glucose level
that requires an insulin drip.
Which of the following statements effectively describes
the ha ges i this patie t’s eta oli state?

A)Resolution of his negative nitrogen balance is


dependent on providing adequate parenteral nutrition.
B)Resolution of his proteolysis is dependent on providing
adequate enteral nutrition.
C)Resolution of this hypermetabolic state will occur by
withholding enteral nutrition.
D)Resolution of his negative nitrogen balance is
dependent on effective treatment of his pneumonia.
E)Resolution of his hyperglycemia will occur by
withholding enteral nutrition.
• Correct Answer: D
• Rationale
The metabolic demands of critically ill patients significantly
increase because of changes in energy expenditure. This
hypermetabolic state is different than a prolonged fasting
state, in which the energy expenditure can decrease by as
much as 20%. Infection, injury, and the resulting inflammation
increase the circulating cytokines. These immunomodulators
stimulate changes in the normal levels of catecholamines,
cortisol, and glucagon. The elevated concentrations of these
hormones push tissue proteolysis, a negative protein balance,
and increase gluconeogenesis and acute-phase protein
p odu tio i the li e . The od ’s p otei a d e e g ala e
remains negative until the underlying disease or injury process
has been effectively treated. Insulin resistance is multifactorial
and includes decreased cell-surface concentration of the
receptor, binding, and intracellular signaling.
• Question 40:
• A 42-year-old woman with no significant past history is
admitted with profound hypoxemic respiratory failure
due to H1N1 influenza pneumonia. Her clinical course is
complicated by septic shock, acute respiratory distress
syndrome, and acute kidney injury. At the time of
admission, she is well nourished with a body mass index
of 25 and weight of 50 kg (110 lbs). On day 2, in the
context of progressive renal failure and shock, she is
started on continuous venovenous hemofiltration. Her
hemodynamic parameters are improving, but she
remains on norepinephrine at 0.04 µg/kg/min (or 2
µg/min). She is sedated and intubated. Her lung sounds
are coarse with crackles and her abdomen is soft and
nontender, with minimal bowel sounds.
• Which of the following describes the best nutritional support
strategy for this patient?

A)Delay initiation of enteral nutrition until bowel sounds are


consistently heard to improve tolerance and minimize aspiration.

B)Start enteral nutrition early, but choose a formulation that is


fluid and protein restricted given her renal failure.

C)Delay introduction of enteral nutrition until hemodynamic


stability is achieved to avoid bowel ischemia.

D)Select an enteral formula with omega-3 fatty acids and borage


oil (gamma-linolenic acid).
• Correct Answer: D
• Rationale
Three prospective, randomized controlled studies have
now shown that use of an enteral formulation fortified
with antioxidants and an antiinflammatory lipid profile
(consisting of omega-3 fatty acids and borage oil) can
improve outcomes among mechanically ventilated
patients with severe sepsis as well as those with acute
lung injury or acute respiratory distress syndrome.
Clinically important morbidity end points, such as ICU
length of stay, ventilator days, and development of
organ failure, as well as mortality, have been
demonstrated to improve with such specialized
formulations. Therefore, option D is the best answer.
Prior studies have shown that among critically ill
patients, presence of bowel sounds and evidence of
flatus and/or stool are a poor predictor of tolerance to
enteral nutrition. Thus, these examination findings are
not a requirement for initiating enteral nutritional
support, provided parameters such as nausea, vomiting,
diarrhea, excessive residuals, abdominal tenderness,
and distension are monitored carefully. Therefore,
option A is incorrect.
When there is concern regarding fluid status or
electrolyte derangements, specialized formulas
targeting those with renal failure can be useful.
However, once renal replacement therapy is initiated,
fluid and electrolyte restrictions become less important.
Critically ill patients on continuous renal replacement
therapy (CRRT) such as continuous venovenous
hemofiltration should receive increased protein, as
significant amino acid losses may occur in the CRRT
effluent. Furthermore, critical illness is a catabolic state
and protein is used as the preferred fuel source.
Guidelines therefore suggest that 1.5-2.0 g/kg/day of
protein be delivered to patients on CRRT. In one
randomized prospective trial, 2.5 g/kg/day of protein
was necessary to achieve a positive nitrogen balance in
this subset. For these reasons, option B is incorrect.
Enteral feeding during significant hemodynamic
compromise (eg, high-dose vasopressors, alone or in
combination with aggressive fluid or blood product
resucitation) places the patient at risk of developing
bowel ischemia. There are 2 purported mechanisms: (1)
redistribution of cardiac output to the splanchnic
circulation leading to reperfusion injury, and (2) further
compromise of the mean arterial pressure as the cardiac
output is diverted. Despite theoretical concerns,
ischemia/reperfusion injury due to initiation of nutrition
is a rare complication and published protocols have
shown safety of enteral nutritional support, particularly
among those who are improving or on stable low doses
of vasopressor agents (as in this case). Option C is
incorrect
• Question 41: Both questions 41 and 42 use the
information below:

A 60-year-old, 60-kg (132-lb), previously healthy man


presents to the ICU following endoscopic retrograde
cholangiopancreatography for an impacted stone in the
common bile duct. Twelve hours later, he becomes
febrile and dyspneic and requires intubation due to
worsening hypoxemia. Laboratory examination shows a
WBC count of 3,000/µL with 20% band cells, amylase
level of 2,000 U/L, and lipase level of 8,000 U/L. A
portable chest radiograph obtained after intubation is
shown in the Figure.
• Which of the following findings is most consistent with
this presentation?

A)Increase in lung surfactant

B)Left atrial pressure <18 mm Hg

C)Increased lung compliance

D)PaO2 to FIO2 ratio >400 mm Hg

E)Decreased extravascular lung water


• Correct Answer: B
• Rationale
Acute respiratory distress syndrome (ARDS) consists of
acute development of severe hypoxemia (ratio of PaO2 to
FIO2 <200 mm Hg), bilateral infiltrates on chest
radiography, and absence of elevated left atrial pressure.
ARDS can develop as a result of a lung injury that is direct
(eg, pneumonia, aspiration, inhalational injury) or indirect
(pancreatitis, sepsis, severe trauma with shock,
transfusion of blood products). Pathophysiology of ARDS is
characterized by increased epithelial and endothelial
permeability resulting in diffuse alveolar damage and
formation of hyaline membrane.
The chest radiograph (Figure) shows bilateral alveolar
infiltrates. Radiographically, the findings are
indistinguishable from those of cardiogenic pulmonary
edema.
Pulmonary artery wedge or left atrial p essu e
Hg, or the absence of clinical evidence of left atrial
hypertension in a patient with severe hypoxemia is
consistent with ARDS. Hence, option B is the correct
response. The loss of epithelial integrity and injury to type
II alveolar cells causes alveolar edema and decrease in
lung surfactant, and the resultant decrease in lung
compliance seen in ARDS. Thus, options A and C are
incorrect. The severity of clinical lung injury is defined by a
ratio of PaO2 to FIO2: patients with less severe hypoxemia
(PaO2/FIO2 of 200-300 mm Hg) are considered to have
acute lung injury, and those with more severe hypoxemia
(PaO2/FIO2 <200 mm Hg) are considered to have ARDS.
Therefore, option D is incorrect. Moreover, lungs that are
acutely inflamed have leaky capillaries and have an
increase, not a decrease in extravascular lung water.
• Question 42:
• A 60-year-old, 60-kg (132-lb), 165-cm (5-ft-5-in)
previously healthy man presents to the ICU following
endoscopic retrograde cholangiopancreatography for an
impacted stone in the common bile duct. Twelve hours
later, he becomes febrile and dyspneic and requires
intubation due to worsening hypoxemia. Laboratory
examination shows a WBC count of 3,000/µL with 20%
band cells, amylase level of 2,000 U/L, and lipase level
of 8,000 U/L. A portable chest radiograph obtained after
intubation is shown in the Figure.
He continues receiving assist-control mechanical ventilation
with RR of 14/min, tidal volume of 500 mL, FIO2 of 0.7,
positive end-expiratory pressure (PEEP) of 10 cm H2O, on a
square waveform and a peak flow of 60 L/min. Arterial
blood gas studies show pH of 7.38, PaCO2 of 40 mm Hg,
and PaO2 of 80 mm Hg. Plateau pressure is 35 cm
H2O. Which of the following is the most appropriate first
intervention in the ventilator management?

A)Increase PEEP to 15 cm H2O.


B)Increase FIO2 to 1.0.
C)Decrease tidal volume to 360 mL.
D)Add an inspiratory pause.
E)Change to a flow rate of 40 L/min.
• Correct Answer: C
• Rationale
Lung-p ote ti e e tilatio usi g lo tidal olu e
mL/kg of predicted body weight), along with adequate
positive end-expiratory pressure (PEEP), and limiting
transpulmonary distending pressure (plateau pressure
H2O after a 0.5-second end-inspiratory pause)
aims to minimize ventilator-associated lung injury
(resulting from alveolar overdistension, volutrauma) or
repeated opening and closing of individual lung units
(atelectrauma). The Acute Respiratory Distress
Syndrome (ARDS) Network study demonstrated an
absolute risk reduction in short-term mortality of nearly
9% in patients receiving this pressure- and volume-
limited strategy.
The main problems in this patient with ARDS are large
tidal volume, 500 mL (8.3 mL/kg) and poor pulmonary
mechanics (high plateau pressure, 35 cm H2O). The most
appropriate first intervention to improve plateau
pressure should be to reduce tidal volume to 360 mL (6
mL/kg), which is consistent with ARDS lung-protective
ventilation. The resultant permissive hypercapnia would
be acceptable. However, ventilator frequency (f) can be
adjusted for a desired PaCO2 using the equation:

• Desired f = (Actual f) (ActualPaCO2)/Desired PaCO2.


The peak airway pressure (PaW), plateau pressure (Ppl),
and change in peak airway pressure (dPaW, cm H2O) are
significantly lower using the decelerating inspiratory flow
waveform compared with sine or square waveform
patterns. Inspiratory time/total ventilation cycle time
(Ti/Ttot) is also lower with the decelerating waveform with
better dynamic compliance compared with the other
waveforms. Decelerating waveform and inspiratory pause
(20%) both prolong time for gas diffusion in the alveoli and
could enhance carbon dioxide elimination,
reduce PaCO2, and improve oxygenation in patients with
acute lung injury. Lower inspiratory flow rate is lung-
protective in ARDS. This can be considered once tidal
volume is reduced to 6 mL/kg. High inspiratory flows
increase tensile stress, resulting in transmission of kinetic
energy to underlying structures. They also increase shear
stress, with parenchymal distortion and deformation of
the epithelial surface.
During mechanical ventilation with high peak inspiratory
pressure, high pulmonary airflow triggers microvascular
injury, whereas reduced inspiratory flow leads to
protection and a lower degree of shunt, histological
injury, pulmonary neutrophil infiltration, and alveolar
edema.
The PaO2 is adequate on arterial blood gas, so increase
in PEEP or FIO2 is not warranted at this point.
• Question 43:
• A 29-year-old man is admitted with a fractured femur
and a traumatic brain injury. He is intubated with an
intracranial monitor in place. He is 178 cm (5 ft 10 in)
tall and weighs 85 kg (189 lbs). The best nutritional
prescription for him is:

A)1,700 kcal and 73 g of protein daily

B)1,700 kcal and 120 g of protein daily

C)2,400 kcal and 73 g protein daily

D)2,400 kcal and 120 g of protein daily


• Correct Answer: B
• Rationale
During an acute inflammatory response, a patient will
not utilize all the exogenous substrate supplied. Several
studies have shown that patients who receive less
calories than their expected need based on
measurement or estimation of their metabolic rate have
better outcomes. For example, despite brain-injured
patients having metabolic rates up to 140% of their
basal metabolic rate, this patient should receive only 20-
25 kcal/kg/day. Current guidelines recommend 1.2-2.0 g
protein/kg/day.
• Question 44:
• Fresh frozen plasma most effectively corrects which of
the following conditions?

A)Gamma globulin deficiency

B)Antithrombin III deficiency

C)Polycythemia vera

D)Hypofibrinogenemia

E)Dabigatran overdose
• Correct Answer: B
• Rationale
Fresh frozen plasma (FFP) includes all coagulation factors,
including antithrombin. For patients with an antithrombin
deficiency who experience bleeding or require surgery,
FFP may be used to effectively replace antithrombin.
Fu the o e, patie ts ho appea to ha e hepa i
esista e a e antithrombin III deficient and are
seemingly paradoxically corrected by FFP infusion.
Additional indications for transfusion of FFP include (a)
coagulopathy due to a documented deficiency of
coagulation factors with active bleeding or planned
invasive procedures, (b) massive blood transfusion with
evidence of a coagulation deficiency, (c) reversal of
warfarin effect, and (d) plasma exchange for thrombotic
thrombocytopenic purpura or hemolytic uremic
syndrome.
Rapid infusion is needed to achieve appropriate factor
levels for hemostasis. The speed of infusion does not
increase adverse reactions. Continuous infusion of FFP is
inappropriate since it does not usually result in
adequate factor levels. FFP may need to be
administered every 6 to 8 hours, since the half-life of
factor VII is approximately 6 hours. Smaller doses of FFP
may be needed if platelets are also transfused. The
processing of platelets results in the presence of plasma,
and for every 5 to 6 U of random donor platelets, the
patient may receive the equivalent of 1 bag of random
donor FFP. FFP is not an efficient source of fibrinogen.
For fibrinogen levels less than 100 mg/dL,
cryoprecipitate is the best source of fibrinogen. FFP
does not reverse dabigatran.
• uestion 45:
• A female patient with a history of heparin-induced
thrombocytopenia and lower extremity deep venous
thrombosis is receiving argatroban for treatment. She
has a history of hepatitis C and moderate hepatic
insufficiency, but normal renal function.
Which of the following best characterizes
recommended laboratory testing to best monitor
a gat o a ’s anticoagulant activity?

A)Prothrombin time/international normalized ratio

B)Activated partial thromboplastin time

C)Lee-White bleeding time determination

D)Antifactor Xa level determination

E)No routine laboratory testing


• Correct Answer: B
• Rationale
Argatroban is a direct thrombin inhibitor and is
approved for the treatment of thrombosis secondary to
heparin-induced thrombocytopenia. Anticoagulant
activity is monitored with activated partial
thromboplastin time (aPTT). Dosing is adjusted to
maintain an aPTT value 1.5-3 times the initial baseline
value. Argatroban can also increase prothrombin time
and international normalized ratio values. Dosing in
patients with hepatic impairment should be adjusted as
patients will have decreased clearance and increased
elimination half-life. Dose adjustment is not required in
the presence of renal impairment.
• Question 46:
• A 76-year-old man arrives intubated in your ICU after an
uncomplicated left hemicolectomy for diverticular
disease with operating room time of 5 hours and
estimated blood loss of 600 mL. His initial platelet count
is 315,000/µL. He receives venous thromboembolism
prophylaxis with unfractionated heparin at a dose of
5,000 U subcutaneously 3 times per day. He weighs 88
kg (196 lbs) and has no other significant past history
besides tobacco use. Three days postoperatively, his
platelet count is 62,000/µL and he is net 6.4 L crystalloid
fluid positive. Heparin-induced thrombocytopenia is
suspected.
• The best initial course of action is to:

A)Continue unfractionated heparin as the


thrombocytopenia is most likely dilutional
B)Discontinue unfractionated heparin and start a low-
molecular-weight heparin
C)Re-cross match the patient and transfuse single donor
platelets to >100,000/µL
D)Stop heparin, send a heparin antibody assay, and start
direct thrombin inhibitor
E)Obtain an urgent lower extremity duplex study to assess
for deep vein thrombosis
• Correct Answer: D
• Rationale
This patient may have heparin-induced
thrombocytopenia (HIT), although most patients with
thrombocytopenia do not; other causes including sepsis,
bone marrow suppression, and platelet consumption
are much more common causes of thrombocytopenia
compared to HIT. Establishing the diagnosis awaits
confirmatory laboratory testing. If the patient requires
venous thromboembolism prophylaxis, as this one does
(abdominal and pelvic surgery, advanced age, bed rest),
then prophylaxis should be continued while the
diagnosis is being ruled in or out. Discontinuing heparin
is essential if the patient truly has HIT, but should be
coupled with sending an appropriate antibody assay.
These maneuvers should be accompanied by a direct
thrombin inhibitor to provide venous thromboembolism
(VTE) prophylaxis while awaiting the results of the
laboratory testing. While it is true that we have no
effective therapy for established HIT, providing VTE
prophylaxis is an intervention that should be utilized,
rendering only option D correct.
• Question 47:
• A 47-year-old man weighing 85 kg (188 lbs) is admitted
for an exacerbation of chronic obstructive pulmonary
disease. He has no other medical history and his only
medications are inhaled steroids and an inhaled long-
acting beta-adrenergic agonist. He requires central
venous catheterization for vascular access. His
international normalized ratio on admission is 1.54.
• The best course of action is to:

A)Proceed with placement of a central venous
catheter
B)Administer plasma, 2 U, and then place the
catheter
C)Administer plasma, 4 U, and then place the
catheter
D)Administer vitamin K, 10 mg, and wait 24 hours
before placing the catheter
E)Administer factor VIIa, 7.7 mg, and then place the
catheter
• Correct Answer: A
• Rationale
Preprocedural prothrombin time (PT) and activated partial
thromboplastin time do not predict procedural bleeding. These
tests were developed to test for abnormal factor levels in
patients with clinically evident bleeding. Furthermore,
increased sensitivity of modern PT reagents means that the
international normalized ratio will be abnormal even with only
mild factor deficiencies, which are rarely clinically
relevant.1 Because the recommended dose of plasma is 15-30
mL/kg, plasma transfusion is associated with an increased risk
of transfusion-associated circulatory overload. Plasma
transfusion is also implicated in transfusion-related acute lung
injury2 and in an increased risk of infectious
complications.3 Vitamin K and factor VIIa have no role, since
the laboratory abnormality does not require treatment. The
risk of complications from central venous catheterization is
more strongly related to operator experience.4 Therefore,
option A is the best answer.
• Question 48:
• An elderly patient is admitted to the ICU via the
emergency department 24 hours after falling from a
rooftop. His only injuries are multiple bilateral rib
fractures (5 on the right and 6 on the left). He reports
poor pain control with IV opioids, his oxygen saturation
is 90% on a 100% nonrebreather mask, and he is taking
rapid, shallow breaths at a rate of 28/min. You
determine that the patient would benefit from
placement of a thoracic epidural catheter for pain
management. The nurse says he is receiving enoxaparin
for deep venous thrombosis prophylaxis and received
his last dose 2 hours ago.
• Regarding the timing of placement for the epidural
catheter, you should:

A)Proceed with epidural catheter insertion now

B)Wait 10-12 hours after the last dose of enoxaparin

C)Wait 24 hours after the last dose of enoxaparin

D)Wait 7 days after the last dose of enoxaparin

E)Place the catheter after giving prothrombinase


complex
• Correct Answer: B
• Rationale
Guidelines published by the American Society of
Regional Anesthesia and Pain Medicine (ASRA)
recommend delaying neuraxial analgesia at least 10-12
hours after thromboprophylactic dosing of low-
molecular-weight heparins because of concerns of
epidural or spinal hematoma formation. For higher
(treatment) doses of low-molecular-weight heparins, it
is recommended to delay the procedure for at least 24
hours to ensure normal hemostasis at time of needle
insertion. There is no role for prothrombinase complex
in reversing low-molecular-weight heparin.
• Question 49:
• A 55-year-old man arrives in the emergency
department after a witnessed episode of syncope that
occurred when he was getting up from his chair. He did
not hit his head and no jerky movements were
observed. He regained consciousness in less than a
minute and no confusion was noted. On presentation,
his BP is 75/45 mm Hg, HR is 95/min, and SaO2 is 90%
on room air. He is alert and oriented, with no neurologic
deficits. His only concern is that he feels weak. Findings
of head CT are normal; serum electrolyte levels and
complete blood counts are normal. No ST-segment
elevation is present on ECG. Contrast CT of the chest is
shown in the Figure.
He has no medical history and is not on any medications.
History is significant for a transatlantic flight a week ago and a
brother who is on antiepileptic medications. He is given 2 L of
normal saline boluses with no response in either his heart rate
or blood pressure. Which of the following is the best next
treatment option?

A)Heparin, 5,000 U subcutaneously 3 times a day, followed by


full-dose warfarin
B)IV rtPA followed by IV heparin for anticoagulation
C)IV fosphenytoin followed by oral phenytoin, 100 mg orally 3
times a day
D)Cardiology referral for permanent defibrillator placement
E)Aspirin, 325 mg, followed by dabigatran, 150 mg orally twice
a day
• Correct Answer: B
• Rationale
The CT of the chest shows a saddle embolus. His risk
factor for embolism is the long-distance flight he took a
week ago. IV thrombolytics are indicated for pulmonary
emboli large enough to cause persistent hypotension. A
few small studies have shown benefit in patients who
have right ventricular strain/failure secondary to emboli.
In these patients there is faster resolution of the right
ventricular strain/failure. However, there is no
advantage in mortality. Thrombolytics are sometimes
used in refractory hypoxemia.
The patient needs to have his lower extremities scanned to
identify the source of pulmonary emboli. If a lower
extremity clot is found, given that his clot burden was
large enough to cause syncope, he may benefit from a
temporary inferior vena cava filter placement. The IV
thrombolytic needs to be followed by IV heparin. The side
effects of the thrombolytic include bleeding (intracranial,
retroperitoneal, or generalized). Bleeding can be severe
enough to require blood transfusion and can result in
death.
IV heparin or low-molecular-weight heparin in full dose is
used if a contraindication exists for thrombolytic use.
Warfarin needs to be overlapped with full anticoagulation.
It can be started the same day. However, given the large
clot burden it is prudent to start it after a day of full
anticoagulation and to continue overlapping warfarin and
heparin or low-molecular-weight heparin till therapeutic
anticoagulation is achieved.
Ou patie t’s s ope episode is due to the saddle
emboli to the lungs. He needs to be monitored in the
ICU for any arrhythmias, but at this time there is no
indication for defibrillator placement.

Generalized seizures can result in syncope. In our


patient the episode was witnessed, no abnormal
movements were witnessed, and the patient had no
postictal state. Given the CT findings, it is highly unlikely
that seizures were responsible for this syncope and the
subsequent hypotension.
• Question 50:
• A 30-year-old pregnant woman presents to the hospital
at 35 weeks of gestation with headache, nausea,
vomiting, and weakness. She is lethargic on
examination. BP is 148/90 mm Hg, HR is 108/min, RR is
18/min and nonlabored, temperature is 37.5°C (99.4°F),
and room air saturation is 99%; portable chest
radiograph findings are normal as are arterial blood gas
results. She is found to have thrombocytopenia
(platelets 70,000/µL) and evidence of microangiopathic
hemolytic anemia and is diagnosed with thrombotic
thrombocytopenic purpura.
• Which of the following is the most appropriate initial
intervention?

A)Delivery of the fetus

B)Subcutaneous terbutaline

C)Total plasma exchange

D)Urgent neurology consultation

E)IV labetalol
• Correct Answer: C
• Rationale
Thrombotic thrombocytopenic purpura (TTP) is a life-
threatening emergency with a mortality of greater than
90% if not treated. The treatment of choice is plasma
exchange. In many cases of idiopathic TTP, the
thrombocytopenia and microangiopathic hemolytic
anemia are the result of circulating large von
Willebrand factor (vWF) multimers. These multimers are
usually degraded by a specific vWF cleaving protease—
ADAMTS13, but patients with TTP are often found to
have low levels of ADAMTS13 activity, or an antibody
directed against ADAMTS13. Plasma exchange removes
both the circulating vWF multimers and any inhibitors,
and provides the missing protease.
While the classic pentad of TTP includes
thrombocytopenia, microangiopathic hemolytic
anemia, acute renal insufficiency, neurologic
abnormalities (usually fluctuating), and fever, the
diagnosis of TTP requires only thrombocytopenia and
microangiopathic hemolytic anemia, in the absence of
an alternative clinically apparent etiology. This presents
challenges in the pregnant patient since both
thrombocytopenia and microangiopathic hemolytic
anemia can also be present in severe preeclampsia and
HELLP (Hemolysis, Elevated Liver enzymes, Low
Platelets) syndrome. Headache is a relatively common
symptom in both preeclampsia and HELLP, but mental
status changes are rare.
The definitive treatment for preeclampsia and HELLP
syndrome is delivery. While delivery does not generally
cause resolution of TTP, in the pregnant patient with
suspected TTP, in addition to plasma exchange, delivery
should be considered if the disease is severe and the
fetus is viable, since this will resolve the preeclampsia
and HELLP syndrome, which may be difficult to
distinguish from TTP. Subcutaneous terbutaline is used
to inhibit preterm labor and is not indicated in this
patient.
• Question 51:
• An 80-year-old, 60-kg (132-lb), woman admitted to the
ICU is receiving therapeutic dalteparin for a recently
diagnosed venous thromboembolism. She has a history of
hepatitis C and moderate hepatic insufficiency, but normal
renal function. Which of the following best characterizes
recommended laboratory testing to monitor daltepa i ’s
anticoagulant activity?

A)Prothrombin time/international normalized ratio


B)Activated partial thromboplastin time
C)Bleeding time
D)Antifactor Xa level
E)No routine laboratory testing
• Correct Answer: E
• Rationale
Dalteparin is a low-molecular-weight heparin (LMWH)
approved for the prophylaxis and treatment of deep
venous thrombosis. Because the anticoagulant response
(antifactor Xa activity) to a fixed dose of LMWH is highly
o elated ith the patie t’s od eight, la o ato
monitoring of the anticoagulant activity and dosing
adjustment is unnecessary except in cases of pregnancy,
morbid obesity, or renal impairment. If one or more of
these conditions is present, antifactor Xa levels should
be followed for correct dosing.
• Question 52:
• A 65-year-old woman with a history of diabetes, obesity, and
obstructive sleep apnea is on mechanical ventilation following
elective knee replacement surgery. Her hospital course has
been complicated by respiratory failure and ventilator-
associated pneumonia. She has been receiving subcutaneous
enoxaparin, 30 mg every 12 hours for deep vein thrombosis
prophylaxis, as well as piperacillin/tazobactam for pneumonia.
On postoperative day 6, her platelet count declines to
90,000/µL from a preoperative value of 200,000/µL. Because of
concern for heparin-induced thrombocytopenia (HIT),
enoxaparin is stopped and she is started on argatroban. Lower
extremity ultrasonography confirms the presence of a deep
vein thrombus in the left common femoral vein. On day 8, her
platelet count has risen to 110,000/µL, and warfarin therapy is
begun. On day 10, her international normalized ratio is 3.5 and
argatroban is discontinued. Two days later, she reports severe
pain in her toes and has new lesions as shown in the Figure.
On examination, she is awake and alert and breathing
comfortably. Pedal pulses are palpable, and there are no
petechial or bullous lesions. Which of the following is
the most likely cause of these lesions?

A)Warfarin-induced skin necrosis

B)Venous limb gangrene

C)Fat embolism syndrome

D)Acute arterial occlusion due to inadequate HIT


treatment
• Correct Answer: B
• Rationale
Unlike the management of deep venous thrombosis that is
unrelated to heparin-induced thrombocytopenia (HIT),
where warfarin therapy is often initiated early after
starting unfractionated or low-molecular-weight heparin,
it is important to delay initiation of warfarin therapy in
patients with HIT. Because of sharp reductions in proteins
C and S, warfarin has a net procoagulant effect early on.
Introduction of warfarin before thrombosis is well
controlled can precipitate devastating thrombotic events
among patients with HIT. Venous limb gangrene typically
appears in the distal limbs and develops despite palpable
pulses, often in the context of a supratherapeutic
international normalized ratio (INR). Extensive thrombosis
is noted in the large and small veins, without arterial
occlusion. Venous limb gangrene usually complicates an
acute ipsilateral deep venous thrombus in the leg.
• Guidelines therefore recommend (1) initiating
warfarin therapy only after the platelet count has
recovered to at least 150,000/µL as this signals
that the acute platelet-consuming thrombotic
process is under control; (2) overlapping warfarin
with alternative anticoagulant therapy for at
least 4-5 days; and (3) discontinuing the
alternative anticoagulant only after 2
international normalized ratios checked 24 hours
apart are within target range. Option B is correct.
Option A, warfarin-induced skin necrosis, is a rare
complication of oral anticoagulation therapy. It has
been linked most frequently to hereditary
deficiencies of protein C, but has also been reported
to occur in patients with deficiencies of protein S
and antithrombin, resistance to activated protein C
(factor V Leiden), and the antiphospholipid antibody
syndrome. Clinically, it typically appears 3 to 7 days
after warfarin initiation, and manifests as painful,
erythematous areas on the skin with petechiae and
bullae. It classically involves the more central, fatty
tissues, such as the abdomen, thighs, and breasts.
The appearance and location of the lesion therefore
is inconsistent with the one in this case.
Option C, fat embolism syndrome, classically manifests 1-
3 days after a long bone or pelvic fracture (although
nontraumatic causes have also been described). Patients
present with a triad of hypoxemia, petechial rash, and
neurologic abnormalities. Although recent studies have
investigated the use of bronchoalveolar lavage to detect
fat droplets in alveolar macrophages, no test has been
shown to have sufficient sensitivity or specificity to include
or exclude the diagnosis. As such, the diagnosis is made on
clinical grounds in the appropriate setting and treatment is
largely supportive. The patient in question does not have
skin manifestations that are consistent with fat embolism.
Although orthopedic procedures have been linked with fat
embolism, the time course here (postoperative day 12) is
inconsistent—and she has no neurologic or pulmonary
manifestations to fit the triad.
Option D, acute arterial occlusion, can complicate HIT,
but less commonly than venous thrombosis. (In some
series, venous thromboses occurred 4 times more
frequently than arterial events.) Arterial thrombus
("white clot") can manifest in myriad ways and may
include stroke, myocardial infarction, limb ischemia, or
organ infarction (eg, kidneys). The patient in question
clearly has palpable pulses, making acute arterial
occlusion less likely.
• Question 53:
• A 65-year-old man with diabetes mellitus and
hypertension is admitted to the ICU for cholangitis-
related sepsis. He is intubated and requires mechanical
ventilation for acute lung injury within 12 hours of
admission. Three days later, he develops deep venous
thrombosis of the right lower extremity and IV
(therapeutic) heparin is started. The patient is receiving
volume-cycled mechanical ventilation and his current
medications include IV heparin, ceftazidime, lorazepam,
and fentanyl. His platelet count was 260,000/µL on
admission and has dropped to 31,000/µL on hospital
day 6. The remainder of his laboratory profile, including
liver function tests, is unremarkable.
• In addition to testing for platelet factor 4 antibodies,
what is the most appropriate next step?

A)Change to therapeutic daily low-molecular-weight


heparin.

B)Discontinue heparin and begin therapeutic


argatroban.

C)Discontinue heparin and begin therapeutic lepirudin.

D)Continue to treat with IV unfractionated heparin.

E)Continue unfractionated heparin but start warfarin.


• Correct Answer: B
• Rationale
Option A is incorrect. There is a substantial risk (up to
85%) of cross-reactivity with low-molecular-weight
heparin and the risk of thrombosis is unacceptably high.

Option B is correct. Argatroban is a direct thrombin


inhibitor and is associated with a reduction in
thrombotic events in patients with heparin-induced
thrombocytopenia. It is cleared by the liver, and can be
administered safely in patients with renal insufficiency.
Option C is incorrect. While lepirudin is indicated for the
reduction in thrombotic events in patients with heparin-
induced thrombocytopenia, its levels rise precipitously
in patients with renal insufficiency and it is
contraindicated in patients such as this one with
elevated serum creatinine.
Option D is incorrect because the patient is suffering
from heparin-induced thrombocytopenia. Continuation
of heparin carries a high risk of arterial and venous
thrombotic events.
Option E is incorrect. Anticoagulation with warfarin is
associated with an increased risk of complications such
as venous limb gangrene and skin necrosis in patients
with heparin-induced thrombocytopenia. If warfarin has
already been started, vitamin K should be administered
as it can otherwise interfere with the activity of direct
thrombin inhibitors.
• Question 54:
• An elderly woman with diabetes undergoes outpatient
laparoscopic cholecystectomy for acute cholecystitis.
She is sent to the emergency department on the second
postoperative day after reporting severe abdominal
pain. Examination reveals altered mental status, high
fever, and a lactate level of 4.2 mg/dL. Glasgow Coma
Scale score is 10. Family members fear that the patient
has suffered a stroke. A photograph of the wound is
shown in the Figure.
• Which of the following is the most appropriate course of
action?

A)Immediate surgical consultation for operative


debridement
B)Monitoring intra-abdominal pressure
C)Broad-spectrum antibiotics to include anaerobic
coverage
D)Opening and packing the wound in the emergency
department
E)CT of brain, abdomen, and pelvis with oral and IV
contrast
• Correct Answer: A
• Rationale
This unfortunate woman with diabetes has a rapidly
progressing soft tissue infection following laparoscopy
and cholecystectomy. The umbilical port is frequently
involved; because this is the site where the gallbladder
was removed through a small opening in the abdominal
wall, and because the patient has significant
subcutaneous fat, contamination is likely. Changes
suggestive of progressive soft tissue infection are
apparent on the CT. Subcutaneous changes in a patient
such as this can occur without obvious cutaneous
manifestations. At the very least, immediate surgical
consultation should be obtained.
The patient has a rapidly spreading necrotizing soft
tissue infection and shows signs of multisystem organ
failure. This patient must be assumed to have deep soft
tissue infection extending at least to the fascia and
possibly below. Appropriate antibiotic choices include
piperacillin/tazobactam and vancomycin. A common
offending organism in this setting is Streptococcus with
toxin production. However, staphylococci have recently
been shown to cause a toxin syndrome similar to
streptococci. This patient received immediate surgical
debridement and antibiotics, as described above. The
peritoneal cavity was opened and the underlying
intestine was uninvolved.
• Question 55:
• A 65-year-old man is treated for diverticulitis with IV
amoxicillin/sulbactam for 5 days. On day 5, he develops a
fever and altered mental status. Chest radiograph reveals a
right lower lobe infiltrate. He is intubated and the sputum
culture specimen grows a resistant Pseudomonas. His
antibiotics are changed to piperacillin/tazobactam and
tobramycin. Two days after the antibiotic change, the
patie t’s fe e de eases, a d he is o e ale t. O da
after the change of antibiotics, the patient develops a
fever to 38.9°C (102°F), his WBC count increases from
12,000 to 19,000/µL, and he has several episodes of
watery diarrhea. A stool sample tests positive for fecal
leukocytes and the presence of yeast. Additional
medications include IV famotidine, prophylactic low-
molecular-weight heparin, and enteral feeding with an iso-
osmotic nutritional product.
• Which of the following is the most appropriate
intervention?

A)Administer oral metronidazole.

B)Administer IV vancomycin.

C)Administer IV fluconazole.

D)Discontinue enteral feedings.

E)Discontinue piperacillin/tazobactam.
• Correct Answer: A
• Rationale
The mostly likely diagnosis in this patient who has been
on multiple antibiotics is Clostridium difficile infection.
Infection can also manifest several days after
discontinuation of antibiotics. The treatment of choice is
the administration of oral metronidazole, 500 mg 3
times daily or 250 mg 4 times daily.

For non-severe C difficile infection, oral vancomycin and


oral metronidazole are equivalent in efficacy, but
metronidazole is preferred because of cost in treatment
of vancomycin-resistant enterococci. Metronidazole is
associated with some concerns of relapse. Although oral
vancomycin, 125 mg 4 times daily, has similar efficacy,
its use is reserved for metronidazole failures.
If the oral route is precluded, IV metronidazole is also
effective, because a moderate concentration of the drug
is attainable in the colon. However, IV vancomycin is not
effective.

The decision to treat is usually based on a positive C


difficile toxin assay, but treatment should be initiated in
high-risk patients prior to obtaining results. This patient
has several risk factors, including hospitalization, older
age, and several days of treatment with penicillin
derivative antibiotics. Treatment is indicated based on
the presence of the fever, the leukocytosis, the evidence
of colitis with fecal leukocytes, and the need to continue
the antibiotic for a resistant nosocomial pneumonia.
Ideally, the implicated antibiotic should be discontinued,
although this may not always be possible in the critically ill
patient. Discontinuation of the antibiotic alone may result
in resolution, but is not a reliable stand-alone sole
treatment of choice for critically ill patients.
Finding yeast in the stool is not an indication for treatment
and Candida would not be expected to cause colitis. There
are many other causes of diarrhea in the critically ill
patient, including osmotic diarrhea due to nutritional
products, the use of drugs containing lactulose or sorbitol,
and the use of laxatives or nonsteriodal antiinflammatory
drugs. There is no indication that any of these agents are
implicated in this patient.
Complications of Clostridium difficile infection include
toxic megacolon (rare) and a relapse that occurs in 20%-
25% of the cases.
• Question 56:
• A patient who has been in the ICU for 2 weeks with a
prolonged illness develops gram-negative bacteremia
from a urinary source with an indwelling bladder
catheter. Empiric IV ceftriaxone had been started when
the patient developed fever and leukocytosis. Blood
culture results become positive for Escherichia coli in 36
hours. The isolate is sensitive to ceftriaxone, cefotaxime,
and imipenem, but it is resistant to ceftazidime and
aztreonam. Seventy-two hours after starting
ceftriaxone, the patient continues to have fever,
tachycardia, and leukocytosis.
• Which of the following is the best treatment strategy?

A)Increase the dose of IV ceftriaxone to 2 g/day.

B)Add gentamicin on an extended daily dosing reigmen.

C)Discontinue ceftriaxone and begin cefotaxime.

D)Add cefotaxime to current regimen.

E)Discontinue ceftriaxone and begin IV meropenem.


• Correct Answer: E
• Rationale
Despite the in vitro susceptibility results, the organism is
not actually sensitive to ceftriaxone in vivo. Extended-
spectrum beta-lactamases (ESBLs) are plasmid-mediated
enzymes that inactivate all beta-lactam antibiotics,
except for cephamycins (cefoxitin) and carbapenems
(imipenem, meropenem). The organisms are also
characteristically resistant to aminoglycosides and
fluoroquinolones. The enzymes are found in a variety of
Enterobacteriaceae (most commonly Klebsiella
pneumoniae and Escherichia coli). ESBLs are challenging,
both because of the difficulty in detecting the presence
of ESBLs, and the limited treatment options available.
ESBLs most likely have emerged because of widespread
use of third-generation cephalosporins and aztreonam.
Because ESBLs are plasmid-mediated, the enzymes are
easily transferred to other bacteria species. Risk factors for
infection with ESBL-producing organisms include
prolonged ICU stay, high severity of illness, long-term and
repeated therapeutic antibiotic exposure, and
instrumentation and/or indwelling catheters of any kind.

Detection of ESBLs is often difficult. To quantify the extent


of the problem in an institution, determining the rate of in
vitro ceftazidime resistance is one way. The organism may
appear to be sensitive to other third-generation
cephalosporins in vitro. However, the organism will not be
clinically susceptible to third-generation cephalosporins,
and a treatment failure will result.
The tip-off for an ESBL-producing organism was the in
vitro resistance to ceftazidime. The ESBL E-test (AB
Biodisk, Sweden) is the most sensitive method for
detecting ESBLs.

Carbapenems (specifically meropenem) are the most


effective agents against ESBLs. An ESBL E-test should be
performed for this isolate, and the patient should be
started on meropenem pending the results.
• Question 57:
• A 45-year-old man with past medical history of type 2
diabetes mellitus presents to the emergency
department with a painful left leg. He is febrile to 38.8°C
(101.8°F) and tachycardic to 122/min; he has BP of
88/52 mm Hg and feels weak. He has no history of
operation or injury of his left leg and is not HIV-positive.
He has pain out of proportion to his examination and
has palpable dorsalis pedis and posterior tibial pulses.
WBCs are 20,000/µL and sodium level is 129 mEq/L;
necrotizing soft tissue infection is suspected.
• The next best management step is to:

A)Admit to the surgical ICU for plasma volume


expansion

B)Insert a central venous pressure line and obtain


ScvO2 to guide therapy

C)Obtain a bedside ultrasonography to evaluate for a


fluid collection

D)Proceed directly to the operating room without


diagnostic testing
• Correct Answer: D
• Rationale
This patient presents with a typical necrotizing soft tissue
infection (NSTI) history. In the US, diabetes mellitus is
strongly associated with NSTI. In up to half the patients
evaluated in the US, no antecedent event is identified.
Pain out of proportion to examination as well as exquisite
tenderness on palpation are also strongly associated with
the presence of an NSTI (necrotizing cellulitis, fasciitis,
myositis, or vasculitis singly or in combination). While
several laboratory profiles (leukocytosis and
hyponatremia) suggest a necrotizing versus nonnecrotizing
soft tissue infection, the diagnosis is a clinical one. While
CT with contrast may demonstrate findings that are
consistent with NSTI, no such data are available for MRI or
ultrasonography. Moreover, radiologic imaging as well as
bedside sampling of any kind add delays to operative
therapy.
All NSTIs are surgical emergencies and require prompt
surgical intervention consisting of radical excisional
debridement of all involved tissue coupled with plasma
volume expansion and broad-spectrum antimicrobial
coverage; fungal infection is rare but has been reported.
Operation should not be delayed for a period of
resuscitation in the emergency department or the
surgical ICU, as one will not be able to successfully
resuscitate a patient with NSTI in the absence of surgical
debridement. Moreover, delays to operative
debridement strongly correlate with inceased mortality.
• Question 58:
• A 21-year-old man presents to the emergency department
with a 4-day history of sore throat and 2 days of fever that has
worsened over 24 hours. He was seen in the emergency
department with normal vital signs 2 days prior and was told
he had pharyngitis that was not due to Streptococcus(normal
rapid Streptococcus test). He was sent home with a
prescription for azithromycin, which he took. One day before
admission, he developed left-sided neck pain and swelling with
increasing subjective fevers. On the day of admission, his neck
pain increased and he noted pleuritic chest pain and rigors.
Temperature was 38.9°C (102°F), BP was 120/80 mm Hg, HR
was 110/min, RR was 20/min, and SaO2 was 94% on room air.
Physical examination revealed left neck swelling and
tenderness. Laboratory testing showed WBCs 12,000/µL with
10% bands. Chest radiography showed patch peripheral
infiltrates. After blood cultures were obtained, antibiotics were
initiated and he was admitted to the floor.
A pulmonary embolism protocol CT showed bilateral
subsegmental pulmonary emboli associated with
peripheral nodular infiltrates. Neck CT showed some soft
tissue swelling of the oropharynx extending into the soft
tissues of the left neck, with thrombosis of the left internal
jugular vein. The following day, 4 out of 4 blood cultures
were positive for the same organism. Which of the
following organisms is the most likely pathogen?

A)Staphylococcus aureus
B)Clostridium species
C)Bacteroides fragilis
D)Fusobacterium species
E)Klebsiella pneumoniae
• Correct Answer: D
• Rationale
This presentation of a normal host with oropharyngeal
infection complicated by septic thrombophlebitis of the
jugular vein and septic emboli is classic for Lemierre
syndrome. This is caused by the anaerobic
bacteriaFusobacterium.
Although Staphylococcus and Clostridium may cause soft
tissue infection, they and the other pathogens listed do
not typically cause pharyngitis or the other manifestions
of this syndrome.
• Question 59: Which of the following statements would be
the best guide in prevention and treatment
ofCandida infections?

A)Candiduria regardless of symptom presence or absence


should be treated with amphotericin B bladder washings.
B)Candida parapsilosis fungemia is rarely associated with
an indwelling central venous catheter.
C)Positive blood cultures in a neutropenic patient are not
an absolute indication for removal of a tunneled catheter.
D)Central line infections in patients receiving parenteral
nutrition are decreased with fluconazole prophylaxis.
E)Empiric therapy for Candida with an echinocandin is
superior to an azole in an HIV-infected patient.
• Correct Answer: C
• Rationale
Candida infections are not uncommon in the ICU in the
urinary tract, central venous catheters, indwelling
catheters, and neutropenic patients. Candiduria is not
effectively treated with amphotericin bladder
washings. Candida parapsilosis fungemia is frequently
associated with indwelling central venous catheters.
However, a positive blood culture in a neutropenic
patient is not an absolute indication for tunneled
catheter removal as many are dialysis lines that may be
salvaged with culture-directed antimicrobial therapy.
There is no evidence to suggest that empiric therapy
for Candida is superior to an azole in HIV-positive
patients.
• Question 60:
• A 41-year-old man is evaluated for sore throat, pleuritic
chest pain, and redness and swelling of the anterior
neck. He was started on a fluoroquinolone 2 days ago by
his primary care physician. On emergency department
arrival, his BP is 162/101 mm Hg, HR is 110/min, RR is
32/min, and temperature is 38.5°C (101.2°F). Pharynx
examination shows bilateral redness with minimal
exudate. Chest examination reveals bilateral basilar
crackles. Anterior neck examination demonstrates
redness, tenderness and swelling, as shown in the
Figure below.
ECG is shown in the Figure below.
• Which of the following is the most appropriate next step
in management?

A)Discontinuing the fluoroquinolone

B)Emergent surgical intervention

C)Adding oral glucocorticoids

D)Adding anaerobic coverage

E)Oral endotracheal intubation


• Correct Answer: B
• Rationale
Clues to the diagnosis in this patient are the previous
pharyngeal infection, the redness and swelling of the
anterior neck extending to the sternal notch, the
pleuritic chest pain, and the ECG finding of pericarditis.
The ECG indicates a diffuse ST-segment elevation (or PR-
segment depression).

This patient has mediastinitis due to extension of an


anterior neck infection due to preexisting pharyngitis.
The most characteristic causative organism in this case
was beta-hemolytic Streptococcus.
Although anaerobic coverage would be indicated, the
key is surgical source control. Depending on the
fluoroquinolone chosen, it may or may not offer
adequate empiric coverage for this patient. While
tempting to establish airway control in the emergency
department, it is best left for the operating room as with
pharyngeal and retropharyngeal infection. Airway
edema is common, creating a difficult airway. As such, a
surgical airway may be required and should be planned
fo he this patie t’s ai a is ulti atel o t olled. If
the patient is able to maintain his own airway, no
intervention is superior in the short term.
• Question 61:
• A 30-year-old man who lives in St. Louis, Missouri, is
admitted to the hospital because of fevers, cough
productive of purulent sputum, and headaches for 4
days. He has been HIV positive for 2 years but has not
had an AIDS-defining illness. His recent CD4 lymphocyte
count was 430/µL. His admission temperature is 38.9°C
(102°F). His pulse rate is 120/min, RR is 25/min, and BP
is 120/80 mm Hg. On examination, he has crackles and
egophony over the right lower region of his chest.
Cardiac examination is normal. Chest radiograph reveals
an opacity in the right lower lobe without evidence of
fluid collection. His leukocyte count is 12,500/µL with
87% neutrophils.
• Which of the following is the most likely cause of this
patie t’s s pto s?

A)Cryptococcus neoformans

B)Streptococcus pneumoniae

C)Histoplasma capsulatum

D)Pneumocystis jiroveci

E)Mycobacterium tuberculosis
• Correct Answer: B
• Rationale
Optio A is i o e t e ause the deg ee of the patie t’s
immunosuppression should not predispose him to this
infection.

Option B is correct. The incidence of pneumonococcal


disease is increased relatively early in HIV disease. HIV-
infected patients are characterized early in the disease
by impairment in immunoglobulin A mucosal activity,
complement-mediated bactericidal activity,
mononuclear cell-killing activity, and humoral
dysfunction.
Option C is incorrect. While the patient lives in an
endemic area, the time course of his illness is rapid and
the clinical picture is more compatible with
pneumonococcal disease.

Optio D is i o e t e ause the patie t’s


immunosuppression should not predispose him to this
infection.

Option E is incorrect. While tuberculosis is possible,


typical bacterial community-acquired pneumonia is
more common in this patient population.
• Question 62:
• A 52-year-old woman with diabetes develops a
perirectal abscess, which is drained. After drainage, she
is placed on clindamycin. She is admitted to the ICU 36
hours later with severe perineal pain. She is confused,
febrile, and hypotensive. Examination of the perineal
area reveals edema and redness extending to the inner
thighs. Her glucose concentration is 400 mg/dL.
• Which of the following is most appropriate next step in
definitive therapy?

A)Adding gram-negative antibiotic coverage

B)Urgent CT of the perineum and thighs

C)Immediate surgical exploration

D)Placement of a subclavian vein catheter

E)IV insulin therapy for glycemic control


• Correct Answer: C
• Rationale
This patient is presumed to have a necrotizing soft tissue
infection that is creating septic shock. This condition will
require radical surgical debridement as well as plasma
volume expansion and broadening of antimicrobial
coverage to cover mixed flora; adding only an anti-gram-
negative agent to clindamycin will be inadequate. In
diabetic patients, the mortality from this infection exceeds
50%. This infection is most commonly (approximately
80%) a mixed-organism infection with fecal organisms. The
extent of the infection is unknown without surgery and it
should be treated as a surgical emergency. Obtaining CT
will only delay therapy and delays in debridement are
associated with increased mortality; admission to a
nonsurgical service is highly associated with delay to
operation.
Skin, subcutaneous tissue, fascia, and muscle
compartments all may be involved. All dead and
infected tissue must be removed. If extensive
involvement of the perineum is present, a colostomy
may be required.

Control of the hyperglycemia is not primary and will be


rather difficult to achieve without control of the
infection.
• Question 63:
• A 48-year-old man presents to the hospital with a 2-day
history of fever, throat pain, and dysphagia. Past medical
history is significant for alcohol abuse and hypertension.
Temperature is 38.2°C (100.7°F), HR is 110/min, BP is
140/90 mm Hg, and RR is 20/min. Room air pulse
oximetry is 100%. On examination, there is a brawny,
hard induration of the floor of the oral cavity with
elevation of the tongue. Oral hygiene and dentition are
poor. The oropharyngeal mucosa is erythematous; there
is no stridor. In addition to close airway monitoring.
• which of the following regimens is most appropriate for
this case?

A)Penicillin G and metronidazole

B)Clindamycin monotherapy

C)Levofloxacin monotherapy

D)Ceftriaxone and vancomycin

E)Amphotericin B monotherapy
• Correct Answer: A
• Rationale
The patient in this question presents with clinical
features suggestive of Ludwig angina, a rapidly
progressive cellulitis of the floor of the mouth involving
the sublingual, submandibular, and submental spaces.
Clinically, it is characterized by a brawny discoloration of
the floor of the mouth, with an elevation of the tongue;
it is very frequently associated with airway compromise.
The infection is usually caused by decayed teeth and is
often seen in patients with a history of alcoholism,
diabetes mellitus, and immunodeficiency disorders.
Management is directed toward securing an airway,
administering systemic antibiotic therapy, and
instituting source control, with early surgical
decompression when needed. The most common
causative pathogens are a mixed flora of aerobic and
anaerobic bacteria, including streptococcal
species, Staphylococcus aureus, Borrelia
vincentii, Fusobacterium, Bacteroides species,
and Eikenella corrodens.

Of the possible choices, the most appropriate antibiotic


regimen would include pencillin G plus metronidazole to
cover the above pathogens.
There are no clinical trials available to guide empiric
therapy for Ludwig angina, but consensus available in
the literature suggests use of penicillin or a beta-
lactamase–resistant antibiotic with anaerobic coverage.
In light of the widespread increase in prevalence of
resistant bacterial strains, such as methicillin-resistant S
aureus (MRSA), if the risk is felt to be sufficiently high,
addition of vancomycin or other newer agents such as
linezolid, daptomycin, or tigecycline may also be
appropriate. A growing incidence of E corrodens, which
does not respond to clindamycin, makes clindamycin
monotherapy inadequate.
• Question 64:
• Which of the following is the most common cause of
community-acquired pneumonia in an HIV-positive
patient with a CD4 count of 523/µL?

A)Pseudomonas aeruginosa

B)Blastomycosis

C)Streptococcus pneumoniae

D)Pneumocystis jiroveci

E)Haemophilus influenzae
• Correct Answer: C
• Rationale
HIV-infected patients often present atypical clinical
features of community-acquired pneumonia (CAP). As in
immunocompetent adults, the most common agent
causing CAP in HIV-infected patients is S
pneumoniae followed by gram-negative bacteria
(including Haemophilus influenzae, Pseudomonas
aeruginosa, and Legionella pneumophila) and is frequently
associated with bacteremic disease even in low-risk
patients according to pneumonia severity index. CD4 cell
count should be carefully considered in HIV patients with
CAP. In consideration of their high mortality risk, patients
with a CD4 cell count of less than 200/mL should be
hospitalized, whereas those with a CD4 cell count of at
least 200/mL could be managed according to the
pneumonia severity index score.
Empiric antibiotic therapy should include a combination
of a beta-lactam and a macrolide or a fluoroquinolone
with a high alveolar partition coefficient alone. Finally,
prevention strategies should include lifestyle
modification, highly active antiretroviral therapy access
and adherence programs, and the implementation of
pneumococcal vaccination. AIDS-associated
opportunistic infections such as Pneumocystis
jiroveci and blastomycosis tend to occur later in the
course of HIV infection when there is substantial
depletion in CD4 cells.
• Question 65:
• Insertion site skin preparation with which of the
following results in the lowest incidence of catheter-
related bloodstream infections

A)Povidone-iodine

B)Antibiotic ointment

C)Isopropyl alcohol

D)Chlorhexidine gluconate

E)Polyhexanide gel
• Correct Answer: D
• Rationale
Povidone-iodine has been the most widely used
antiseptic for cleansing the skin prior to insertion of
arterial or central venous catheters. However, a
prospective randomized study demonstrated that the
preparation of central venous and arterial sites with a
2% chlorhexidine gluconate solution decreased
catheter-related bloodstream infections when
compared with site preparation with 10% povidone-
iodine or 70% alcohol.

A 2% tincture of chlorhexidine for skin antiseptic


preparation was approved by the US Food and Drug
Administration in 2000. Other preparations of
chlorhexidine might not be as effective.
• In another study, the use of chlorhexidine gluconate
0.5% was no more effective in preventing catheter-
related bloodstream infections or central venous
catheter colonization than the use of 10% povidone-
iodine.

Current recommendations from the Centers for Disease


Control and Prevention for decreasing the incidence of
catheter-related infections include the use of 2%
chlorhexidine gluconate. Polyhexanide decanoate is
used as a gel for the management of acute or
chronically infected bone or soft tissues; it is not a
catheter insertion site preparant.
• Question 66:
• A 22-year-old o a at eeks’ gestatio is
hospitalized with premature contractions. Her BP is
169/110 mm Hg, HR is 90/min, RR is 18/min, and
temperature is 37.1°C (98.7°F). Urinalysis shows 3+
protein, and she has 1+ edema of her lower extremities.
Fetal heart tones are normal. She is started on IV fluids
at 200 mL/h, IV magnesium sulfate, and subcutaneous
terbutaline. Three hours later, she reports shortness of
breath and cough. While receiving room air, pulse
oximetry reveals 91% saturation, RR is 26/min, and rales
are noted in the lower lung fields. No jugular venous
distension or cardiac gallops are appreciated. Chest
radiography demonstrates pulmonary edema.
• Which of the following is the most likely etiology of the
pulmonary edema?

A)Acute cardiomyopathy

B)Right heart failure

C)Tocolytic agent

D)Amniotic fluid embolism

E)Aspiration pneumonia
• Correct Answer: C
• Rationale
Pulmonary edema in pregnancy is associated with an
increased risk of maternal and fetal morbidity and
mortality. In this patient, the most likely cause of the
pulmonary edema is the use of a tocolytic agent,
terbutaline. A recent study identified the combination of
magnesium sulfate and subcutaneous terbutaline as the
most common cause of tocolytic-associated pulmonary
edema. The exact pathophysiology is controversial, and
potential mechanisms include fluid overload from salt and
water retention, increased cardiac output and hydrostatic
pressure, and increased capillary permeability. Possible
risk factors for tocolytic-associated pulmonary edema
include multiple gestations, preeclampsia, and sepsis.
Therapy includes discontinuation of the agents,
supplemental oxygen, and possible diuretic
administration.
Although peripartum cardiomyopathy may present
in the last month of pregnancy, the absence of
jugular venous distension or cardiac gallop, as well
as the acuteness of onset, make this diagnosis less
likely. Fluid overload has also been identified as a
cause of pulmonary edema in pregnancy, but the
amount of fluid administered in this patient would
have only been 600 mL prior to the development of
symptoms. Most fluid overload associated with
pulmonary edema has occurred in the postpartum
period. Other causes of pulmonary edema in a
pregnant patient include preeclampsia, severe
infection, and cardiac disease, such as valvular
disorders and hypertrophic subaortic stenosis.
• Question 67:
• An adult patient with no significant past medical history
or prior hospitalizations comes to the emergency
department because of a soft tissue infection of the leg
with purulent drainage. She is admitted to the hospital
for antibiotic therapy because of the extent of her
cellulitis. She travelled to England, Ireland, and Wales 2
months ago. She was treated for otitis media and
externa 5 weeks ago after a scuba diving vacation in
Belize; she was successfully treated with 10 days of oral
ciprofloxacin. She has gluten enteropathy and takes
regular probiotic supplements including fructo-
oligosaccharides. She has a Maine Coon cat as a pet.
• Which of the following elements would impact empiric
antibiotic selection for this patient?

A)Travel to England, Ireland, and Wales

B)Scuba diving in Belize

C)Maine Coon cat as a pet

D)Fructo-oligosaccharide consumption

E)Antibiotic exposure 5 weeks ago


• Correct Answer: E
• Rationale
This patie t’s pu ule t soft tissue i fe tio should
be assumed to be secondary to methicillin-
resistantStaphylococcus aureus (MRSA) until proven
otherwise. Recent therapeutic antibiotic exposure (<
3 months prior), in particular with a
fluoroquinolone, increases the risk for MRSA. In a
prospective multicenter study of purulent soft tissue
infections in the emergency department setting,
Moran and colleagues2 identified MRSA as the most
commonly isolated pathogen. Assuming that this
could be MRSA, contact precautions should be used
in the hospital. Monotherapy with cephazolin would
not cover MRSA, and therefore alternative
antibiotics with activity against MRSA should be
utilized until culture results are available.
A history of close contact with another person
having a purulent soft tissue infection would support
the likelihood of MRSA being the pathogen and
would certainly necessitate utilization of antibiotics
with activity against MRSA.
Travel outside of the US does not increase the
infection risk for MRSA per se. Gluten enteropathy
has no relation to MRSA infection risk nor should it
influence the choice of antibiotics for a skin- and
skin structure–based infection. Fructo-
oligosaccharides promote the growth of healthy
gastrointestinal flora and would support gut mucosal
barrier function. While cat ownership increases the
risk for catscratch fever, most of these are on the
upper extremities and are associated with
epitrochelar adenopathy, not abscess formation.
• Question 68:
• Which of the following is an acceptable strategy for
decreasing antimicrobial resistance in the ICU?

A)Exclusive use of broad-spectrum antibiotics

B)Changing central venous catheters and sites every 6


days

C)Culturing blood, urine, and respiratory secretions daily

D)De-escalation (narrowing) of antibiotic regimen after


72 hours
• Correct Answer: D
• Rationale
The indiscriminate use of broad-spectrum antibiotics
in the ICU setting can result in increased infections
with antimicrobial-resistant organisms. Therefore,
decreasing broad-spectrum antibiotic use and
narrowing the antibiotic regimen to specifically
target the identified pathogen as soon as possible
after initial empiric therapy are critical actions.

However, critical care clinicians caring for patients


with a high severity of illness must often utilize
broad-spectrum antibiotics early in the course of
therapy since failing to appropriately cover a
pathogen could have catastrophic consequences.
Evidence-based strategies for decreasing antimicrobial
esista e i the ICU setti g i lude a ti ioti li g
(rotating empiric antibiotic choices at regularly
scheduled intervals in order to minimize resistance to
any one particular drug) and automated evidence-based
protocols for selecting antibiotics.

Over time, heterogeneity in antibiotic utilization helps


minimize antimicrobial resistance in the ICU, while
homogeneity in antibiotic use clearly increases
antimicrobial resistance in ICU pathogens. Therefore, a
highly restrictive formulary that limits the number of
broad-spectrum antibiotics to a select few would be
unfavorable.
• Question 69:
• A 23-year-old man with acute myeloid leukemia has
been hospitalized for weakness and malaise after recent
allogeneic bone marrow transplant. He develops fever
and hypotension. His chest radiograph is clear, but the
insertion site of his central venous catheter is
erythematous. He requires norepinephrine for blood
pressure support and has become oliguric. Broad-
spectrum antibiotics are started. At the same time,
invasive Candida is suspected as a possible etiology of
his sepsis and the catheter is removed for suspicion of
central line–associated bloodstream infection. A
surveillance blood culture drawn several days ago is now
growing Candida glabrata.
• Which of the following is the most appropriate
treatment for this patient?

A)Amphotericin B

B)Voriconazole

C)Micafungin

D)Fluconazole

E)Itraconazole
• Correct Answer: C
• Rationale
The most common cause of invasive fungal disease in
the ICU is infection with Candida species.
Although Candida albicans is the most commonly
identified spcies, the incidence of infection with non-
albicans Candida species (such as C glabrata, C
parapsilosis, C tropicalis, C krusei) is increasing. Risk
factors for developing non-albicans Candida infection
include systemic antifungal therapy, central venous
catheter placement, and prior gastrointestinal surgery,
especially upper gastrointestinal surgery in those with
gastric achlorhydria or on acid-suppressive therapy.
The answer for this question would be option C,
micafungin. The Infectious Diseases Society of
America 2009 guidelines recommend use of an
echinocandin in unstable patients, and the 2011
American Thoracic Society statement on treatment
of fungal infections recommends echinocandins,
amphotericin B, voriconazole, or high-dose
fluconazole in clinically unstable patients with
unknown species of candidemia.
This patient has documented candidemia with a
non-albicans species, C glabrata. Treatment of
choice for invasive candidiasis in known candidemia
in a hemodynamically unstable patient involves the
use of either echinocandins such as caspofungin,
micafungin, or anidulafungin, or a liposomal form of
amphotericin B.
Liposomal amphotericin B can be considered but would
not be the first choice for this patient given its association
ith e al a d li e side effe ts a d this patie t’s sig s of
renal dysfunction with oliguria in the setting of his
hypotension and likely sepsis. Fluconazole therapy (option
D) should be avoided if infection with C glabrata and C
krusei is common in the institution, since these species
can account for about 15% of Candida. Fluconazole should
also be avoided if infection is associated with a history of
exposure to azole therapy within the past 30 days or if the
patient has had persistent Candida infection for more than
5 days. Also, if fluconazole is to be used, it would be high-
dose fluconazole at 800 mg/day. Treatment in general is
continued for 2 weeks after the last positive blood
cultures, provided that clinical signs of infection have
resolved.
• Question 70:
• In critically ill surgical patients, which of the following is
associated with the lowest risk of developing a systemic
fungal infection?

A)Hyperalimentation

B)Upper gastrointestinal surgery

C)Abdominal compartment syndrome

D)Broad-spectrum antibiotic therapy

E)Acute hemodialysis
• Correct Answer: C
• Rationale
Risks for developing an invasive, systemic fungal
infection in the ICU are numerous, and include
presence of a central venous catheter, lack of enteral
nutrition, utilization of total parenteral nutrition,
utilization of broad-spectrum antibiotics (with an
especially heightened risk with utilization of
anaerobic coverage), acute hemodialysis, upper
gastrointestinal surgery, and any degree of
immunosuppression. In addition, colonization
with Candida species is a major risk factor for
developing a fungal infection. Both the duration and
the number of sites colonized with Candidaspecies
have been identified as important risk factors for
developing invasive, systemic fungal infection.
The abdominal compartment syndrome is associated
with injury (including burns), emergency general
surgery, pneumonia, and aggressive resuscitation. There
is no data that having the abdominal compartment
syndrome increases the risk of invasive fungal infection
in the critically ill, and fungal prophylaxis is not indicated
for those managed with an open peritoneal
compartment.
• Question 71: Which of the following courses of action is most
appropriate in the critical care of an HIV-infected patient
receiving highly active antiretroviral therapy (HAART)?

A)Hold therapy for immune reconstitution inflammatory


syndrome–induced hepatitis.
B)Reduce therapy to avoid inducing hyperchloremic metabolic
acidosis.
C)Reduce HAART until bronchoalveolar lavage excludes
possibility of a pneumonia.
D)Without suspected HAART toxicity, chronic therapy should
be continued.
E)HAART should be increased with concomitant bacterial
infection.
• Correct Answer: D
• Rationale
Although the initiation of highly active antiretroviral
therapy (HAART) de novo in the ICU setting may not
always be indicated when an HIV-infected patient is
admitted with acute critical illness, it is generally
accepted that (in the absence of suspected HAART
toxicity) chronic HAART should be continued in the ICU.
Long-term survival is most closely linked with success
with antiretroviral therapy.
However, the critical care clinician needs to be aware of
several important complications that can be associated
with HAART. HAART can cause a severe lactic acidosis.
Also, a life-threatening hypersensitivity reaction has been
associated with inclusion of abacavir in a HAART regimen.
Another possible complication of HAART is the immune
reconstitution inflammatory syndrome (IRIS), of which the
most common clinical manifestation is acute lung injury.
The lung injury associated with IRIS is believed to be due
to systemic inflammation associated with an enhancement
of the lymphoproliferative response. As lung injury with
IRIS may be difficult to differentiate from an infectious
process, the diagnosis of IRIS-mediated lung injury often
requires bronchoscopy with bronchoalveolar lavage to
exclude an acute infectious process. Patients with IRIS-
mediated lung injury improve after discontinuation of
HAART.
• Question 72:
• A 28-year-old man was recently diagnosed with AIDS
and Pneumocystis pneumonia. He was started on
trimethoprim/sulfamethoxazole and antiretroviral
therapy as an outpatient. He presents to the hospital 2
weeks later with fever, myalgias, and rash. The rash
comprises erythematous confluent macules covering
more than 50% of the body surface with blisters and
with sloughing in some areas. Oral and conjunctival
lesions are noted.
• Which of the following treatments should be
administered?

A)Broad-spectrum antibiotics

B)Systemic corticosteroids

C)IV infliximab

D)IV immunoglobulin

E)Topical antimicrobials
• Correct Answer: E
• Rationale
The clinical situation and the description of the
mucocutaneous lesions make a diagnosis of Stevens-
Johnson syndrome/toxic epidermal necrolysis (TEN)
most likely in this patient. In addition, HIV-infected
patients are predisposed to this condition, probably
because of an increased exposure to drugs. The majority
of cases (>80%) are attributed to drugs, with antibiotics
(sulfonamides, quinolones, etc) and anticonvulsants
(phenytoin, carbamazepine, etc) being common.
Infection with viruses or Mycoplasma is much less
common.
Stevens-Johnson and TEN are thought to be due to an
immune response to antigenic complexes formed
between drug metabolites and host tissue. Both are part
of the same disease spectrum with TEN defined as
having greater than 30% epidermal loss. Immediate
withdrawal of the suspected drug and supportive care
are the primary interventions. Patients with significant
skin involvement should be cared for in a burn or wound
care unit or ICU. Care is needed to maintain fluid and
electrolyte balance, normothermia, nutrition, and pain
relief, as well as monitoring for evidence of infection.
Ophthalmologic evaluation should also be obtained.
Routine use of systemic antibiotics is not recommended
in the absence of suspected infection. Infection
withStaphylococcus aureus and Pseudomonas
aeruginosa is the most common cause of death.

The general consensus is that corticosteroids should not


be administered, although adequate clinical trials have
not been performed. Likewise, IV immunoglobulin has
not been evaluated in clinical trials for evidence of
benefit. Case series of immunoglobulin use have
reported mixed results. Infliximab is a monoclonal
antibiody indicated for the therapy of Crohn disease and
is not indicated for this patient.
• Question 73: Rifaximin is potentially useful in which of the
following scenarios?

A)Chronic suppression of methicillin-


resistant Staphylococcus aureus
B)Adjuvant treatment of vancomycin-
resistant Enterococcus faecium
C)Adjunct in preventing acute hepatic encephalopathy
recurrence
D)Prevention of ventilator-associated pneumonia as a
probiotic
E)As an alternative to doxycycline in those with
gastrointestinal intolerance
• Correct Answer: C
• Rationale
In 2005, rifaximin received orphan drug status as a
treatment for hepatic encephalopathy from the FDA.
Again, in March 2010, the FDA approved rifaximin to
reduce recurrence of hepatic encephalopathy. Rifaximin
is a semisynthetic nonsystemic antibiotic
(nonabsorbable) derivative of rifampin. The dose is 400
mg orally 3 times a day. It is found to be as effective as
lactulose and neomycin in improving symptoms of
hepatic encephalopathy. Also, it is very well tolerated.
However, there is no standard of care recommendation
with regard to the use of rifaximin.
• Question 74:
• A 42-year-old man with a history of severe dilated
cardiomyopathy (ejection fraction 10%-15%)
presents to the emergency department after a
syncopal event. On arrival he is conscious and alert.
While waiting in the triage area, he becomes
unresponsive. He is rushed into a treatment area,
where he is found to be nonbreathing and pulseless.
The cardiac monitor shows ventricular fibrillation.
He is promptly intubated; Advanced Cardiac Life
Support care is rendered and return of spontaneous
circulation is achieved. At that time, HR is 110/min,
RR is 20/min (mechanically ventilated), and oxygen
saturation is 100% on FIO2 of 1.0. The total
estimated down time is 5-7 minutes. After checking
a portable chest radiograph to confirm the adequacy
of the endotracheal tube placement,
• which of the following is the most appropriate next
step?

A)Transesophageal echocardiography

B)Perfusion-weighted brain MRI

C)Brain CT with/without contrast

D)Administration of magnesium sulfate

E)Induction of core hypothermia


• Correct Answer: E
• Rationale
This patient with severe cardiomyopathy has suffered a
ventricular fibrillation cardiac arrest. In addition to
antiarrhythmic therapies and insertion of an
implantable defibrillator, neuroprotection after the
return of spontaneous circulation is essential, to help
ensure the best possible neurological outcome for the
patient after his cardiac arrest. Therapeutic hypothermia
has been shown in 2 large randomized controlled trials
to improve neurologic outcome. Currently, therapeutic
hypothermia is recommended by the International
Liaison Council on Resuscitation after cardiac arrest.
Achieving therapeutic hypothermia to a target of 33°C
(91.4°F) can be challenging. Although numerous novel
devices have been developed (and are in development)
for the optimal cooling of patients, a strategy has also
been described utilizing therapies and devices that are
commonly available in routine practice. One of these
strategies is to apply cooling blankets and cold IV fluids
to achieve a target temperature of 33°C (91.4°F). This is
after mechanical ventilation, deep sedation, and
neuromuscular blockade have been achieved. There are
also external devices that can be used to facilitate the
hypothermia process. Most sources recommend
therapeutic hypothermia for 12-24 hours after arrest,
with a slow return to a normal temperature of
approximately 0.25°C per hour.
Acute brain or cardiac imaging will not change the acute
course but may be important for prognostication. While
magnesium sulfate is a membrane-stabilizing agent, and
the treatment of choice for torsade de pointes, it is not
the treatment of choice for ventricular fibrillation and is
less important than core hypothermia—an intervention
that provides enhanced neurologic outcome after
cardiac arrest.
• Question 75:
• In a normal intracranial pressure waveform, which of
the following best describes the relative sizes of
upstrokes?

A)P1 has the highest upstroke.

B)P2 has the highest upstroke.

C)P3 has the highest upstroke.

D)All 3 components have equal upstrokes.

E)All 3 exceed that of the crescendo wave.


• Correct Answer: A
• Rationale
A normal intracranial pressure waveform has a flow of 3
upstrokes. P1, which is also called the percussion wave,
represents arterial pulsation and has the highest upstroke.
P2, also known as the tidal wave, represents intracranial
compliance and has an upstroke of medium size. P3, the
dicrotic wave, represents aortic valve closure and shows
the lowest upstroke.

Upstroke size is an important clinical point. On visual


inspection of the waveform on monitor, if P2 has a higher
upstroke than P1, it indicates intracranial hypertension. A
crescendo wave is not a normal waveform and occurs with
severe intracranial hypertension.
• Question 76:
• A 13-year-old boy who was in a motor vehicle collison and
sustained a closed head injury has increased intracranial
pressure. Which of the following statements best
describes the cardiac response to traumatic brain injury?

A)Right ventricular function is commonly impaired.

B)Enhanced parasympathetic response is observed.

C)Increased systemic vascular resistance is common.

D)Decreased systemic vascular resistance is common.

E)Right ventricular function is commonly hyperdynamic.


• Correct Answer: C
• Rationale
Biventricular dysfunction is common in the setting of
traumatic brain injury. Traumatic brain injury results
in a state of high stress with exorbitant release of
catecholamines and resulting enhanced sympathetic
response. This response yields a high systemic
vascular resistance, with or without traumatic injury
to the myocardium and can result in significant
myocardial dysfunction and notable stress or strain
changes on ECG, including ST-segment changes. One
needs a high index of suspicion for myocardial injury
in the setting of thoracic trauma and can serially
follow troponin and creatine kinase-MB levels.
Inovasodilators may be useful in myocardial
insufficiency after trauma.
• Question 77:
• A 65-year-old man wakes up with left-sided weakness and
slurred speech. In the emergency department he is alert
and responds appropriately. His speech is mildly slurred
and manual muscle testing reveals 3/5 power in his left-
sided extremities. BP is 165/100 mm Hg, HR is 92/min,
and pulse oximetry reads 92% on room air. Head CT shows
no intracerebral leedi g. The patie t’s edi al histo is
significant for hypertension and hyperlipidemia. He has
not been taking his medications for the past 4 months.
Doppler duplex studies performed in the emergency
department are not revealing. CT angiography shows 85%
stenosis of the right middle cerebral artery. ECG
demonstrates normal sinus rhythm. Echocardiogram
shows normal ejection fraction, normal left atrium, and a
negative bubble study. In addition to long-term aggressive
treatment of hypertension and hyperlipidemia.
• the most appropriate next management option is:

A)Intracerebral angioplasty and stent placement


across stenosis
B)Early anticoagulation with full-dose heparin
followed by aspirin
C)Recombinant tPA followed by full-dose aspirin and
fish oil
D)Full-dose aspirin therapy plus speech and physical
therapy
E)Full-dose aspirin therapy plus oral warfarin
therapy
• Correct Answer: D
• Rationale
This patient has suffered an acute ischemic stroke in the
right middle cerebral artery territory. The time of onset
of the stroke is not known since the symptoms were
noticed after the patient woke up. Recombinant tPA
(alteplase) is only administered when the patient has
suffered an ischemic stroke and presents within 3 hours
of onset (4.5 hours in some cases). His risk factors for
stroke are hypertension and hyperlipidemia. These need
to be treated and well controlled. He needs to be on
aspirin or asprin/extended-release dipyridamole.
Swallow evaluation followed by physical and speech
therapy are needed for rehabilitation.
This patient has significant stenosis of his intracerebral
artery, which is a risk factor for recurrent stroke in the
same region. Angioplasty and stent placement across the
stenotic segment have been tried for stroke prevention. A
recent randomized controlled trial (Stenting and
Aggressive Medical Management for Preventing Recurrent
Stroke in Intracranial Stenosis [SAMMPRIS]) evaluated
stent placement and medical treatment versus aggressive
medical treatment alone. The patients had suffered a
transient ischemic stroke or acute stroke and had
significant stenosis of the intracerebral artery in the region
of the stroke. The trial had to be stopped early after
randomization of 451 patients as the adverse events (30-
day new stroke or death) were higher in the stenting arm
(14.7% versus 5.8%, P= 0.002). The fatal strokes were also
more in the stenting arm (2.2% versus 0.4%).
IV heparin, when used after acute ischemic stroke, has
not been shown to affect outcome. Even in patients
with a cardioembolic source of stroke and treatment
with early full anticoagulation (within 48 hours), the
small benefit of further stroke prevention is offset by
increased bleeding. Similarly, there is no indication to
pair aspirin and warfarin in this setting.
• Question 78:
• A 54-year-old man presents to the emergency
department about 4 hours after onset of a presumed
ischemic cerebrovascular a ide t. The hospital’s st oke
protocol specifies a 3-hour window for thrombolytic
therapy in acute cerebrovascular accident.
Which of the following is the most appropriate next step in
management?

A)Have a multidisciplinary consultation regarding proceeding


despite exceeding 3-hour limit.
B)Do not give thrombolytic therapy because the standard 3-
hour window has passed.
C)Perform an immediate brain MRI with/without contrast and
before any further management.
D)Start antihypertensive medication aggressively to decrease
systolic BP below 110 mm Hg.
E)Arrange palliative and pastoral care consultations to aid the
family with grief counseling.
• Correct Answer: A
• Rationale
New American Heart Association/American Stroke
Association guidelines for the administration of rtPA
following acute stroke have expanded the window of
treatment in acute ischemic cerebrovascular accident
from 3 to 4.5 hours. Therefore, option A is correct.

Option C is incorrect as there is no need to obtain MRI


with or without contrast as the patient has already
been diagnosed with a stroke. Moreover, the decision
for thrombolysis does not hinge on the results of MRI.
Option D is incorrect as an aggressive reduction in
blood pressure is not recommended in acute
cerebrovascular accident. A gradual drop with a goal of
15% is recommended in the first 24 hours of
presentation.

Palliative care or pastoral care consultation is not


appropriate at this time as the patient has not had
potentially salvaging therapy (thrombolysis) and
prognosis is unclear. Pastoral care alone may be helpful
ea l i the ou se as the patie t’s p og osis e o es
more clear. Palliative care may be rather helpful for
those with end-of-life issues and should be considered
early in those settings.
• Question 79:
• A 28-year-old man is involved in a motor vehicle crash at
high speed. He is obtunded upon emergency medical
services arrival and is intubated in the field. He arrives
at the emergency department with a Glasgow Coma
Scale (GCS) score of 3I. He has mild tachycardia but a
normal blood pressure. SaO2 is 100% on 100% oxygen.
CT of the brain is presented in the Figure below.
Two days later, he still has a GCS of 3I, is on no sedatives
or analgesics and has had none since arrival. He has no
cough, no gag, no corneal reflexes, and his pupils are 6
mm and fixed bilaterally equal. His temperature is 36.8°C
(98.2°F) rectally. His neck is not cleared, so oculocephalic
reflexes are not assessed but he has no eye motion on
oculovestibular reflex testing. He has no response to
central or peripheral noxious stimulation. Arterial blood
gas results show pH of 7.46, PaCO2 of 37 mm Hg, and
PaO2 of 112 mm Hg on assist-control ventilation with rate
of 10/min, tidal volume of 750 mL, FIO2 of 40%, and
positive end-expiratory pressure of 5 cm H2O with a flow
rate of 40 L/min using a decelerating waveform. Urine
output for the past 6 hours has been approximately 75
mL/h. Hemoglobin level is 11.2 g/dL and blood urea
nitrogen and creatinine levels are 19 mg/dL and 1.2
mg/dL, respectively. He is on no pressor agents.
• The most appropriate next step in management is to:

A)Place him on 100% oxygen to preoxygenate

B)Obtain a nuclear medicine flow study

C)Assess the patient for euvolemia

D)Repeat the noncontrast brain CT

E)Place a pelvic external fixator device


• Correct Answer: C
• Rationale
This patient has satisfied almost all the criteria necessary
to perform an apnea test as the next step in establishing
brain death. These prerequisites include a structural
abnormality that explains the neurologic examination, no
confounders including hypothermia (temperature >36°C
[96.7°F]), no mood- or mind-altering agents within 5 half-
lives of their administration, no neuromuscular blocking
agents (or an intact train-of-4 response to peripheral
stimulation), and a physical examination that identifies no
evidence of cortical activity or brainstem reflexes. There
must also be no severe acid-base or endocrine
abnormalities and the patient must be judged to be
euvolemic. This patient has evidence of an accelerated
urine flow that, in light of the CT with evidence of midline
shift, is concerning for diabetes insipidus. The duration of
time that the urine flow has been increased should raise
the suspicion that this patient is not euvolemic.
• An appropriate investigation should be undertaken and
euvolemia restored, the therapy for which may include
plasma volume expansion and the provision of intranasal
desmopressin or IV vasopressin. Once euvolemia is
satisfied, the examination should be repeated, and then if
there are no changes, one may proceed with an apnea
test. If the apnea test demonstrates an increase in
PaCO2 to 60 mm Hg from 40 mm Hg, or an increase in
PaCO2 of 20 mm Hg from the pretest baseline without
evidence of respiratory efforts, no ancillary test such as
cerebral angiography or a nuclear medicine flow study is
needed to declare brain death. To guard against hypoxia,
intratracheal oxygen at a flow of 6 L/min may be provided.
Ancillary testing is required when the patient cannot
tolerate or complete the apnea test due to hypoxia or
hemodynamic instability.
• Question 80:
• An 18-year-old term female, who did not receive any
prenatal care, presents to the hospital and has an
uneventful full-term vaginal delivery. The next day, her
BP is 190/110 mm Hg, HR is 110/min, RR is 18/min, and
she is afebrile. Her urine output is noted to be 25 mL/h
for the past 5 hours.
• Which of the following interventions is most
appropriate?

A)IV furosemide

B)IV enalaprilat

C)IV magnesium sulfate

D)Plasma volume expansion

E)IV mannitol
• Correct Answer: C
• Rationale
This patient is most likely suffering from severe
preeclampsia, which can occur up to 1 week after delivery;
due to the postdelivery time frame, preeclampsia may be
less frequently considered in the differential diagnosis of
these symptoms than in the predelivery time frame. The
treatment of choice for elevated blood pressure is the
administration of magnesium sulfate to prevent
progression to eclampsia. The recommended dose is a 2-g
IV bolus, followed by a 1- to 2-g/h continuous IV infusion.
Further blood pressure control may be required if the
diastolic pressure remains above 100 mm Hg. Because
patients with preeclampsia/eclampsia are intravascularly
volume depleted, diuretics such as furosemide and
mannitol are not indicated. Although a fluid challenge may
be appropriate in this patient, continued observation
without intervention with magnesium is inappropriate.
Blood pressure control with an angiotensin-converting
enzyme inhibitor is not indicated.
• Question 81:
• A 20-year-old o a at eeks’ gestatio al age
presents to your emergency department with malaise
a d heada he of o e da ’s du atio . BP is /
Hg and pulse rate is 92/min; she is admitted to your
intermediate care unit for monitoring and care. Fetal
heart monitoring is reassuring and maternal urinalysis
reveals 3+ proteinuria. She suddenly develops a tonic-
clonic seizure and you are called for emergency
management.
• The most appropriate therapy at this time is to:

A)Administer diazepam, 10 mg IV

B)Administer lorazepam, 5 mg IV

C)Administer magnesium sulfate, 4 g IV

D)Perform an emergency cesarean section

E)Administer hydralazine, 10 mg IV
• Correct Answer: C
• Rationale
Eclampsia is the occurrence of seizures in a patient in the
setting of preeclampsia and in the absence of other
attributable causes. Clinical manifestations may appear
anytime from the second trimester to the puerperium.
The exact cause of eclamptic seizures is not known. Two
proposed hypotheses include (1) cerebral overregulation
in response to high systemic blood pressure resulting in
vasospasm of cerebral arteries and (2) loss of
autoregulation of cerebral blood flow in response to high
systemic pressure.
It is universally accepted that women with eclampsia
require anticonvulsant therapy to prevent further seizures
and the potential complications of repeated seizure
activity including but not limited to neuronal death,
rhabdomyolysis, metabolic acidosis, aspiration
pneumonia, neurogenic pulmonary edema, and acute
respiratory failure.
Magnesium therapy is a more effective treatment than
phenytoin or benzodiazepines for eclamptic seizures. The
Eclamptic Trial Collaborative Group, which conducted 2
prospective studies, demonstrated that magnesium sulfate
for prevention of recurrent seizures in women with
eclampsia can reduce the rate of recurrent seizures by
one-half to two-thirds and can reduce the rate of maternal
death by one-third. A series of systematic reviews
reported magnesium sulfate was safer and more effective
than phenytoin, diazepam, or the so- alled lytic o ktail
(ie, chlorpromazine, promethazine, and pethidine) for
prevention of repeat seizures in eclamptic women.
Additional advantages of magnesium sulfate therapy
include reduced cost, ease of administration (IV or
intramuscular), and less sedation than either diazepam or
phenytoin. Magnesium also appears to selectively increase
cerebral blood flow and oxygen consumption in women
with preeclampsia; these observations are not true for
phenytoin.
Antihypertensive therapy is not routinely administered
to women in the preeclamptic nor in the eclamptic
period as this has not been shown to alter the course of
the disease, nor to diminish perinatal morbidity or
mortality. Therapy is reserved for women in whom
blood pressure is elevated to a degree concerning for
intracranial hemorrhage.

Resolution of maternal seizure activity is associated with


compensatory fetal changes that can be identified as a
nonreassuring fetal heart rate tracing. If the fetal tracing
remains nonreassuring for more than 10 to 15 minutes
with no improvement despite maternal and fetal
resuscitative interventions, then the possibility of occult
abruption and emergent delivery should be considered.
• Question 82: The best policy for the use of Rh-immune
globulin in a pregnant patient with minor abdominal
trauma from a motor vehicle accident is to administer it to
Rh-negative women in:

A)Any stage of pregnancy if they are primiparous

B)The second trimester of pregnancy regardless of parity

C)The third trimester of pregnancy regardless of parity

D)Any stage of pregnancy with a positive Kleihauer-Betke


test

E)Any stage of pregnancy if they require blood transfusion


• Correct Answer: A
• Rationale
Even minor trauma can be associated with
significant fetomaternal hemorrhage. Guidelines
suggest that all Rh-negative pregnant women with
abdominal trauma should be given anti-D
immunoglobulin. Rh antigen is developed fully in
fetuses by 6 weeks of gestation, therefore
alloimmunization can occur in the first trimester. The
Kleihauer-Betke test can quantify fetomaternal
hemorrhage greater than 0.5 mL, but maternal
exposure to as little as 0.15 mL can elicit
immunization. A negative test does not obviate the
need for administration of Rh-immune globulin. The
Kleihauer-Betke test is important to determine the
need for additional dosing based on quantity of
fetomaternal hemorrhage.
• Question 83:
• A 39-year-old woman who is 29 weeks pregnant is
admitted to the hospital with vaginal bleeding and right
upper quadrant (RUQ) abdominal pain. Within 8 hours of
admission she suffers fetal loss due to acute and complete
placental abruption. Several hours later, she develops
hypotension, tachycardia, and reports increased RUQ pain.
Laboratory values at that time are as follows: sodium, 130
mEq/L; potassium, 5.1 mEq/L; blood urea nitrogen, 34
mg/dL; creatinine, 2.1 mg/dL; leukocytes, 11,000/µL;
hemoglobin, 5.9 g/dL; hematocrit, 17%; platelets,
32,000/µL; international normalized ratio, 1.1; partial
thromboplastin time, 26 seconds; total bilirubin, 1.2
mg/dL; lactate dehydrogenase, 20,320 U/L; aspartate
aminotransferase, 2,340 U/L; alanine aminotransferase,
2,100 U/L; gamma-glutamyltransferase, 40 U/mL. CT of
the abdomen obtained to evaluate the right upper
quadrant pain is shown in the Figure.
The patient is transferred to the ICU. Which of the
following management options is most appropriate at
this point?

A)Transfusion of packed red blood cells and platelets

B)Emergent laparotomy for hemorrhage control

C)Right upper quadrant CT-guided drainage

D)Plasmapheresis with O-negative plasma

E)Listing the patient for emergency hepatic


transplantation
• Correct Answer: A
• Rationale
This patient has developed a hepatic hematoma
(exemplified by the less dense crescent-shaped rim around
the lateral aspect of the liver on the CT) as a complication
from HELLP (hemolysis, elevated liver
function, low platelet count) syndrome.
HELLP syndrome is characterized by the following clinical
findings: (1) elevated liver enzyme levels (aspartate and
alanine aminotransferase); (2) microangiopathic hemolytic
anemia; and (3) thrombocytopenia (platelet count
<100,000/µL). Patients develop HELLP syndrome in the
third trimester of pregnancy, and up to 30% may develop
the syndrome postpartum. Typical presenting symptoms
are abdominal pain (right upper quadrant), malaise,
nausea, and vomiting. Complications include disseminated
intravascular coagulation, placental abruption, renal
failure, and liver hematomas.
Treatment for HELLP syndrome has traditionally
involved expeditious termination of pregnancy.
Conservative treatment may be indicated in selected
cases allowing improved fetal maturity. Treatment of
postpartum HELLP syndrome is mostly supportive. The
most feared complications of this syndrome are liver
subcapsular hematoma and rupture with shock.

The CT shown in the question illustrates a hepatic


hematoma with an intact capsule. Treatment for these
cases is supportive. Transfusion of platelets (platelet
count <50,000/µL with evidence of bleeding) and
packed red blood cells is appropriate in this case.
Surgery is reserved for cases of rupture and shock.
There is no indication for a percutaneous drainage,
which could lead to serious complications.
With a normal partial thromboplastin time and
international normalized ratio, fresh frozen plasma is not
indicated. Steroids have been used in HELLP syndrome
with mixed results, but a recent Cochrane Review showed
no improvement in overall outcomes, but did show better
platelet count recovery in patients who received
dexamethasone (versus placebo and/or betamethasone).
Plasmapheresis has been recommended in HELLP
syndrome lasting more than 72 hours after delivery based
on case reports. Although possibly helpful in some
circumstances, the use of plasma exchange would not be
the first treatment option in this case.

A liver transplant may be required in rare cases when


surgical removal to control a bleeding hepatic hematoma
is required. Again, at this point it is not indicated, as most
patients will recover with aggressive supportive care.
• Question 84:
• A previously healthy, 36-year-old, gravida 2, para 2
o a at eeks’ gestatio ith a u o pli ated
pregnancy is found to have a hemoglobin level of 11.6
g/dL, platelet count of 40,000/µL, and creatinine level of
2.6 mg/dL. The rest of her laboratory values are normal.
Her BP is 160/100 mm Hg, HR is 88/min, and RR is
16/min. Urinary output is less than 30 mL/h for the past
3 hours.
• The most appropriate next step is to:

A)Begin induction of labor with oxytocin

B)Perform urgent plasmapheresis

C)Perform a transthoracic echocardiography

D)Perform a cesarean section delivery

E)Begin plasma expansion with 3% saline


• Correct Answer: A
• Rationale
This patient has anemia, thrombocytopenia, and renal
insufficiency. This triad is associated with thrombotic
thrombocytopenic purpura (TTP) and hemolytic uremic
syndrome (HUS). The distinction between the 2 is that
neurologic symptoms predominate in TTP and renal failure
is the hall a k of HU“. This patie t’s i easi g creatinine
level and decreasing urinary output suggest she has HUS.
This condition has a very high maternal and fetal mortality.
The clinical presentation can be confused with severe
sepsis and disseminated intravascular coagulation, which
are more common and should be ruled out as soon as
possible. The placental vessels can thrombose so that a
viable fetus should be delivered as soon as possible.
Induction with oxytocin and a vaginal delivery would be
preferable to performing a cesarean section in this
expectedly coagulopathic woman. There is some evidence
that HUS can resolve with the delivery.
Since there could be some confusion that this
represents severe preeclampsia with or without HELLP
syndrome (hemolysis, elevated liver function, and low
platelets), prompt delivery of the fetus would also be
indicated. Failure to respond following delivery confirms
the diagnosis of HUS over these other causes.

Further management of the patient who fails to improve


following delivery is plasmapheresis. While
consideration for plasmapheresis prior to delivery
should be entertained, induction of labor should begin
as soon as the diagnosis is ascertained. While the
patient probably would benefit from volume expansion
and hemodynamic monitoring, they are secondary
considerations in the management scheme.
• Question 85:
• Which of the following clinical conditions most likely
suggests the diagnosis of amniotic fluid embolism rather
than venous air embolism?

A)Sudden cardiovascular collapse

B)Occurrence of embolism during labor

C)Diffuse bilateral pulmonary infiltrates

D)Disseminated intravascular coagulation

E)PaO2/FIO2 ratio <200


• Correct Answer: D
• Rationale
Amniotic fluid and venous air embolism may present
with sudden cardiovascular collapse. Both may occur
during labor. Amniotic fluid embolism is usually
associated with a prolonged or difficult labor, and
venous air embolism may be associated with a normal
labor with placenta previa. Both types of embolism may
result in release of inflammatory mediators and
pulmonary vasoconstriction with diffuse pulmonary
infiltrates. The presence of disseminated intravascular
coagulation (DIC) is more typical of amniotic fluid
embolism. If one survives the early cardiorespiratory
collapse, DIC with frank bleeding commonly occurs over
the next 1 to 2 hours.
• Question 86:
• A 30-year-old, pregnant woman who has no prenatal care
presents to the hospital at 34 weeks of gestation with
pedal edema and hypertension and is admitted to the ICU.
Initial laboratory data include the following: blood urea
nitrogen, 22 mg/dL; creatinine, 1.6 mg/dL; alanine
aminotransferase, 350 U/L; bilirubin, 5 mg/dL; ammonia,
70 mg/dL; glucose, 50 mg/dL; leukocytes, 19,000/µL;
hemoglobin, 9.6 g/dL; platelets, 120,000/µL; prothrombin
time, 20.2 seconds; partial thromboplastin time, 45
seconds. On physical examination, she is slightly lethargic,
with scattered bilateral basilar rales. A normal-sized liver is
appreciated on percussion and an appropriately sized
uterus is palpable and nontender. A fetal ultrasonogram is
unremarkable except for an unexpected placenta previa.
Which of the following is the most likely diagnosis?

A)Acute viral hepatitis

B)Life-threatening preeclampsia

C)Acute HELLP syndrome

D)Intrahepatic cholestasis of pregnancy

E)Acute fatty liver of pregnancy


• Correct Answer: E
• Rationale
There are several potential etiologies of liver disease in
pregnancy and distinguishing between them may be
difficult. The laboratory data in this patient are most
consistent with acute fatty liver of pregnancy, which often
presents in a fulminant manner. The table below lists
some of the helpful laboratory findings in acute fatty liver,
eclampsia/preeclampsia, and HELLP
(hemolysis, elevated liver enzyme levels, low platelet
counts) syndrome. In acute fatty liver of pregnancy, the
liver size is normal or small, and the aspartate
aminotransferase (AST) levels rise to higher levels than in
eclampsia/preeclampsia or HELLP syndrome. Signs of
hepatic failure, such as higher levels of bilirubin,
hypoglycemia, elevated ammonia level, and disseminated
intravascular coagulation (DIC), are often present.
Hepatitis in pregnancy presents with similar laboratory
findings to those of a nonpregnant patient, with higher
levels of transaminases, absence of DIC, and often an
enlarged liver. Intrahepatic cholestasis of pregnancy is a
benign condition associated with pruritus, jaundice,
elevated transaminase levels, elevated bilirubin level,
and normal prothrombin time. The most difficult
distinction in a pregnant patient is liver disease due to
eclampsia/severe preeclampsia and HELLP syndrome.
The ammonia level may be particularly helpful in these
situations. Consultation with a maternal fetal specialist
is warranted for consideration of immediate delivery.
• Question 87:
• A 21-year-old, primigravida, Rh-positive woman at 18
eeks’ gestatio is ad itted to the ICU follo i g a
motor vehicle crash for observation. She was a
restrained driver and tested positive for alcohol and
cocaine. Her BP is 140/60 mm Hg, HR is 110/min, and
the fetal heart rate is 120/min. The patient now reports
new-onset uterine contractions and has a moderate
amount of vaginal bleeding.
• The most appropriate next step is to:

A)Obtain pelvic radiograph to determine the need for


angiography and embolization
B)Perform a transvaginal level 2 ultrasonography
evaluating for fetal injury
C)Administer Rho(D) immunoglobulin, 300 mg IV,
immediately
D)Obtain ultrasonography for a hypoechoic collection
between the placenta and myometrium
E)Proceed with an emergency bedside cesarean section
for uterine rupture
• Correct Answer: D
• Rationale
This patient most likely has a placental abruption and,
because of the gestational age, the fetus would be
nonviable. Survival of the fetus depends on survival of the
mother and the focus should be on keeping the mother
stable. Major trauma is associated with placental
abruption; depending on the severity of the trauma,
abruption rates are reported to be between 1% and 5%.
Motor vehicle crashes are one of the leading causes of
abruption and uterine rupture; the most likely mechanism
for abruption is from mechanical shearing forces acting on
the placenta and uterine stretching during deceleration.
The diagnosis of abruption is usually made with a high
index of suspicion and ultrasonography. Early hemorrhage
on sonogram is typically hyperechoic or isoechoic,
whereas resolving hematomas are hypoechoic within 1
week and sonolucent within 2 weeks.
While an assessment of the bony pelvis is necessary,
radiography alone would not dictate the need for
embolization, especially in the face of stable vital signs. If
the physical exam suggests a major pelvic fracture, then
CT of the pelvis with contrast would be indicated.
Extravasation of contrast on the CT with hemodynamic or
fetal instability would be an indication for
angioembolization. CT should not be withheld when
indicated in the pregnant trauma patient.
Rho(D) immunoglobulin is administered to mothers with
placental abruption when they are Rh-negative only.
Since the fetus is not viable, an emergency cesarean
section is not an option, and it is unlikely for a fetal
ultrasonography to lead to any useful predelivery
interventions.
• Question 88:
• A 37 year-old, gravida 5, para 5, vaginally delivered her
4,200-g (9.4-lb) infant after laboring for 24 hours. The
patient has preeclampsia and is transferred to your ICU
for acute management. On transfer she has BP of
140/90 mm Hg and a pulse rate of 130/min. She has an
estimated blood loss of 1.2 L.
The most appropriate management of this patient's
condition is:

A)Transfusion of 2 U of type-specific fresh frozen plasma

B)Administration of 90 µg/kg of recombinant factor VIIa

C)Administration of methylergonovine, 0.2 mg


intramuscularly

D)Administration of carboprost tromethamine, 250 µg


intramuscularly

E)Emergent total abdominal hysterectomy


• Correct Answer: D
• Rationale
Postpartum hemorrhage (PPH) is an obstetrical emergency
that may follow vaginal or cesarean delivery. It is a major
cause of maternal morbidity, with sequelae such as shock,
renal failure, acute respiratory distress syndrome,
coagulopathy, and Sheehan syndrome (ie, postpartum
hypopituitarism). PPH is also one of the top 5 causes of
maternal mortality in both high and low per capita income
countries.
PPH is a clinical diagnosis that is best defined as
postpartum bleeding that makes the patient symptomatic
(eg, light-headedness, vertigo, syncope) and/or results in
signs of hypovolemia (eg, hypotension, tachycardia, or
oliguria). The most commonly associated hemorrhage
volumes are greater than or equal to 500 mL after vaginal
birth or at least 1,000 mL after cesarean delivery.
The most common causes of PPH are, in order of
decreasing frequency, atony (most commonly secondary
to overdistension, drugs, retained placenta, uterine
infection), trauma (lacerations, incisions, or uterine
rupture), and preexisting coagulation defects. Additionally,
placenta previa, history of previous PPH, obesity, high
parity, Asian or Hispanic race, precipitous labor, and
preeclampsia have also been associated with PPH.
Initial management strategies should include fundal
massage to stimulate uterine contraction, IV access with
large-bore catheters, and administration of uterotonic
drugs. Since atony is the most common cause of PPH, the
following uterotonic drugs are administered in sequence
until a therapeutic effect is observed:
Oxytocin, 40 units in 1 L of normal saline IV or 10 units
intramuscular (IM) (including directly into the
myometrium)
Methylergonovine, 0.2 mg IM (including directly into the
myometrium, never IV) if no hypertension, Reynaud
phenomenon, or scleroderma. May repeat at 2- to 4-hour
intervals.
Carboprost tromethamine (prostaglandin F2-alpha), 250 µg
IM (including directly into the myometrium) every 15 to 90
minutes as needed, to a total dose of 2 mg if no
bronchospasm.
Misoprostol (prostaglandin E1) is also effective, although
data are more limited and the optimum dose and route
are unclear. Unlike methylergonovine and carboprost,
misoprostol may be given to women with hypertension or
reactive airways diseases.
Dinoprostone (prostaglandin E2), 20 mg via vaginal or
rectal suppository, which can be repeated in 2 hours, is an
alternative prostaglandin E to misoprostol.
Fluid resuscitation and baseline laboratory evaluation
should also be employed.
If the initial interventions described above are not
immediately successful in controlling hemorrhage, full
examination of the birth canal with the woman in stirrups
in an operating suite should occur. Blood product
replacement may be necessary with concurrent initiation
of invasive procedures such as arterial embolization,
balloon tamponade, uterine compression sutures, or
uterine devascularization. These interventions have
successfully controlled hemorrhage in 85%-90% of
reported cases. Hysterectomy is the last resort, but should
not be delayed in women who require prompt control of
uterine hemorrhage to prevent death.
Human recombinant factor VIIa has been approved by the
US Food and Drug Administration for treatment of
individuals with bleeding related to hemophilia A and B
inhibitors, acquired inhibitors, and congenital factor VII
deficiency. It has also been used successfully for control of
bleeding in situations of intractable bleeding associated
with postpartum uterine atony, placenta accreta, or
uterine rupture, although this is currently an off-label use.
• Question 89:
• Which of the following tests most appropriately
distinguishes between acute fatty liver and HELLP
(hemolysis, elevated liver function, low platelets)
syndrome in pregnancy?

A)Serum ammonia level

B)Serum bilirubin level

C)Ferritin level

D)Disseminated intravascular coagulation profile

E)Liver biopsy
• Correct Answer: A
• Rationale
During pregnancy, women may develop acute fatty liver,
HELLP syndrome, or eclampsia, and they may very well
have overlapping of these conditions. The differential
diagnosis between acute fatty liver and other conditions is
important to make because subcapsular hematoma and
rupture are common and can quickly progress to
fulminant liver failure. Moreover, acute fatty liver can
affect the fetus with significant muscle dysfunction and is
said to be associated with sudden infant death
syndrome. Extreme vigilance is very important. Since usual
onset is around 36 weeks of gestation, the treatment is
delivery of the fetus.
Increased ammonia level is highly suspicious for acute
fatty liver when accompanied by clinical progression.
Disseminated intravascular coagulation, anemia, and
hyperbilirubinemia are universally common in eclampsia,
HELLP syndrome, and acute fatty liver, so options B, C, and
D are incorrect .
• Question 90:
• Twenty-four hours after a laparoscopic removal of an
ectopic pregnancy, a 28-year-old woman with chronic
alcohol abuse is admitted to the ICU for early alcohol
withdrawal syndrome. She has been treated with
increasing doses of lorazepam that were ineffective, and
a clonidine patch and scheduled haloperidol are added.
Two hours later she develops fever to 39°C (102.1°F)
and is increasingly agitated. Laboratory studies indicate
a WBC count of 18,000/µL. A diagnosis of neuroleptic
malignant syndrome is entertained.
Which of the following best describes this diagnosis?

A)It shares a similar molecular etiology with malignant


hyperthermia.

B)It only occurs with prolonged dosing.

C)The condition is self-limiting but aided by using an


alpha-blocker.

D)Leukocytosis essentially excludes a pharmacologic


etiology for fever.

E)The diagnosis is often delayed, and is associated with


antiemetic use.
• Correct Answer: E
• Rationale
Neuroleptic malignant syndrome is a life-threatening
condition that is caused by an alteration of the central
dopamine neurotransmission, in contrast to the
hypermetabolism of muscle seen in malignant
hyperthermia; the 2 conditions do not share a similar
molecular underpinning. Psychiatric and postoperative
patients who have nausea are at risk.
Neuroleptic malignant syndrome can be caused by
dopamine antagonists such as prochlorperazine,
promethazine, droperidol, and metoclopramide that are
commonly used as antiemetics; typical and atypical
antipsychotic medications used for sedation are similarly
implicated. Symptoms may include psychomotor agitation,
rigidity, akinesia, dystonia, fever greater than 38°C
(100.4°F), autonomic instability, leukocytosis, and elevated
creatine kinase.
Moderate elevations in liver function test results are
often present. Treatment is the discontinuation of the
offending agent, supportive care, and the administration
of bromocriptine and/or dantrolene. Alpha-blocking
agents have no role in the management of this
condition.

The diagnosis of neuroleptic malignant syndrome can be


confounded in patients with psychiatric problems. It can
be insidious in ICU patients who may have multiple
causes of leukocytosis, fever, and agitation. It may be
difficult to differentiate from the hypermetabolism of
severe early alcohol withdrawal syndrome in the chronic
alcohol abuse patient population as well.
• Question 91:
• Following a modified radical neck dissection, a 72-year-
old man is admitted to the ICU for routine progressive
care. Pertinent past medical history includes
hypertension treated with lisinopril. Shortly after
admission, the nurse notices that his tongue appears to
be larger than upon admission. When you arrive, the
nurse informs you that the tongue has now doubled in
size since you were called to evaluate him. The patient
has become increasingly anxious.
Which of the following is the most appropriate
treatment at this time?

A)Nebulized racemic epinephrine and 100% oxygen

B)Diphenydramine, 50 mg, and hydrocortisone, 100 mg

C)Nebulized albuterol with a 70/30 heliox admixture

D)Intubation by an anesthesiologist with a surgeon


present

E)Elevation of the head of the bed to 30°


• Correct Answer: D
• Rationale
Angioedema that can cause a rapid and progressive
airway compromise can occur with the use of
angiotensin-converting enzyme (ACE) inhibitors. ACE
inhibitors cause 25%-39% of nonhereditary
angioedemas. It has been suggested that the incidence
of angioedema with lisinopril is greater than with the
other ACE inhibitors. The exact pathophysiologic
mechanism is unknown, but it may involve inhibition of
kinase activity with the accumulation of tissue
mediators. Up to 20% of patients may present with
acute dyspnea, dysphagia, dysphonia, and stridor.
As many as 22% of patients with angioedema induced
by ACE inhibitors require airway intervention, which
may include cricothyroidotomy if attempts at direct
visualization of the larynx fail. Once the airway is
secured, the angioedema is self-limiting and resolves
with withdrawal of the offending agent. There is no
evidence that pharmacologic intervention effectively
treats this type of angioedema. Heliox has not been
demonstrated to be helpful, and the rapidly progressive
nature of the tongue swelling should prompt urgent
airway control.
• Question 92:
• A 50-year-old man has a history of chronic alcohol
abuse, alcoholic hepatitis and pancreatitis, poorly
controlled hypertension, and chronic kidney disease
with a baseline creatinine level of 1.7 mg/dL. He was
admitted to the hospital 10 days ago with symptoms
consistent with alcohol withdrawal. He became
increasingly agitated despite escalating doses of
benzodiazepines. He was transferred to the ICU on
hospital day 3 after developing respiratory failure after
aspirating during a witnessed generalized tonic-clonic
seizure. A lorazepam infusion managed his alcohol
withdrawal delirium, and ampicillin/sulbactam
addressed right lower lobe aspiration pneumonia.
He has done well, and his lorazepam dose has been
steadily declining from a peak of 15 mg/h to 10 mg/h.
Yesterday, he appeared lightly sedated and was able to
briefly follow verbal commands. You are called to
evaluate him because of lethargy. Laboratory results
show the following: sodium, 140 mEq/L; chloride, 110
mEq/L; serum bicarbonate, 12 mEq/L; blood urea
nitrogen, 80 mEq/L; and creatinine, 2.4 mg/dL. His
arterial blood gas results show a pH of 7.28, PaCO2 of 27
mm Hg, and PaO2 of 100.
Which of the following is the most appropriate next
step given this clinical context?

A)Discontinue lorazepam infusion and check serum


osmolality.

B)Start a bicarbonate infusion to raise the pH to >7.30.

C)Obtain a urinalysis and check for urine eosinophils.

D)Consult nephrology for possible initiation of


hemodialysis.

E)Obtain an immediate CT of the brain to evaluate for


stroke.
• Correct Answer: A
• Rationale
The patient in question probably has propylene glycol
toxicity. Propylene glycol is used as a solvent for IV
preparations of lorazepam. Those receiving high-dose
lorazepam infusion with a large cumulative dose are at
risk. (Some studies have used a threshold of 1 mg/kg/day,
while others have identified 10 mg/h and 0.1 mg/kg/h.)
Other patient-related factors that predispose to this
toxicity include alcohol abuse, renal and hepatic
dysfunction, pregnancy, and impairment of the alcohol
dehydrogenase enzyme system (eg, as seen in young
children). The patient in this case had multiple risk factors.
Accumulation of propylene glycol is associated with an
anion gap metabolic acidosis, osmolar gap, lactic acidosis,
acute renal tubular injury, cardiac arrhythmias, and
hypotension. Patients may develop central nervous system
abnormalities that typically manifest as confusion and
lethargy. In severe cases, seizures and coma can be seen,
and multisystem organ failure can mimic sepsis.
The osmolar gap is strongly correlated with serum
propylene glycol concentrations and can support the
diagnosis of propylene glycol toxicity. When there is
sufficient suspicion, the offending agent—lorazepam—
should be promptly discontinued. If further
benzodiazepine treatment is deemed necessary,
alternative agents such as midazolam (which does not
have propylene glycol as a solvent) can be used.

Option B, bicarbonate infusion, may temporarily


improve the acidemia, but does not address the
potential cause (lorazepam) or help define it further.
The patient has been receiving ampicillin/sulbactam,
which has been associated with acute interstitial nephritis.
Assessing the urinalysis for active sediments and checking
for eosinophiluria could further support the diagnosis, but
acute interstitial nephritis is best confirmed via biopsy.
Many patients with drug-induced acute interstitial
nephritis have other features of an allergic-type reaction,
such as rash, fever, and eosinophilia (but there is no
mention of such symptoms in our patient). Furthermore,
the onset of the renal dysfunction usually occurs over
weeks and months after first exposure to an offending
drug, or 3 to 5 days after the second exposure. Given the
overall clinical context, option C should therefore not be
the next step to further elucidate the etiology of the
worsening kidney function, lethargy, and metabolic
derangements.
Option D, initiation of hemodialysis, is typically reserved
for patients with evidence of life-threatening toxicity
manifesting with severe metabolic derangements and/or
multisystem organ failure. Propylene glycol is a small-
molecular-weight alcohol with no significant protein
binding and is readily cleared via hemodialysis. Before
considering initiation of dialysis, however, additional
evidence should be sought to support the diagnosis of
propylene glycol toxicity. Although direct serum propylene
glycol levels can be measured, this test is not readily
available. As such, the osmolar gap is a useful surrogate
marker and has been shown to correlate with serum
propylene glycol concentrations. Without lateralizing
signs, a brain CT is less helpful than evaluating for the
unintended side effects of the currently applied therapy.
Therefore option A, as described above, would be the
most appropriate next step.
• Question 93:
• A 55-year-old man with alcohol dependence is admitted
with status epilepticus and intubated for airway
protection. Seizures are initially controlled with bolus
doses of lorazepam and loading with phenytoin. He is
initially sedated with continuous infusion of lorazepam at
2 mg/h but requires increasing doses (10-12 mg/h) by 24
hours because of manifestations of alcohol withdrawal.
After control of seizures, his baseline laboratory results
show the following: sodium, 132 mEq/L; chloride, 100
mEq/L; potassium, 3.4 mEq/L; bicarbonate, 22 mEq/L;
blood urea nitrogen, 22 mg/dL; creatinine, 1.5 mg/dL; and
glucose, 120 mg/dL. Three days into the ICU course, the
patient has a bicarbonate level of 16 mEq/L, with an anion
gap of 18 mEq/L, creatinine level of 1.8 mg/dL, and lactate
level of 2.0 mg/dL. He has an osmole gap of 24 mOsm/kg.
He remains sedated, with HR of 90/min, BP of 104/74
mm Hg, RR of 12/min (on ventilator), and temperature
of 36.9°C (98.4°F). Empiric antibiotics are started. Which
of the following interventions is most appropriate?

A)Initiate hemodialysis to clear fixed acids.

B)Stop lorazepam and start dexmedetomidine infusion.

C)Stop lorazepam and start midazolam infusion.

D)Stop lorazepam and start fentanyl infusion.

E)Administer 5% dextrose in water and 150 mEq/L of


sodium bicarbonate at maintenance rate.
• Correct Answer: C
• Rationale
This clinical situation and the blood chemistry findings in
this patient suggest propylene glycol toxicity. Each vial of
lorazepam (2 mg/mL) contains 830 mg/mL of propylene
glycol. The most common manifestations of propylene
glycol accumulation are anion-gap metabolic acidosis and
increased osmole gap. Approximately 12%-45% of
propylene glycol is excreted unchanged in the urine in
healthy individuals, and the remainder is metabolized by
the liver to lactate and pyruvate. Accumulation may occur
when doses exceed the upper recommended dose of 0.1
mg/kg/h. Renal and/or hepatic insufficiency may also play
a role in accumulation. Serious reported toxicities include
renal dysfunction, hemolysis, cardiac arrhythmias,
seizures, and central nervous system depression or
agitation. The metabolic findings of propylene glycol
toxicity may also be confused with sepsis or severe
inflammatory states.
Clinical studies suggest that the osmole gap correlates
with propylene glycol accumulation. Although toxicity is
more common after long periods of lorazepam infusion
da s , to i it has o u ed ith sho t-term and high-
dose use. The treatment of choice is to stop the lorazepam
infusion and sedate with an agent that does not contain
propylene glycol. Diazepam contains propylene glycol, but
midazolam does not. It would not be appropriate to
change to fentanyl alone in a patient who requires
benzodiazepines for alcohol withdrawal. Hemodialysis
would remove propylene glycol but is not required unless
severe renal dysfunction develops. There is a case report
of propylene glycol toxicity developing in a patient
receiving continuous renal replacement. Propylene glycol
is metabolized by alcohol dehydrogenase, but there are no
reports of using ethanol infusion or fomepizole for
toxicity.
• Question 94:
• A patient presents with severe community-acquired
pneumonia accompanied by tachycardia, hypotension,
and hypoxemia. Intubation, mechanical ventilation,
central venous access, and arterial catheter insertion is
accomplished in the emergency department. Despite
2,000 mL of crystalloid fluid, the tachycardia and
hypotension does not improve. Norepinephrine is
added, but the mean arterial pressure does not reach 65
mm Hg. Vasopressin is added to the regimen.
Which of the following best characterizes vasopressin in
this setting of septic shock?

A)Despite normal to high levels, vasopressin receptors


are down-regulated.
B Vasop essi ’s effe t o hemodynamics are similar in
healthy volunteers.
C)Vasopressin increases platelet counts and
microvascular thrombosis risk.
D)Doses >0.04 U/min decrease cardiac output because
of negative chronotropy.
E)Tachyphylaxis commonly develops with therapy
prolonged >96 hours.
• Correct Answer: D
• Rationale
Circulating levels of vasopressin are elevated during
hypovolemic, cardiogenic, and obstructive shock.
However, such a sustained response does not occur in
septic shock. Initially, vasopressin levels are significantly
elevated; over the ensuing days of sepsis/septic shock,
these levels fall, approaching normal levels between
days 2 and 3. There is no evidence that vasopressin
receptors are down-regulated with septic shock. With
increasing doses of vasopressin, blood flow is reduced
to the skin, skeletal muscle, bowel and fat.
Vasopressin induces an increase in mean arterial
pressure and a decrease in cardiac output, mainly linked
to negative chronotropic effect. Rebound hypotension
often occurs when the drug is discontinued. Vasopressin
can significantly decrease platelet count during infusion.
Vasopressin at doses of 0.04 U/min or less has a
minimal effect on the cardiac index. Normal individuals
have no blood pressure effects during exogenous
administration of vasopressin up to levels of 0.26 U/min.
However, for patients in shock, a hypersensitivity to
vasopressin has been noted, with a concomitant
decrease in the effectiveness of the catecholamines.
• Question 95:
• A 36-year-old woman is noted to be unresponsive.
Emergency medical service personnel found her in a
parked car with its engine running. She is intubated
for airway protection and brought to the hospital. BP
is 96/54 mmHg, HR is 115/min, RR is 12/min, and
temperature is 36.1°C (96.9°F). Physical examination
is remarkable for a Glasgow Coma Scale score of 6,
tachycardia, and bibasilar rales. Pupils are 3 mm and
sluggishly reactive. Laboratory examination reveals
the following: arterial blood gas pH, 7.27; PaCO2, 35
mm Hg; PaO2, 365 mm Hg (while receiving 100%
oxygen); anion gap, 18 mmol/L; osmolar gap, 12
mmol/L; serum ethanol, negative;
carboxyhemoglobin, 20%; serum salicylate, pending;
serum acetaminophen, pending. ECG shows
nonspecific ST-segment changes.
Which of the following interventions is most
appropriate for this patient at this time?

A)Administration of flumazenil

B)Administration of naloxone

C)Hemodialysis with hemoadsorption

D)Continuing 100% oxygen

E)Administering hyperbaric oxygen


• Correct Answer: E
• Rationale
This patie t’s p ese tatio suggests p o a le a o
monoxide poisoning for several reasons. She was found in
the appropriate setting, the clinical findings are consistent
with carbon monoxide toxicity, and the laboratory results
also support exposure to carbon monoxide. The best
confirmation of significant carbon monoxide exposure is
an elevated carboxyhemoglobin level (venous or arterial
sample), but the blood gas must be run on a co-oximeter
device to detect the moiety that is saturation of the
hemoglobin. The standard arterial blood gas device only
measures 2 wavelengths of light and will not be able to
determine whether there is carbon monoxide attached to
the hemoglobin or not, while the co-oximeter measures 6
different wavelengths and can render that determination.
However, the initial level may not be helpful if significant
time has elapsed since exposure. Carboxyhemoglobin
levels of up to 10% can be found in people in some urban
areas and in heavy smokers.
Pulse oximetry often overestimates oxygenation in the
setting of carbon monoxide toxicity, so it is not helpful in
the assessment of patients.
The presence of metabolic acidosis suggests tissue
hypoxia, and a lactate level is more reliable than the
carboxyhemoglobin level in determining the severity of
toxicity. Although there is continued debate on the
indications for hyperbaric oxygen therapy, most clinicians
would agree that the presence of significant neurologic
impairment would warrant treatment. Hemoglobin binds
carbon monoxide tightly (approximately 240 times greater
affinity than for oxygen) and forms a complex that is only
slowly reversible. This binding can be overcome by high
tissue levels of oxygen. The half-life of carboxyhemoglobin
falls from 4 to 6 hours while receiving room air, to
approximately 90 minutes while receiving 100% oxygen at
1 atmosphere of pressure, and to approximately 30
minutes while receiving 100% oxygen at 3 atmospheres of
pressure.
Although 100% oxygen shortens the half-life of
carboxyhemoglobin, a recent study suggests that it is
less effective at reducing postexposure cognitive deficits
than hyberbaric oxygen. There is no reason to
administer flumazenil or naloxone, because the patient
is supported by mechanical ventilation. There is no
indication for dialysis in this patient.
• Question 96:
• A 54-year-old man is admitted to the hospital because of
severe community-acquired pneumococcal pneumonia.
Acute respiratory distress syndrome develops and the
patient requires intubation and mechanical ventilation. He
is placed on assisted mandatory ventilation with a tidal
volume of 7 mL/kg of predicted ideal body weight. He
requires IV infusions of lorazepam and fentanyl to
maintain comfort and allow for adequate patient-
ventilator synchrony. On his fourth ICU day, his oxygen
requirement decreases. His current ventilator settings are
FIO2 of 0.7, positive end-expiratory pressure of 14 cm H2O,
and a set respiratory rate of 18/min. The patient is
breathing at 18-24/min. The minute ventilation is 10
L/min, and the static pressure is 29 cm H2O. Current
clinical trial evidence supports
which of the following treatment options for the
management of sedation and analgesia?

A)Discontinuation of infusion of lorazepam and fentanyl


daily with the resumption of infusion of sedatives as
needed for comfort, at a lower dose if tolerated
B)Discontinuation of infusion of lorazepam and fentanyl
and administration of sedatives as needed to maintain a
Richmond Agitation Sedation Score of –3 to –4
C)Continuation of infusion of lorazepam and fentanyl if the
patient is comfortable and weaning of sedation when
FIO2 is 0.5
D)Increase in fentanyl dosage so the patient does not
initiate breaths over the set rate and continuation of
lorazepam
• Correct Answer: A
• Rationale
Option A is correct, as previous prospective randomized
research has demonstrated that daily interruption of
sedatives is associated with shorter ICU stays, shorter
duration of mechanical ventilation and a decrease in
evaluations for altered mental status.

Option B is incorrect, as Richmond Agitation Sedation


Score of –3 to –4 describes a patient who is unconscious
and difficult to arouse. The goal is to titrate sedatives
and analgesics to so that the patient is comfortable, not
agitated, and easily arousable.
Option C is incorrect, as there are no clinical trials
demonstrating that sedation should only be weaned
when FIO2 is 0.5 or less.

Option D is incorrect. The Acute Respiratory Distress


Syndrome (ARDS) Network study permitted respiratory
rates up to 35/min.
• Question 97:
• A 35-year-old woman presents with acute liver failure.
She was feeling well until a few days ago when she
reported abdominal pain and fatigue. She has no
significant past medical history, but has been taking over-
the-counter acetaminophen for back pain and also takes
herbal supplements. She has no history of alcohol abuse
or illicit drug use. On the day before admission, family
members noted mild confusion. Admission laboratory
results include the following: blood urea nitrogen, 30
mg/dL; creatinine, 1.6 mg/dL; international normalized
ratio, 2.9; total bilirubin, 5.9 mg/dL; direct bilirubin, 4.3
mg/dL; aspartate aminotransferase, 3,556 U/L; alanine
aminotransferase, 2,998 U/L; and gamma-
glutamyltransferase, 112 U/L. On examination, she has
jaundice, mild confusion consistent with grade 1
encephalopathy and tenderness in the right upper
quadrant. Vital signs are stable. Results of toxicology
screen, including acetaminophen levels, are otherwise
negative.
Which of the following should be administered IV in the
initial management?

A)Fresh frozen plasma

B)Empiric antibiotics

C)N-acetylcysteine

D)Single-donor platelets
• Correct Answer: C
• Rationale
This patient is presenting with hyperacute liver failure,
defined as the development of jaundice to
encephalopathy within 7 days. This tempo of illness is
most commonly seen with viral hepatitides and
acetaminophen toxicity. Patients who present in a
hyperacute fashion have the highest likelihood of survival,
but also have the highest chance of developing cerebral
edema. Despite advances in ICU care, one-third of patients
with acute liver failure die, and the 3 most common
causes of death are cerebral edema, sepsis, and
multiorgan dysfunction syndrome.
As the synthetic function of the liver declines, significant
coagulopathy is often observed. Prophylactic reversal via
administration of blood products, however, is unwarranted
as this puts the patient at risk of transfusion-associated
circulatory overload (TACO). As many patients have
concurrent kidney dysfunction, volume management is
particularly crucial. Thus, option A is incorrect.
Furthermore, normalization of coagulation parameters
precludes trending the prothrombin time, which is a
powerful prognostic tool. Finally, no study has shown
that aggressive administration of blood products
significantly alters the risk of bleeding, even with
platelet counts down to 10,000/µL; lower platelet
counts are associated with spontaneous hemorrhage
and should be treated with platelet transfusion. This
observation may be underpinned in part by the
reduction in procoagulants as well as anticoagulants as
the liver sustains significant injury. Studies have shown
that subclinical vitamin K deficiency can occur in up to
25% of patients, so consensus guidelines recommend
empiric administration of vitamin K.
Sepsis is the leading cause of death in acute liver failure.
Thus, aggressive surveillance with periodic cultures and
vigilance for signs of infection are important. Although
most recommend a low threshold for antibiotic initiation
(for example, in cases of unexplained, refractory shock),
there are no definitive data to support the use of
prophylactic antibiotics. Therefore, option B is incorrect.
Common sites of infection include the bloodstream, urine,
and lungs. Catheter-related infections make up a
percentage of these infections, and therefore, routine
insertions of central venous catheters are ill advised until a
need for such specialized catheters arises. Therefore,
option D is incorrect. Most procedures (including
intracranial monitoring devices), can be safely inserted
with the use of blood products, which may include a
combination of fresh frozen plasma, cryoprecipitate (if
fibrinogen level is lower than 100 mg/dL), and
recombinant factor VII—if needed.
Although the acetaminophen level in this patient was
undetectable, N-acetylcysteine should be promptly started
for this patient. She has a history of acetaminophen use,
and it is important to remember that up to 50% of
acetaminophen-related acute liver injury arises from
therapeutic misadventures due to repeated dosing. As the
patient has already been ill for several days,
acetaminophen levels may no longer be detectable, as
they rapidly decline. In research settings, acetaminophen
protein adducts, which have a longer half-life, have been
tested. In up to 20% of patients classified as presenting
with "indeterminate" cause, positive protein adduct levels
were noted, suggesting unrecognized acetaminophen
overdose. Therefore, consensus guidelines recommend
administration ofN-acetylcysteine regardless of dose
ingested, levels, or timing.
This should be continued until there is consistent
evidence of clinical recovery. In general, IV N-
acetylcysteine is preferred among critically ill patients
(particularly those with signs of encephalopathy), as it
eliminates the risk of aspiration. Finally, the use of IV N-
acteylcysteine for non–acetaminophen related acute
liver failure was investigated by the US Acute Liver
Failure study group. In their multicenter, randomized
trial, the group demonstrated a significant improvement
in transplant-free survival among patients with early
encephalopathy (grade 1 or 2)—52% among the
intervention group versus 30% in the placebo group.
• Question 98:
• A 73-year-old woman had a left upper lobectomy 1 day
ago. She has been receiving continuous epidural
analgesia with 0.1% bupivacaine with fentanyl, 10
µg/mL at 5 mL/h. In the morning, she is somnolent but
awakens with stimulation. The morning arterial blood
gas study shows pH of 7.28, PaCO2 of 54 mm Hg, and
PaO2 of 100 mm Hg on oxygen, 2 L/min via nasal
cannula. Her BP is 136/84 mm Hg, HR is 62/min, and RR
is 8/min.
Which of the following represents the most appropriate
intervention?

A)Reduce the epidural infusion rate by 50% and reevaluate


in 1 hour.
B)Change the epidural infusion to 0.1% bupivacaine
without an opioid.
C)Administer naloxone, 80 µg IV push, and start a
naloxone infusion.
D)Remove the epidural and substitute patient-controlled
analgesia.
E)Start bilateral positive airway pressure to help clear
carbon dioxide and normalize the acidotic pH.
• Correct Answer: B
• Rationale
Low respiratory rate and hypercapnia indicate that this
patient has respiratory depression. When opiates are
given as part of an epidural analgesic regimen,
respiratory depression is a common result, caused by
the systemic absorption of opiates from the epidural
space. Epidural fentanyl infusions are equivalent to IV
administration. Because this patient was stable and not
in acute respiratory distress, changing the epidural
infusion to 0.1% bupivacaine without an opiate should
improve her respiratory status.
Although lowering the rate would limit the total opiate
given, it would also limit the amount of local anesthetic
and might not provide adequate analgesia. Similarly,
reversing the analgesic effect with naloxone may be
counterproductive. If the opioid is removed, the patient
should spontaneously improve her carbon dioxide
clearance. Additionally, with a fresh bronchial stump
closure, one would want to avoid positive pressure
ventilation unless absolutely necessary.
• Question 99:
• A 62-year-old man with myasthenia gravis is admitted to
the ICU after a tree limb falls on his left lower extremity
while he is shoveling snow. He has multiple facial
lacerations and a possible open globe injury from the
tree branches. Pertinent laboratory studies show +4
heme in urine and a creatinine level of 3.8 mg/dL
(335.92 mmol/L). Prior to transport to the operating
room for urgent ocular and leg explorations, he
becomes hemodynamically unstable and requires
endotracheal intubation.
Which of the following represents a contraindication to
the use of succinylcholine?

A)Acute renal failure

B)Open globe injury

C)Crush injury

D)Myasthenia gravis

E)Facial lacerations
• Correct Answer: C
• Rationale
Succinylcholine, a depolarizing neuromuscular blocker,
has many advantages, including its rapid onset of action
and its ultrashort duration of neuromuscular blocking
activity. However, this drug is associated with several
negative effects. In normal patients, succinylcholine
administration results in a serum potassium increase of
0.5-1.0 mEq/L. The administration of succinylcholine to
patients with burns, prolonged immobility, crush
injuries, and muscular dystrophies produces life-
threatening hyperkalemia secondary to the up-
regulation of extrajunctional receptors for acetylcholine
of the skeletal muscle membrane.
Patients with renal insufficiency often have elevated
potassium levels, but the extrajunctional receptors do
not proliferate. Although succinylcholine will increase
intraocular pressure, further damage to the eye with its
use during rapid-sequence inductions of general
anesthesia has not been borne out. Patients with
myasthenia gravis are resistant to succinylcholine.
• Question 100:
• Which of the following best characterizes
dexmedetomidine?

A)Potent amnestic agent

B)Twice as potent as clonidine

C)Rare respiratory depression

D)Provides no analgesic effect

E)Associated with hepatic failure


• Correct Answer: C
• Rationale
Alpha2 agonists are commonly used for sedation in the
critically ill. Dexmedetomidine is a selective
alpha2 agonist and is 8 times more potent than
clonidine. Advantages of dexmedetomidine in the
critically ill patient include the ability to relieve anxiety
and agitation and promote analgesia without clinically
significant respiratory depression. It can be used in an
unintubated patient. Dexmedetomidine has a mild
analgesic effect and opioid sparing effect. Common side
effects include bradycardia and hypotension. It is a good
anxiolyic, but is not an amnestic agent. There is no
association with hepatic failure.
• Question 101:
• A 45-kg (99-lb), 15-year-old male with severe acute
asthma does not improve following inhaled
bronchodilator and systemic steroid therapy and
becomes progressively hypoxemic. The decision is made
for intubation and mechanical ventilation. Following
administration of propofol, 100 mg, and succinylcholine,
50 mg, to facilitate intubation, his mouth cannot be
opened.
Which of the following is the most likely cause of
inability to open the mouth in this patient?

A)Inadequate dose of propofol

B)Succinylcholine-induced hypokalemia

C)Acute masseter muscle spasm

D)Hypovolemia and poor circulation time

E)Acute hypercalcemic tetanic spasm


• Correct Answer: C
• Rationale
Rapid-sequence intubation is commonly facilitated by a
combination of an induction agent and a short-acting
muscle relaxant such as propofol and succinylcholine. In
order to preserve the protective airway reflexes and
spontaneous ventilation, the use of muscle relaxant
should preferably be avoided in the critically ill. The
induction dose of propofol in a pediatric patient is 2-2.5
mg/kg of body weight. Succinylcholine is a depolarizing
neuromuscular blocking drug with a rapid onset (30-60
seconds) and ultrashort duration (5-10 min) of action.
These characteristics make it ideal for providing rapid
skeletal muscle relaxation to facilitate intubation. The
sustained depolarization produced by the initial
administration of succinylcholine is initially manifested as
transient generalized skeletal muscle contractions known
as fasciculations.
Incomplete jaw relaxation with masseter muscle spasm
is not uncommon in children (4.4% of patients) and
resolves when fasciculations stop. In extreme cases, this
response may be so severe that the ability to
e ha i all ope the patie t’s outh is li ited. It is
difficult to separate this condition from masseter muscle
rigidity associated with malignant hyperthermia.
Hypokalemia usually decreases muscle tone.
Hypovolemia and reduced circulation of intravenously
delivered agents delays the onset of activity of the
agents but is not a cause of masseter muscle spasm.
• Question 102:
• A patient requiring surgery with which of the following
conditions is most likely to have associated hyperkalemia
after the administration of succinylcholine?

A)24 hours after being severely burned

B)3 weeks after spinal cord injury at the level of C5

C)Acute renal failure

D)Chronic renal failure

E)Right middle cerebral artery thromboembolic stroke and


significant left-sided hemiplegia 2 weeks ago
• Correct Answer: B
• Rationale
Several conditions have been found to markedly
exaggerate the release of potassium associated with
succinylcholine administration. Burns and neurological
injury are well described as causes of this phenomenon.
Other conditions resulting in succinylcholine-induced
hyperkalemia include direct muscle trauma, denervation
phenomena (upper motor neuron lesions such as stroke
or cord section; lower motor neuron lesions such as
Guillain-Barré syndrome, motor nerve section, ventral
horn disorders), intra-abdominal infections, disuse
atrophy, and pharmacologic denervation by
nondepolarizing neuromuscular blocking drugs.
In acute spinal cord injury and stroke, administration of
succinylcholine is not problematic because the period
during which severe hyperkalemia may occur after
succinylcholine administration begins approximately 3
weeks after the injury. In processes that are progressive,
such as neuropathies or intra-abdominal infection and
sepsis, prolonged paralysis with nondepolarizing muscle
relaxants seems to put the patient at risk of hyperkalemia
after 7 days.

The risk of severe hyperkalemia begins 3 days after severe


burns. In peripheral nerve injury, animal studies and
experience suggest that the period of increased
susceptibility begins 4 days after the injury. The magnitude
of the hyperkalemia correlates with the severity and
extent of burn or neurological deficit.
The mechanism by which succinylcholine causes
hyperkalemia is through up-regulation of acetylcholine
receptors and destabilization of injured muscle
membrane. Up-regulation of skeletal muscle nicotinic
acetylcholine receptors is manifestated by increased
numbers of altered receptors at and around the end plate,
with possible extension of receptors across the entire
muscle membrane. The altered acetylcholine receptor is
an immature isoform that has a prolonged open channel
time, which may exaggerate the potassium efflux with
depolarization. Burn injury of a single limb (8%-9% body
surface area) has been reported to cause lethal
hyperkalemia.
Succinylcholine-induced rhabdomyolysis from damaged
muscle cells can cause a rapid release of intracellular
potassium. This mechanism may be more important than
up-regulation of acetylcholine receptors in burn patients
with hyperkalemia after administration of succinylcholine.
Typically patients that do not have any predisposition
for hyperkalemia will have an increase of 0.5 mEq/L with
the use of succinylcholine. In the high-risk patients
described above, the serum potassium concentration
can go up more than 4 mEq/L.

In burn patients, the exaggerated succinylcholine


response will persist until there is healing and weight
gain, generally 2 months after the burn. In patients with
permanent upper motor neuron injury after stroke or
spinal cord injury, the exaggerated succinylcholine
response may be prolonged. The use of succinylcholine
should be avoided until normalization of acetylcholine
receptors that is demonstrated by the loss of resistance
to nondepolarizing muscle relaxants.
• Question 103:
• The action of nondepolarizing neuromuscular blocking
agents is greatly prolonged by:

A)Thermal injury >40% body surface area

B)Myasthenia gravis

C)Myotonic dystrophy

D)Critical illness polyneuropathy

E)Sick euthyrhoid syndrome


• Correct Answer: B
• Rationale
Burns are associated with proliferation of extrajunctional
nicotinic receptors. This results in resistance to the effect
of nondepolarizing muscle relaxants and relatively short
duration of neuromuscular blockade. Myasthenia gravis is
associated with decreased release of acetylcholine at the
neuromuscular junction and makes it sensitive to the
effects of nondepolarizing neuromuscular blocking agents
(NMBAs), a pharmacodynamic response. Myotonic
dystrophy and critical illness polyneuropathy do not affect
the duration of nondepolarizing NMBA action. Calcium
channel blockers, hypokalemia, and hypermagnesemia can
also prolong the duration of nondepolarizing NMBA
action. The sick euthyroid syndrome does not affect
nondepolarizing NMBA agent duration of action.
• Question 104:
• A 65-year-old man is admitted to the ICU following a
pancreaticoduodenectomy. An epidural catheter was
placed preoperatively for postoperative analgesia. Upon
admission, his drug regimen includes a low-molecular-
weight heparin administered every 12 hours for
prophylaxis of venous thromboembolism. The epidural
was inadequate for his pain control, and he was placed
on patient-controlled analgesia.
How long after the last dose of low-molecular-weight
heparin is given is it safe to remove the epidural
catheter?

A)Immediately

B)4 hours later

C)12 hours later

D)24 hours later

E)36 hours later


• Correct Answer: C
• Rationale
Patients who have received low-molecular-weight heparin
(LMWH) are assumed to have altered coagulation. Twelve
hours following injection, 50% of the peak anti-Xa activity
will be present. In patients treated with LMWH for
pharmacologic prophylaxis of venous thrmoboembolism,
the risk of causing an epidural hematoma during either
the placement or removal of an epidural catheter is high.
Therefore, for patients administered prophylactic LMWH
doses every 12 hours, the catheter can be removed 10-12
hours after the anticoagulant is given. The LMWH can be
restarted 2 hours after catheter removal. Patients on
therapeutic anticoagulation with LMWH need to wait 24
hours after their last dose but generally are not good
candidates for epidural catheter placement in the first
place; therefore, data on absolute safety in this patient
population are sparse.
• Question 105:
• A patient is admitted to the ICU with a 2-week history
of intractable diarrhea. To differentiate secretory from
osmotic diarrhea which of the following stool
electrolytes should be obtained?

A)Sodium and calcium

B)Potassium and calcium

C)Sodium and potassium

D)Potassium and glucose


• Correct Answer: C
• Rationale
Diarrhea is a very common problem in the ICU resulting
in often unnecessary laboratory and microbiology
evaluations. Stool osmolal gap is one of the methods to
differentiate between secretory and osmotic diarrhea.
Low stool osmolal gap implies secretory diarrhea, while
a high stool osmolal gap implies osmotic diarrhea.

The formula is:

Stool Osmotic Gap =


Stool Osmolality – [2 × (Stool Sodium + Stool Potassium)]
Normal values are:
Stool osmolality = 290 mOsm/kg
Sodium = 30 mEq/L
Potassium = 75 mEq/L

Normal stool osmolal gap is less than 50 mEq/L.


Diarrhea associated with a stool osmolal gap greater
than 100 mOsm/kg usually implies osmotic diarrhea, a
common entity with enteral feeding. Only option C
includes the required elements to calculate the osmolal
gap.
• Question 106:
• A 65-year-old man has right-sided chest and upper abdominal
pain. His supine chest radiograph is shown in the Figure.
Ultrasonography is not available. Of the following
choices, which of the following tests or procedures
would be most useful at this time?

A)Right-sided ECG and troponin I assay

B)CT of chest with oral and IV contrast

C)Initiation of bilateral positive airway pressure by face


mask

D)Insertion of right thoracostomy tube


• Correct Answer: D
• Rationale
This chest radiograph demonstrates a deep sulcus sign
in the right costophrenic angle. This is one of the signs
of a pneumothorax on supine chest radiographs. Supine
and semirecumbent films have poor sensitivity for
detecting pneumothoraces. We are trained to recognize
what is called a visceral pleural line as an indicator of
pneumothorax. The visceral pleural line is the thin white
line (the visceral pleura) that with a pneumothorax is
pulled away from the chest wall, with lung parenchyma
(radiolucency with small lines) on the inside, and air
(radiolucency without small white lines) on the outside.
The lack of sensitivity of supine films is particularly
problematic, as there is a high progression to tension
pneumothorax, probably because of the association of
supine chest radiographs with mechanical ventilation.
There are, however, some subtle radiographic clues to
the presence of pneumothorax on supine chest
radiographs. A deep lateral costophrenic angle may be
the only sign of pneumothorax in over 50% of supine
position radiographs. This is because the most anterior
portion of the pleura is at the base. On occasions, the
only evidence of a pneumothorax is a generalized
hyperlucency over the base of the lung.
When the question of pneumothorax is raised, either due
to deep sulcus sign or basilar hyperlucency, there are 2
options. One can use a lateral decubitus film, with the lung
of concern up, to note presence or absence of air outside
the pleural space in the nondependent area of the lung.
Or one can sit the patient as upright as possible for
another film. In addition to basilar hyperlucency and deep
sulcus sign, supine radiographic findings of pneumothorax
include the depression of the diaphragm, a sharp
mediastinal or diaphragmatic contour, a double diaphragm
sign, a distinct cardiac apex, and apical pericardial fat tags
(lobulated/rounded).

A chest CT with oral and IV contrast is unnecessary as the


current radiograph supports expeditious treatment with
pleural decompression.
• Question 107:
• An 18-year-old, African American man is brought to the
emergency department by emergency medical services
after friends find him minimally responsive on the floor
of his apartment. On arrival, his airway is patent, his
respirations are rapid and deep, and his breath has a
fruity odor. He is hemodynamically stable but somewhat
tachycardic with weak peripheral pulses. He is afebrile.
His initial chemistry results are as follows: sodium, 134
mEq/L; potassium, 5.6 mEq/L; chloride, 94 mEq/L;
bicarbonate, 10 mEq/L; and glucose, 890 mg/dL. His
initial arterial blood gas measurements while receiving
room air are as follows: pH, 7.25; PaCO2, 24 mm Hg;
PaO2, 87 mm Hg; and bicarbonate, 10 mEq/L.
• Which of the following acid-base disturbances are present
in this patient?

A)High-anion-gap metabolic acidosis and non-gap


metabolic acidosis
B)Normal-gap metabolic acidosis and compensated
respiratory alkalosis
C)High-anion-gap metabolic acidosis and uncompensated
respiratory alkalosis
D)High-anion-gap metabolic acidosis, non-gap metabolic
acidosis, respiratory alkalosis
E)High-anion-gap metabolic acidosis, metabolic alkalosis,
and respiratory alkalosis
• Correct Answer: D
• Rationale
The patient has a metabolic acidosis as defined by a pH
less than 7.4 and a compensatory respiratory alkalosis
with PaCO2 less than 40 mm Hg. The calculated anion
gap is 30 [Sodium – (Chloride + Bicarbonate)] and the
delta gap is 4 [(Calculated Anion Gap – Normal Anion
Gap) – (Normal Bicarbonate – Measured Bicarbonate)].
The elevated delta gap reveals that an additional
disorder, non-anion gap metabolic acidosis, is present.
Therefore, this patient has a tertiary acid-base disorder.
There is no evidence of a metabolic alkalosis. Thus, only
option D has the correct acid-base disorders.
• Question 108:
• A 25-year-old woman who is 30 weeks pregnant
presents with a severe exacerbation of asthma that
began 3 days ago. Her BP is 100/60 mm Hg, HR is
120/min, RR is 36/min, and temperature is 36.9°C
(98.4°F). She is noted to have inspiratory and expiratory
wheezing with use of accessory muscles of respiration.
She is nauseated and has been frequently vomiting.
Arterial blood gas studies while receiving oxygen, 4
L/min via nasal cannula demonstrate a pH of 7.38,
PaCO2 of 45 mm Hg, and PaO2 of 68 mm Hg.
Which of the following is the most appropriate
intervention at this time?

A)Nebulized albuterol using high-flow oxygen

B)Nebulized albuterol and IV corticosteroids

C)Subcutaneous terbutaline and nebulized albuterol

D)Intubation followed by nebulized albuterol

E)Noninvasive ventilation and nebulized albuterol


• Correct Answer: D
• Rationale
This patient has severe, stage 4 asthma with carbon
dioxide retention. The normal PaCO2 in a pregnant woman
is 30-32 mm Hg; therefore, a PaCO2 of 45 mm Hg
represents significant hypercapnia due to airway
obstruction. In addition, the patient has impaired
oxygenation that needs rapid improvement to ensure
oxygen delivery to the fetus. The best option for this
patient is intubation followed by aggressive treatment of
the bronchospasm and airway inflammation with a
nebulized alpha2-agonist such as albuterol and the
subsequent IV administration of corticosteroids. Both
agents are safe to administer in pregnancy. Systemic beta-
agonists have greater side effects, with no added benefit
compared with inhaled agents. Noninvasive ventilation is
contraindicated because of nausea and vomiting.
• Question 109:
• Noninvasive ventilation is ineffective and potentially
harmful for the treatment of acute respiratory failure in
which of the following clinical settings?
A)Acute asthma exacerbation

B)24 hours after pneumonectomy

C)Community-acquired pneumonia

D)Postextubation respiratory failure

E)New-onset obstructive sleep apnea


• Correct Answer: D
• Rationale
Noninvasive ventilation (NIV) is well described for
treatment of acute cardiogenic pulmonary edema and
exacerbations of chronic obstructive pulmonary disease as
well as the management of obstructive sleep apnea. More
recently, evidence has shown possible benefit for patients
with acute respiratory failure secondary to asthma
exacerbations, community-acquired pneumonia, and
postsurgical respiratory failure. Efficacy and safety of NIV
has even been demonstrated for patients who recently
underwent open lung resection. Postextubation
management must be divided into prevention of acute
respiratory failure and treatment of respiratory failure.
Evidence supports NIV for the prevention of
postextubation respiratory failure, but studies have
demonstrated no efficacy and possible increased mortality
with the use of NIV for the treatment of postextubation
acute respiratory failure.
• Question 110:
• A 59-year-old man is brought to the emergency
department with sudden severe chest pain and
dyspnea. Oxygen saturation is 90% by pulse oximeter
with high-flow supplemental oxygen delivered by
nonrebreather face mask. RR is 35/min and HR is
125/min. A contrast-enhanced thoracic CT is shown in
the Figure.
Which of the following diagnoses can be made from the
CT?

A)Chronic pericardial effusion

B)Saddle pulmonary embolism

C)Ascending thoracic aorta dissection

D)Descending thoracic aorta dissection

E)Ascending aortic aneurysm


• Correct Answer: B
• Rationale
This CT shows a large proximal (ie, saddle pul o a
embolism (see arrows in Figure below). The heart is not
imaged, rendering answer A incorrect. The ascending and
descending aorta appear normal in this slice of the CT,
rendering answers C, D, and E also incorrect.

The risk of mortality in massive pulmonary embolism


(MPE) is directly related to the risk of a hemodynamic
collapse from an acute right heart failure. Although the
degree of hypoxemia (as estimated by PaO2 to FIO2 ratio
or A-aO2gradient) can be severe, hypoxic respiratory
failure is rarely the cause of death. The pathophysiology of
shock in MPE has been described in detail elsewhere, and
can be briefly summarized as follows.
MPE increases the right ventricular (RV) outflow
impedance. Because the RV cannot acutely generate mean
pulmonary artery pressures greater than 40 mm Hg, the
RV systolic function is impaired. This decreases left-sided
filling pressures (left ventricular preload) and diminishes
cardiac output. Shock occurs when catecholamine-induced
vasoconstriction cannot maintain the mean arterial
pressure.
The presence of hypotension would indicate a
hemodynamically significant pulmonary embolism. Not
only would this indicate a higher mortality rate, but there
is also a consensus that hypotension is the main indication
for thrombolytic therapy. Although the decision to use
thrombolytics based on ECG evidence is controversial in a
normotensive patient, an ECG can provide valuable
information.The presence of a right ventricular dysplasia
(RVD) would indicate a hemodynamically significant
thromboembolism and would confer a much higher acute
mortality risk.
This information could help to risk-stratify a patient in
terms of mortality. The absence of RVD would indicate a
negligible acute mortality risk (unless there are
recurrent thromboembolic events). Biomarkers such as
troponin and brain natriuretic peptide (BNP) have been
used to assess degree of right heart strain as a result of
the pulmonary embolisms. Patients with prior
cardiopulmonary disease manifest a greater degree of
hemodynamic sequelae for a relatively smaller clot
burden. However, in patients without a history of prior
cardiopulmonary disease, there is a consistent
relationship between the degree of embolic obstruction
and the propensity to develop shock. A prior history of
cardiopulmonary disease would therefore confer a
greater mortality risk for a significant thromboembolic
event of any size.
• Question 111:
• A patient has been mechanically ventilated in the ICU
for 5 days after an emergency subtotal colectomy for
ischemic colitis. He has developed slowly progressive
hypoxemia on the following ventilator settings: assist-
control; rate, 12/min; tidal volume, 750 mL; FIO2, 100%;
positive end-expiratory pressure (PEEP), 5 cm H2O; flow
rate, 60 L/min on a decelerating waveform; and peak
airway pressure, 31 cm H2O. Ideal body weight is 78 kg
(173 lbs), with actual weight of 92 kg (204 lbs). His last
arterial blood gas results are pH of 7.34, PaCO2 of 38
mm Hg; PaO2 of 54 mm Hg, and SaO2 of 86%. CT
angiography to rule out pulmonary embolus is obtained
and shown in the Figure.
Based on the CT findings, the best next step to improve
the patie t’s o ge atio is to:

A)Diurese using furosemide

B)Change PEEP to 12 cm H2O

C)Decrease tidal volume and increase rate

D)Therapeutically anticoagulate

E)Perform toilet bronchoscopy


• Correct Answer: B
• Rationale
The patient has both hypoxemia and
mild hypercarbia with a concomitant metabolic
alkalosis. The CT demonstrates a large portion of the
posterior aspect of one lung with collapse; contrast-
enhanced vessels are identified going into the
collapsed region leading to perfusion of unventilated
regions. Correcting the collapse would be a rational
first step to improving oxygenation and carbon
dioxide clearance; the assist-control, volume-cycled
ventilation settings have not been optimized. While
the settings do not conform to Acute Respiratory
Distress Syndrome Network (ARDSNet) settings, his
peak airway pressure is less than 35 cm H2O,
indicating that his plateau pressure will be lower
than 30 cm H2O.
Therefore, there is no need to decrease his tidal volume
and increase his rate to maintain minute ventilation at the
current setting. Collapsed segments are often
accompanied by thick secretions, so diuresis is not ideal
a d a i ease the patie t’s isk fo radiocontrast
nephropathy in certain settings. Using a decelerating
waveform is more ideal as a square waveform would
decrease the inspiratory time and may increase the
proportion of collapsed lung. Therapeutic anticoagulation
is not indicated as there is no pulmonary embolus
identified on the CT angiogram. Toilet bronchoscopy is a
term that indicates therapeutic aspiraton of the
tracheobronchial tree to remove debris or secretions.
While this may be appropriate, the ventilator settings
should be first optimized as that intervention bears less
risk than does bronchoscopy. Thus, increasing the positive
end-expiratory pressure to 12 cm H2O is the best initial
intervention.
• Question 112:
• A 42-year-old man was in a motor vehicle collision 2
months ago and sustained right-sided rib fractures 2-10
associated with a right hemothorax that was managed
with a chest tube. He was left with a residual
hemothorax that was not evacuated. He presents to the
emergency department with fever, chills, a productive
cough, and leukocytosis. He is febrile (39.1°C [102.4°F])
and tachycardic (108/min) but not hypotensive (BP of
102/68 mm Hg) and has SaO2 of 92% on room air.
Radiography demonstrates opacification of the lower
half of the right lung field. CT documents the retained
hemothorax with pulmonary parenchymal compression
and gas bubbles within the hemothorax.
The best course of management is:

A)IV antibiotics, IV fluid resuscitation, and planned


thoracotomy for decortication
B)Insertion of a 36-Fr chest tube in the emergency
department, face mask continuous positive airway
pressure, and IV antibiotics
C)Intubation, bronchoscopy, insertion of a 36-Fr chest
tube and IV antibiotics
D)Interventional radiology–guided percutaneous sampling
of the hematoma for organism recovery
E)IV antibiotics, IV fluid resuscitation, and blood and
induced sputum cultures
• Correct Answer: A
• Rationale
This patient has a retained hemothorax that has become
infected leading to empyema. It is likely that his
innoculum is derived from the insertion of his chest
tube that resided in an incompletely evacuated space.
Therefore, had he had a timely evacuation of his
retained hemothorax, it is likely that he would not have
developed an empyema 3 months postinjury.
Alternatively, with multiple rib fractures, he is at high
risk for postinjury pneumonia with direct extension to
his retained hematoma. In either case, the patient
needs antibiotics, IV fluid resuscitation, and drainage of
his infected pleural space.
A chest tube alone will not address the consolidated
and contracted clot that surrounds his right lower lobe,
may be difficult to insert due to adhesion formation,
and may result in an iatrogenic pulmonary injury
including air leak and hemorrhage. Thus, immediate
chest tube insertion is incorrect. While this condition
may be managed with video-assisted thoracic surgery
(VATS), the management of the parenchymal peel is
more difficult in the setting of infection as opposed to a
simple retained hemothorax. Thus, the majority of
empyemas after injury are safely managed using an
open approach, but some patients may be satisfactorily
managed using VATS.
There is no role for intubation and bronchoscopy in this
setting with an acceptable SaO2, and a culpable process
that is external to the pulmonary parenchyma
(empyema a d e t i si o p essio . Toilet
bronchoscopy a e useful at the e d of the
decortication to ensure that inspissated secretions are
mobilized after the relief of the fibrothorax. Similarly,
there is no role for interventional radiology–guided
percutaneous sampling of the hematoma to direct
antibiotic therapy. Skin flora predominate and coverage
should address skin organisms, as well as nosocomial
flora as the patient had been previously hospitalized and
on antibiotic therapy.
• Question 113:
• A 55-year-old, clinically severely obese patient who
weighs 100 kg (220 lbs) is transferred to the ICU from an
outside hospital for acute respiratory distress syndrome.
The patient has been sedated with propofol and
changed from assist-control, volume-cycled ventilation
to pressure controlled ventilation to manage peak
airway pressure and hypoxemic respiratory failure. The
current mechanical ventilator settings are: pressure-
controlled ventilation; rate, 12/min; inspiratory
pressure, 25 cm H2O; inspiratory time, 1.5 seconds; FIO2,
60%; positive end-expiratory pressure, 10 cm H2O.
The presence of alveolar overdistension will be best
assessed by:

A)Flow-time trace

B)Pressure-time trace

C)Volume-time loop

D)Pressure-volume loop

E)Area-under-the-flow trace
• Correct Answer: D
• Rationale
Alveolar overdistension may be most readily detected
using the pressure-volume loop (also known as the
hysteresis curve). An increase in pressure that does not
result in a corresponding change in volume creates the
i d’s eak phe o e o alo g the e t e e ight a d
portion of the inspiratory limb of the tracing. The flow-
time trace is best used to assess for auto–positive end-
expiratory pressure. The pressure-time trace is best to
assess mean airway pressure and the area under the curve
that corresponds with mean airway pressure. The volume-
time loop may be best used to look for leaks within the
system in those being mechanically ventilated. The area-
under-the-flow-time trace is generally not routinely
calculated in clinical settings.
• Question 114:
• Which of the following best characterizes the correct
method to determine plateau pressure in a patient
spontaneously breathing on volume-controlled
mechanical ventilation?
A)Peak pressure minus positive end-expiratory pressure

B)Peak pressure minus mean positive airway pressure

C)Pressure measured after pause at end-inspiration

D)Pressure measured after pause at end-expiration

E)Cannot be determined with spontaneous breathing


• Correct Answer: C
• Rationale
Plateau pressure is the pressure measured after a 0.5-
second pause at the end of inspiration. This pressure is
often utilized in mechanical ventilation protocols to
prevent worsening morbidity and mortality in patients
with acute lung injury and acute respiratory distress
syndrome. Peak pressure minus positive end-expiratory
pressure (option A) is also known as the driving pressure
and is the pressure that opens and ventilates the lung.
Peak pressure minus mean positive airway pressure
(option B) is not a commonly calculated value. Pressure
measured after pause at end-expiration (option D) is the
positive end-expiratory pressure. Plateau pressures can be
measured regardless of spontaneous ventilation, so option
E is incorrect.
• Question 115:
• A 25-year-old man with no significant past medical history
comes to the emergency department because of left-sided
pleuritic chest pain. He reports the onset of chest
discomfort yesterday accompanied by mild dyspnea. He
denies any fever, cough, night sweats, or weight loss. He
reports no risk factors for HIV. He has never smoked
cigarettes. On presentation, he is in mild distress. His weight
is 62.5 kg (139 lbs) and his height is 188 cm (6 ft 2 in). His
body mass index is 17.7 kg/m2. His respirations are 26/min.
His trachea is midline and no crepitus is noted in the
supraclavicular and cervical region. Auscultation reveals
diminished air movement throughout the left lung. He has
no notable pulmonary hypertrophic osteoarthropathy,
cyanosis, or peripheral edema. Posteroanterior chest
radiograph shows a 6.5-cm pneumothorax on the left (from
the lung apex to the pleural cupula in the midclavicular
line). No mediastinal shift or diaphragmatic depression is
noted and the lung fields appear normal. His hemodynamic
parameters are within normal ranges.
Which of the following is the most appropriate initial
management step?

A)Video-assisted thoracoscopic biopsy and pleurodesis

B)Noncontrast CT of the chest

C)100% oxygen and repeat chest radiography in 24


hours

D)Insertion of an over-the-needle catheter and manual


aspiration

E)Tube thoracostomy with instillation of a sclerosing


agent
• Correct Answer: D
• Rationale
Option A is incorrect. While video-assisted thoracoscopic
biopsy and pleurodesis are a potential management
option, they do not constitute initial therapy.

Option B is incorrect. The patient has an acute


pneumothorax. Chest CT should not be the initial
management option for this patient. Chest CT has a
relatively limited role in the management of primary
spontaneous pneumothorax.

Option C is incorrect. While supplemental oxygen is


beneficial at helping resorption of the pneumothorax, a
pneumothorax greater than 2 cm, or a pneumothorax of
any size with symptoms should be evacuated.
Option D is correct. The success rate of aspiration of a
primary spontaneous pneumothorax is high,
comparable to tube thoracostomy. The actual morbidity
associated with this procedure and the duration of
hospital stay are less than in patients treated with tube
thoracostomy.

Option E is incorrect. While a tube thoracostomy is a


potentially reasonable option, a sclerosing agent might
be reserved for a recurrent episode of pneumothorax.
• Question 116:
• Which of the following are risk factors that predispose
patients to pulmonary embolism?
A)Nephrotic syndrome

B)Weight loss

C)Arterial catheters

D)Antidepressant agents
• Correct Answer: A
• Rationale
The important risk factors for venous thrombosis, and
thus pulmonary embolism (PE), are central venous
catheter presence, especially in younger children, and
certain medical conditions. These underlying medical
diagnoses include congenital heart disease, protein-
losing states like nephrotic syndrome or inflammatory
bowel disease, morbid obesity, or prolonged
immobilization with a surgical procedure in the
preceding 2 weeks. Nephrotic syndrome is a commonly
unrecognized predisposing factor to the development of
PE.
Nephrotic syndrome is a high-risk factor because of loss of
antithrombin III and the impaired red blood cell
deformability. Decreased profibrinolysin; increased levels
of factors V, VII, VIII, von Willebrand factor, and lipoprotein
a; increasing platelet counts and aggregation; and
attachment and release of platelet alpha-granules with the
use of diuretics and steroids may be the other reasons.
Reported incidence of thrombi developing in these
patients is 5% to 10% while on steroids and diuretics. A
low threshold to order investigations for PE should be
maintained in this subset of patients. PE is not associated
with weight loss or arterial catheters. PE associated with
the use of oral contraceptives is thought to occur as a
result of the resistance of the third generation birth
control norgestimate to endogenous anticoagulant-
activated protein C.
• Question 117:
• A 24-year-old man, who is 178 cm (5 feet 10 inches)
tall and weighs 98 kg (218 lbs) is admitted to the ICU
after a motor vehicle collision in which he sustains
multiple rib fractures and a pelvic fracture. He is
hypotensive on admission and undergoes
angioembolization of his pudendal arteries. He is
resuscitated with packed red blood cells, 6 U; fresh
frozen plasma, 6 U; platelets, 5 U; and lactated
Ringer solution, 4,500 mL. On postinjury day 1 he
remains intubated and mechanically ventilated;
however, his arterial blood gas results are pH of
7.47; PaCO2 of 32 mm Hg, and PaO2 of 68 mm Hg.
His ventilator settings are assist-control at 14/min,
tidal volume of 700 mL, FIO2 of 0.6, positive end-
expiratory pressure (PEEP) of 8 cm H2O, and plateau
pressure is 35 cm H2O.
Which of the following steps is most important in
implementing protective lung ventilation?

A)Increase FIO2 until PaO2 is >100 mm Hg.

B)Titrate PEEP until PaO2 is >100 mm Hg.

C)Decrease tidal volume to 450 mL.

D)Reduce tidal volume to 600 mL.

E)Change to pressure control ventilation.


• Correct Answer: C
• Rationale
The goal in protective lung ventilation is to keep the plateau
pressure below 30 cm H2O.1 Using the strategy
recommended by the Acute Respiratory Distress Syndrome
Network (ARDSNet) protocol, this is done by decreasing the
tidal volume to 4-6 mL/kg ideal body weight (IBW). This
patie t’s IBW is kg Hamwi equation). He is being
ventilated with 9.6 mL/kg IBW. His tidal volume should be
reduced to 450 mL, which is 6.2 mL/kg IBW. If this does not
result in a plateau pressure less than 30 cm H2O, tidal
volume should be reduced further. At the same time, the
rate should be increased to maintain the minute ventilation.
Increasing positive end-expiratory pressure will only serve
to increase the airway pressures. While one can change to
pressure controlled ventilation, the current patient is
overventilated with regard to both tidal volume and minute
ventilation and tidal volume reduction would be the first
most appropriate maneuver.
• Question 118:
• A 52-year-old, homeless man with a history of alcohol
dependence and polysubstance abuse is brought to the
emergency department for 3 days of increasing shortness
of breath, fevers, shaking chills, and right-sided chest pain.
The patient is an active smoker. His chest radiograph
shows a right lower lobe opacity. He is admitted to the
hospital and treated with supplemental oxygen, IV fluids,
ampicillin/sulbactam, thiamin, folate, and multivitamin.
Several hours later, during the day, the patient becomes
more breathless despite increasing requirement. He is
intubated and placed on mechanical ventilation in the
assist-control mode setting. The initial ventilator settings
are FIO2 of 1.0, set rate of 18/min, tidal volume of 10
mL/kg of predicted ideal body weight, positive end-
expiratory pressure (PEEP) of 7.5 cm H2O, and a constant
inspiratory flow rate of 60 L/min.
On these settings, peak airway pressure is 45 cm H2O and
plateau pressure is 38 cm H2O. No auto-PEEP is noted. His
arterial blood PaO2 is 70 mm Hg, PaCO2 is 46 mm Hg, and
pH is 7.32. Repeat chest radiograph shows diffuse bilateral
opacities, no fluid collections or pneumothorax, and a
normal cardiac silhouette. Which of the following changes
in ventilator settings should be made first?

A)Decrease tidal volume.


B)Decrease FIO2.
C)Decrease inspiratory flow rate.
D)Decrease PEEP.
E)Change to pressure control ventilation.
• Correct Answer: A
• Rationale
Option A is correct. Lower tidal volumes have been shown
to result in better mortality outcomes compared to higher
tidal volumes.
Option B is incorrect. While the level of FIO2 can pose a
risk of oxygen toxicity, a lower tidal volume strategy of
ventilation has been demonstrated to have a substantial
impact on mortality and should be corrected first.
Option C is incorrect. A decrease in inspiratory flow rate
would likely decrease peak airway pressure and have a
modest impact on the mean airway pressure by
prolonging the inspiratory time. However, it would not be
e pe ted to ha e a i pa t o this patie t’s ele ated
plateau pressure and risk of ventilator-induced lung injury.
Option D is incorrect. Ventilation with lower positive
end-expiratory pressure (PEEP) strategies in acute
respiratory distress syndrome (ARDS) has not been
demonstrated to improve mortality in randomized,
controlled trials. In fact, some studies suggest that
higher PEEP levels are associated with improved survival
among the subgroup of patients with ARDS.

Option E is incorrect. While small studies have


suggested a decrease in work of breathing in patients
treated with pressure control ventilation, there have
been no studies demonstrating a survival advantage in
those ventilated using pressure control ventilation.
• Question 119:
• An 80-year-old man with a history of advanced
Alzheimer disease, hearing impairment, and chronic
obstructive pulmonary disease is admitted to the ICU
with septic shock due to aspiration pneumonia.
• Which of the following is the best strategy to prevent
pressure ulcers in this patient?

A)Use of rectal and urinary catheters to minimize the


consequences of incontinence, such as soiling and
moisture buildup in vulnerable areas
B)Implementation of a prevention and treatment bundle
that includes risk assessment, skin checks, repositioning,
and other adjunctive devices
C)Early and proactive use of specialized mattresses in light
of the patie t’s high-risk category, given advanced age and
sensory impairment
D Opti izi g the patie t’s ut itio al status ith total
parenteral nutrition if caloric needs are not met within the
first few days of ICU admission
• Correct Answer: B
• Rationale
Critically ill patients are at particular risk for developing
decubitus ulcers. Risks that have been identified include
immobility, incontinence, coexisting nutritional deficits,
advanced age, and ICU length of stay. Additionally,
presence of sepsis (and particularly shock needing
vasopressor/inotropic therapy) as in this patient, is a
notable risk factor. Concurrent administration of sedatives
and analgesics further impairs patient mobility as well as
ability to notice increased tissue pressure. Preexisting
sensory impairments (hearing and visual deficits) further
exacerbate the problem. Taken together, this elderly
patient has multiple characteristics that place him in a
high-risk category.
Although incontinence is a mitigating factor, particularly
once a decubitus ulcer develops, prophylactic use of
diverting devices such as rectal and urinary catheters
has never been shown in itself to prevent ulcer
development. Therefore, option A is incorrect.

One study explored the use of a specialized pressure-


reducing mattress at the first sign of nonblanchable
erythema or expected immobility of greater than 72
hours and showed that such preventive transfers were
the strongest predictor of ulcer-free days. However, this
was done in the context of implementing a
comprehensive ulcer prevention and treatment
protocol, and use of such devices alone is inadequate.
Other studies have similarly shown that an organized,
interdisciplinary program that focuses on education, risk
stratification, consistent skin assessment, prevention (for
example, via repositioning maneuvers, utilization of
pressure-reducing mattresses and kinetic beds) and
appropriate treatment can effectively reduce ulcer
prevalence. Therefore, option B is the correct answer, and
C is incorrect.
Finally, the number of days without adequate nutrition is
an important risk factor for ulcer development (and a key
factor for healing once ulcers are present). Nutritional
assessment and support is an important adjunct to any
ulcer prevention and treatment program. However,
aggressive and early use of total parenteral nutrition has
not been shown efficacious or rigorously examined as an
intervention to mitigate decubitus ulcer development.
Therefore, option D is incorrect.
• Question 120:
• RIFLE criteria are designed to grade the disease process
of which of the following organs?

A)Liver

B)Lung

C)Kidney

D)Brain

E)Heart
• Correct Answer: C
• Rationale
The Acute Dialysis Quality Initiative (ADQI) Group
published a consensus definition for acute kidney injury
(AKI) and acute renal failure that is known as the RIFLE
criteria (Risk, Injury, Failure, Loss of kidney function, and
End-stage kidney disease). It has become a standard in
terms of defining decrements in renal function,
particularly in the ICU or hospital setting. Subsequent
studies have shown its utilization in predicting hospital
mortality, where mortality correlates with the maximum
RIFLE classification reached during the ICU stay. The
Acute Kidney Injury Network (AKIN) has further clarified
AKI into 3 stages based on changes in urine flow and
serum creatinine.
• Question 121:
• A previously healthy, 54-year-old, 84-kg (187-lb) man
undergoes a laparoscopic converted to open distal
pancreatectomy and splenectomy for a pancreatic tail
mass. He does well intraoperatively, has an uneventful
stay in the postanesthesia care unit, and is transferred to
the floor. On postoperative day 2 he develops fever and
undergoes a workup. He is tachycardic to 104/min, has a
normal BP of 126/84 mm Hg, RR of 26/min (nonlabored),
and room-air SaO2 of 93%. His wound has a 1-cm zone of
surrounding erythema and no drainage. His WBCs have
increased from 9,200 to 15,600/µL, blood urea nitrogen
level has increased from 12 to 26 mg/dL, and creatinine
has increased from 1.2 to 1.6 mg/dL with an associated
urine output over the past two 12-hour shifts of 510 mL
and then 186 mL. Fo the hou s of toda ’s shift, he has
made 42 mL of urine.
• Which of the following best describes his acute kidney
injury?

A)Acute Kidney Injury Network (AKIN) stage 2

B)AKIN stage 3

C)Septic acute kidney injury

D RIFLE F

E)Kidney Disease: Improving Global Outcomes (KDIGO)


class 2a
• Correct Answer: A
• Rationale
Acute kidney injury has been explored by several different
organizations including the Acute Dialysis Quality Initiative
(ADQI), the Acute Kidney Injury Network (AKIN), and
Kidney Disease: Improving Global Outcomes (KDIGO). Each
organization has built upon the RIFLE criteria (Risk, Injury,
Failure, Loss, and End-stage disease) articulated by ADQI
that utilized 3 different urine ouput and time metrics,
serum creatinine (Scr), or glomerular filtration rate (GFR)
criteria. AKIN has further refined the analysis of injury to
having 3 stages using urine output criteria similar to RIFLE
or serum creatinine criteria; no GFR criteria are present.
KDIGO further simplified the definition of AKI to 3 criteria:
increased Scr greater than 0.3 mg/dL within 48 hours;
increased Scr greater than 1.5 times baseline that has
occurred or is presumed to have occurred within 7 days;
and urine output less than 0.5 mg/kg/h for 6 hours.
Furthermore, KDIGO refined the AKIN criteria including
the addition of estimated GFR for those younger than 18
years (<35 mL/min/1.73 m2 body surface area).

This patient has a rise in Scr that is more than 0.3 mg/dL
(stage 1), but his increase in Scr is not greater than
200% but less than 300% of his baseline (ie, not stage
2). However, he has oliguria (urine output <0.5 mL/kg/h)
present for more than 12 hours (stage 2). Using the
worst criterion—the urine output—he fits into AKIN
stage 2. He does not meet the Scr nor urine output nor
receipt of renal replacement therapy criteria for AKIN
stage 3.
Septic acute kidney injury identifies the proximate
cause of the AKI, and it is not clear that this patient has
this diagnosis since he is not febrile, his wound has no
drainage, and he has a normal blood pressure. Similarly,
the patient does not meet the urine output, Scr, nor GFR
ite ia fo RIFLE F lassifi atio . The e is o KDIGO
Class 2a designation. However, since the KDIGO and
AKIN urine output criteria are similar, this patient would
satisfy the KDIGO class 2 criteria.
• Question 122:
• After a 70% body surface area burn, your patient has
persistent acute renal failure and is judged to be
appropriate for continuous renal replacement therapy.
Continuous renal replacement therapy is selected as the
patient also has ventilator-associated pneumonia,
relative adrenal insufficiency, and requires
norepinephrine at 0.08 µg/kg/min and vasopressin at
0.04 U/h. He is being treated for methicillin-
resistant Staphylococcus aureus with IV vancomycin and
for Pseudomonas aeruginosa with
piperacillin/tazobactam.
The antibiotics should be dosed for renal failure being
managed with continuous renal replacement therapy
based on:

A)Daily estimated glomerular filtration rate values

B)Daily random levels

C)Assumption of a glomerular filtration rate of 25


mL/min

D)Weekly 24-hour creatinine clearance

E)24-hour urinary nitrogen values


• Correct Answer: C
• Rationale
Continuous renal replacement therapy (CRRT) provides a
stable platform from which to dose medications that
require adjustment based on renal function. CRRT
provides a glomerular filtration rate (GFR) of
approximately 25 mL/min. Additional adjustments may
need to be made when using CRRT with hemofiltration or
diafiltration, including in some circumstances an increase
in the medication dose if that medication is efficiently
cleared across the dialysis membrane. Estimated GFR is an
algorithmically derived value that provides an estimate of
GFR that is useful in identifying patients with diminished
renal function but is not useful for medication titration.
While vancomycin can be adjusted on the basis of a
random level, piperacillin/tazobactam cannot.
The initial vancomycin and piperacillin/tazobactam
doses should be provided assuming a creatinine
clearance of 25 mL/min when using CRRT. A 24-hour
evaluation of creatinine clearance is useful for
identifying renal function before starting or after
terminating CRRT but is not useful while the patient is
receiving CRRT. Twenty-four-hour urinary nitrogen
values are used to adjust the protein aspect of a
nutritional prescription and do not influence antibiotic
dosing.
• Question 123:
• A 19-year-old female college student is brought to the
emergency department by friends who state that she
fainted several times today when attempting to stand. She
denies nausea, emesis, or diarrhea. She admits that she
at hes hat she eats. “he takes o edi atio s. I itial
resting HR is 92/min with BP of 98/52 mm Hg while in the
supine position. When her legs dangle off the edge of the
ed, she e o es light-headed. The HR a d BP a e o
112/min and 75/38 mm Hg, respectively. Her initial
chemistry results are as follows: sodium, 140 mEq/L;
potassium, 3 mEq/L; chloride, 90 mEq/L; bicarbonate, 34
mEq/L; glucose, 115 mg/dL; blood urea nitrogen, 25
mg/dL; and creatinine, 0.7 mg/dL. Her initial arterial blood
gas measurements on room air show pH of 7.48, PaCO2 of
47 mm Hg, PaO2 of 89 mm Hg, and bicarbonate level of 34
mEq/L.
Which of the following acid-base disturbances is/are
now present in this patient?

A)Normal-gap metabolic acidosis

B)Metabolic alkalosis only

C)Anion-gap metabolic acidosis and respiratory alkalosis

D)Metabolic alkalosis and compensatory respiratory


acidosis

E)Respiratory acidosis only


• Correct Answer: D
• Rationale
The patient has a hypochloremic metabolic alkalosis
probably secondary to self-induced vomiting. The
diagnosis of metabolic alkalosis is defined by a pH greater
than 7.4 and a serum bicarbonate level greater than 24
mEq/L. Partial respiratory compensation is defined by a
PaCO2 over 40 mm Hg and establishes the presence of a
respiratory acidosis. While metabolic acidosis may be
categorized on the basis of the presence or absence of an
abnormal anion gap, no such categorization exists for
metabolic alkalosis. Respiratory acidosis as an isolated
disorder can be ruled out as the pH is above 7.4 and one
would expect that the pH would be less than 7.4 if
respiratory acidosis were the only disorder (pH
reciprocally changes by 0.08 units for every 10-mm Hg
change in PaCO2).
• Question 124:
• A 52-year-old man was admitted 5 days ago with a
massive pulmonary embolus. He initially required
mechanical ventilation for hypoxemia and volume
expansion for hypotension. Since extubation the
hypoxemia has improved, but the patient still reports
shortness of breath. His oxygen saturation on 2 L/min
nasal cannula, is 98% and a repeat CT angiography of
the chest shows no new thrombus with a normally sized
right ventricle. The following laboratory values are
obtained: sodium, 139 mEq/L; potassium, 5 mEq/L;
chloride, 115 mEq/L; bicarbonate, 12 mEq/L; blood urea
nitrogen, 10 mg/dL; creatinine, 1.2 mg/dL; and glucose,
78 mg/dL. His arterial blood gas results show pH of 7.24,
PaCO2 of 28 mm Hg, and PaO2 of 90 mm Hg.
Which of the following acid-base disturbances is/are
present in this patient?

A)Normal-gap metabolic acidosis

B)Respiratory alkalosis only

C)Hyperchloremic metabolic acidosis and compensatory


respiratory alkalosis

D)High anion-gap metabolic acidosis and respiratory


alkalosis

E)Metabolic alkalosis and respiratory acidosis


• Correct Answer: C
• Rationale
The patient has a hyperchloremic metabolic acidosis
probably secondary to volume expansion with normal
saline. The diagnosis of metabolic acidosis is defined by
a pH less than 7.4 and a serum bicarbonate level less
than 24 mEq/L. The patient also has an elevated
chloride level. Partial respiratory compensation (and
symptoms of shortness of breath most likely related to a
compensatory increase in minute ventilation) are
secondary to hyperventilation as evidenced by a
PaCO2 under 40 mm Hg.
• Question 125:
• A 75-year-old man presents with confusion and anxiety.
Emergency medical services personnel report that he is
healthy except for generalized arthritis. Several over-
the-counter medications were found at the scene,
including aspirin and acetaminophen. He is tachypneic
and mildly tachycardic but otherwise hemodynamically
stable. The following laboratory values are obtained:
sodium, 140 mEq/L; potassium, 3.2 mEq/L; chloride, 105
mEq/L; total carbon dioxide, 24 mEq/L; glucose, 85
mg/dL; blood urea nitrogen, 10 mg/dL; and creatinine,
1.1 mg/dL. His initial arterial blood gas measurements
while receiving room air are as follows: pH, 7.48; PaCO2,
30 mm Hg; PaO2, 73 mm Hg; and bicarbonate, 22
mEq/L.
Which of the following acid-base disturbances is/are
present in this patient?

A)Normal-gap metabolic acidosis and metabolic


alkalosis

B)Respiratory alkalosis only

C)High-anion-gap metabolic acidosis and respiratory


alkalosis

D)Metabolic alkalosis and respiratory acidosis

E)High-anion-gap metabolic acidosis, normal-gap


metabolic acidosis, and respiratory alkalosis
• Correct Answer: B
• Rationale
The patient has a respiratory alkalosis defined by a pH
above 7.4 and a PaCO2 less than 40 mm Hg. The serum
bicarbonate level is 22 mEq/L and thus within normal
range, excluding a diagnosis of metabolic acidosis or
alkalosis. Moreover, the sodium and chloride levels are
also normal, excluding hyperchloremic metabolic
acidosis. Similarly, the blood urea nitrogen, serum
creatinine, and glucose levels are normal, effectively
excluding abnormal levels of fixed acids. Anion gap is
calculated as (Sodium + Potassium) – (Chloride +
Bicarbonate). In this case, anion gap would be (140 +
3.2) – (105 + 22) = 143.2 – 127 = 16.2.
• Question 126:
• A 76-year-old man with mild congestive heart failure
and chronic obstructive pulmonary disease is admitted
to the hospital with recurrent pneumonia. He requires
intubation for 2 days and is treated with ceftriaxone and
enalapril. Five days later, when he is awake, alert, and
ambulating, the following laboratory values are
obtained: sodium, 129 mEq/L; potassium, 3.2 mEq/L;
chloride, 81 mEq/L; bicarbonate, 38 mEq/L; glucose, 108
mg/dL; blood urea nitrogen, 21 mg/dL; and creatinine,
1.8 mg/dL. His initial arterial blood gas measurements
while receiving room air are as follows: pH, 7.51; PaCO2,
49 mm Hg; PaO2, 68 mm Hg; and bicarbonate, 38
mEq/L.
Which of the following acid-base disturbances is/are
present in this patient?

A)Normal-gap metabolic acidosis

B)Metabolic alkalosis only

C)High-anion-gap metabolic acidosis and respiratory


acidosis

D)Metabolic alkalosis and compensatory respiratory


acidosis

E)Respiratory acidosis only


• Correct Answer: D
• Rationale
The primary disorder in this patient is a metabolic
alkalosis. There is a compensatory respiratory acidosis
indicated by the PaCO2 of 49 mm Hg. The compensation
for an acid-base disorder is never complete, so the pH will
always indicate the direction of the primary disorder. In
this case, the pH is 7.51, which indicates the primary
disorder is an alkalosis. The PaCO2 is elevated indicating a
respiratory acidosis, which, because the pH is alkalotic,
must be compensatory. The presence of a metabolic
alkalosis as the primary disorder is further supported by
the presence of hypochloremia. The patient is also
hyponatremic, but the strong ion difference (SID) (Na+ –
Cl–) is still 48, larger than its normal value of ~38 (140 –
102 = 38). An SID greater than 38 indicates a metabolic
alkalosis, while one less than 38 indicates a metabolic
acidosis.
• Question 127:
• A spot urine sample for electrolytes on postoperative
day 3 after a right colon resection for adenocarcinoma
reveals the following results: sodium, 51 mEq/L;
potassium, 75 mEq/L; and chloride, 46 mEq/L. A
current chest radiograph reveals right upper lobe
infiltrate and a questionable mass. Unfortunately, no
preoperative chest radiography was performed. A room
air arterial blood gas study reveals the following: pH,
7.48; PaCO2, 49 mm Hg; PaO2, 85 mm Hg.
Which of the following is the most likely diagnosis?

A)Acute renal failure with isosthenuria

B)Posthypercapnia syndrome

C)Effect of furosemide administration

D)Ectopic corticotropin secretion

E)Mobilization of postoperative fluids


• Correct Answer: D
• Rationale
The arterial blood gas measurement is not normal
and reflects a respiratory acidosis (elevated PaCO2)
and a metabolic alkalosis (pH higher than expected
for the PaCO2). Thus, acute renal failure with
metabolic acidosis is incorrect. Post-hypercapnia
syndrome is not supported by the pH and PaCO2. In
the simplest terms, metabolic alkalosis is generated
by the loss of a hydrogen ion from a variety of
causes. However, due to the normally functioning
kid e s’ t e e dous apa it fo solute e etio
(and correction of acid-base imbalances), coupled
with the ready regeneration of protons from water
dissociation, the condition must be maintained by
another process.
Extracellular fluid volume (ECFV) depletion caused by
gastrointestinal loss and diuretics is the most common
cause and is treated with volume replacement. Since most
volume losses are accompanied by chloride loss (not
regeneratable) and the administration of chloride is
acidifying, most metabolic alkalosis is termed chloride
responsive. Cases of mineralocorticoid excess, such as
Cushing syndrome, primary hyperaldosteronism, ectopic
corticotropin production, and secondary
hyperaldosteronism from diseases such as renovascular
disease are associated with ECFV expansion, are
unassociated with chloride loss, and are therefore termed
chloride-resistant. All causes of metabolic alkalosis
typically have hypokalemia as a coexisting feature and
should result in a lower urinary potassium concentration.
Since the potassium level is excessively high relative to the
sodium and chloride, another cause must be identified.
While furosemide administration does lead to
hypochloremic metabolic alkalosis, given the history of
lung disease and a chest radiograph showing a possible
mass, an ectopic corticotropin-secreting tumor is the
most likely diagnosis. Mobilization of postoperative
fluids without furosemide administration is unlikely to
create a metabolic alkalosis.
• Question 128:
• A 42-year-old man is involved in a high-speed car crash.
He undergoes contrast CT scanning at an outside facility
and is found to have a complex left acetabulum fracture
as well as a left midshaft femur fracture; his pelvis is
wrapped and his femur is managed with a traction
splint. He is transferred to your facility for definitive
management. On arrival he is hemodynamically
appropriate, 74 kg (167 lbs) in weight, and is noted to
have a cool left foot with a right-sided ankle-brachial
index of 1.0 and a left-sided ankle-brachial index of 0.6.
CT is presented in the Figure.
• The orthopedic surgeons want a CT angiography to
evaluate for a fracture-associated arterial injury. Which of
the following represents the best method of preparation
for the radiocontrast load?

A)6% hydroxyethyl starch in balanced salt solution bolus


B)Non–dextrose containing normotonic crystalloid bolus
C)5% dextrose supplemented hypotonic maintenance fluid
D)Normotonic crystalloid bolus plus sodium bicarbonate–
based maintenance fluid
E)5% dextrose supplemented sodium bicarbonate–based
maintenance fluid
• Correct Answer: D
• Rationale
This patient will receive a second radiocontrast exposure
and is therefore at increased risk of radiocontrast
nephropathy. Risk factors include contrast infusion,
hypovolemia, diabetes, and the concomitant use of certain
medications such as metformin. The CT demonstrates a
flat inferior vena cava at the level of the renal veins
indicating significant hypovolemia. Therefore, this patient
has 2 risk factors: repeated short-time-frame exposure
and hypovolemia. Therefore, optimal preparation should
address hypovolemia and is best done with a non–
dextrose containing normotonic crystalloid fluid; starch-
based colloids do not provide free water and large
volumes will induce hyperoncoticity leading to kidney
injury.
In addition, management with a bicarbonate-based
maintenance fluid has been demonstrated to reduce the
incidence of radiocontrast nephropathy compared to
nonbicarbonated fluid supplementation. Therefore,
option D is most appropriate as it incorporates both key
elements in renal protection.
• Question 129:
• Which of the following factors is most likely to have
negative effects on coronary perfusion pressure during
cardiopulmonary resuscitation?
A)Chest compression rate of 100/min

B)Chest compression depth of 2.0 inches (51 mm)

C)Ventilation rate of 25/min after intubation

D)Not interrupting chest compressions for ventilation

E)Washout of tissue acids during plasma volume


expansion
• Correct Answer: C
• Rationale
Recent investigations have identified that the quality of
CPR in both in-hospital and out-of-hospital cardiac
arrests does not meet consensus recommendations.
These studies identified several opportunities to
improve the quality of CPR, in both in-hospital and out-
of-hospital cardiac arrest, by focusing on adherence to
the recommendations for the rate of chest
compressions, depth of chest compressions,
i i izatio of o-flo pe iods he hest
compressions are interrupted, and avoidance of
overventilation. Current consensus recommendations
advocate a chest compression rate of 100/min. The
depth of compressions is recommended to be 2 inches.
Overventilating the cardiac arrest patient has been
reported and can be deleterious by creating breath
stacking and a progressively positive intrathoracic
pressure similar to high levels of positive end-expiratory
pressure. This process will decrease venous return,
decrease left-sided filling, and decrease stroke volume,
leading to a decrease in mean arterial pressure and thus
in coronary perfusion pressure. The recommendations
are for ventilations to be provided after every 30 chest
compressions before intubation, and 8-10 ventilations
per minute after the patient is intubated.
• Question 130:
• A 38-year-old woman who is 30 weeks pregnant is
admitted to the ICU for septic shock related to
pyelonephritis. She is treated with IV antibiotics and
plasma volume expansion with subsequent clinical
improvement. On ICU day 1, she suffers an unexpected
cardiac arrest.
Which of the following interventions during resuscitation
is most appropriate?

A)Resuscitation in the supine position with elevation of


lower extremities
B)Resuscitation with the left hip elevated to improve
venous return
C)Standard Advanced Cardiac Life Support medications
with the exclusion of epinephrine due to pregnancy
D)Elevation of the right hip at a 15° to 30° angle during
resuscitation
E)Cesarean section for failure of return of spontaneous
circulation in 10 minutes
• Correct Answer: D
• Rationale
Cardiopulmonary arrest during pregnancy is treated with
the same Advanced Cardiac Life Support protocols as in
nonpregnant patients. The only difference is the optimal
recommended position for resuscitation. During
pregnancy, the gravid uterus compresses the inferior vena
cava, limiting venous return to the heart. This compression
increases with the size of the uterus and varies with the
patie t’s positio . Co p essio is a i al i the supi e
and right decubitus positions, but is minimized with the
left lateral decubitus position. Blood flow and
hemodynamics will improve with left lateral decubitus
position, but efficacy of chest compressions should also be
considered. Rees et al3 examined the efficacy of
resuscitation with the body at various angles of
inclination.
• At 0° (supine position) the efficacy of chest compressions is
maximal, whereas at 90°, chest compressions are least
effective, yet there is the least impediment of maternal venous
return. These investigators determined that an angle of 27° on
an inclined board provided the most effective position for
chest compressions with the least impact on venous return.
The current recommendation for positioning in pregnant
patients during cardiac arrest is elevation of the right hip at 15°
to 30° with a resuscitation wedge. Treating the mother takes
precedence in this situation. The decision to perform
postmortem cesarean section is a difficult one. Delivery of the
infant could remove the compromising effects of aortocaval
compression and restore venous return, while likewise
removing a potentially viable fetus from an increasingly toxic
environment. Many believe that a decision to perform a
cesarean section should be made early, within the first 5
minutes, because the optimal infant and maternal survival
occurs when the interval from arrest to delivery is less than 4
minutes.
• Question 131:
• Which of the following is expected to occur during the
cooling process in temperature-targeted management
of patients after cardiac arrest?
A)Hyperkalemia

B)Hypoglycemia

C)Hypoxemia

D)Oliguria

E)Prothrombotic state
• Correct Answer: C
• Rationale
The dissolved oxygen content of blood increases during
cooling. This a result of the increased solubility of
oxygen into the aqueous portion of blood as the
temperature decreases. Analysis of a blood sample
drawn from a hypothermic patient that is then analyzed
at 37°C (98.6°F) will have a higher amount of dissolved
oxygen than one obtained with the same inspired
oxygen content in the normothermic state.
The potassium concentration decreases due to
increased intracellular shift. Hyperglycemia occurs due
to decreased insulin production and decreased insulin
receptor binding and activity.

Hypothermia leads to coagulopathy due to decreased


platelet count, platelet function, and decreased activity
of the clotting factors. This coagulopathy will not be
apparent if the samples are warmed to 37°C when they
are analyzed.
• Question 132:
• Under the appropriate clinical conditions, which of the
following parameters provides the most accurate
predictor of fluid responsiveness in a mechanically
ventilated patient?
A)Central venous pressure

B)Pulmonary artery occlusion pressure

C)Pulse pressure variation

D)Left ventricle end-diastolic volume by


echocardiography

E)Subclavian vein oxygen saturation


• Correct Answer: C
• Rationale
Neither the central venous pressure nor pulmonary artery
occlusion pressure is useful in predicting fluid
responsiveness. Nor is a single echocardiographic
measurement of left ventricular end-diastolic volume
(LVEDV) predictive since a small ventricular cavity (eg, with
left ventricular hypertrophy or right ventricular overload)
may not respond to a volume. Similarly, a large LVEDV in a
patient with a dilated cardiomyopathy and volume
depletion may be fluid responsive. The ScvO2, although a
great parameter to estimate adequacy of global oxygen
delivery, does not predict volume responsiveness in a
broad group of critically ill patients. Pulse pressure
variation is very accurate at predicting volume
responsiveness under the appropriate clinical conditions,
which include an intubated and mechanically ventilated
patient, use of a moderate-sized tidal volume (eg, >8 mL/
kg), passive ventilation (ie, no spontaneous respiratory
efforts), and sinus rhythm.
• Question 133:
• An 80-year-old man with a long-standing history of
hypertension is involved in a head-on motor vehicle
crash with a bent steering column and wheel. He is
neurologically intact. Initially, HR is 100/min, BP is 92/70
mm Hg, and RR is 28/min. Admission laboratory results
are notable for hemoglobin level of 9.1 g/dL, and base
deficit of –9 mEq/L. Plasma volume expansion is
undertaken with lactated Ringer solution by bolus. CT of
the head, neck, chest, abdomen, and pelvis shows a
large pelvic hematoma and a contrast blush as well as a
T4 burst fracture. His HR is 140/min, BP is 70/40 mm Hg,
and RR is 26/min, with subsequent cardiopulmonary
arrest.
Which of the following is the most likely cause of his
arrest?

A)Neurogenic shock

B)Hypovolemic shock

C)Cardiogenic shock

D)Septic shock

E)Carotid dissection
• Correct Answer: B
• Rationale
Hypovolemic shock is one of the leading causes of
preventable death after traumatic injury. Pelvic
fractures, particularly in the elderly, who have limited
tissue turgor, can result in loss of more than 1 liter of
blood. In this patient the combination of tachycardia,
hypotension, base deficit, and decreased hematocrit
suggest that the patient has sustained significant blood
loss due to trauma. The shock index, defined as the
heart rate divided by systolic blood pressure, has been
shown to be a better indicator of early shock after injury
than traditional vital signs, particularly in the elderly.
While the patient may have crashed as a result of an
undisclosed source of sepsis, the pelvic hematoma and
a contrast blush suggest active hemorrhage. The
tachycardia should be interpreted as a rate-responsive
cardiac output sign due to hypovolemia rather than
evidence of cardiogenic shock despite the bent steering
wheel, as those who have a severe blunt cardiac injury
generally present with profound shock rather than
developing it later in their hospital course. Blunt carotid
injury leading to dissection generally occurs in those
with a seat belt sign and is uncommon in those who are
neurologically intact.
• Question 134:
• Which of the following is the most common cause of
sepsis in the United States?
A)Gram-negative organisms

B)Gram-positive organisms

C)Opportunistic fungi

D)Virus-based infection

E)Mixed gram-positive and -negative infection


• Correct Answer: B
• Rationale
Although gram-negative organisms were the most
common cause of sepsis a decade ago, gram-positive
organisms are currently the most common cause of
sepsis and exceed the rate of mixed infection as well.
The incidence of fungal infections has increased during
the past decade. Viral infection is more likely in children
and the immunoincompetent but still ranks behind
bacterial infection. Pneumonia is the most common
cause of sepsis in the United States.
• Question 135:
• A 27-year-old woman is admitted to the ICU with sepsis
secondary to severe pelvic inflammatory disease. She is
receiving oxygen, 2 L/min by nasal cannula; mean arterial
pressure is increased to 70 mm Hg from a low of 48 mm
Hg, HR has decreased to 105/min from 130/min,
temperature is 39°C (102.1°F), and oxygen saturation is
97% on face mask with FIO2 of 0.4. The patient is currently
being treated with lactated Ringer solution at 200 mL/h
after receiving a 2,000-mL bolus, as well as
ampicillin/sulbactam and metronidazole. Laboratory
findings are as follows: sodium, 144 mEq/L; potassium, 4.2
mEq/L; bicarbonate, 10 mEq/L; chloride, 110 mEq/L; and
blood glucose, 104 mg/dL. Arterial pH is 7.23 with
PaCO2 of 25 mm Hg and PaO2 of 79 mm Hg. Her urine
output is 65 mL/h for the past 2 hours.
Which of the following is the best approach to
management of her metabolic acidosis?

A)Order a venous lactic acid level, and if it is elevated start


a bicarbonate infusion.
B)Start a bicarbonate infusion immediately and continue
until pH is at least 7.30.
C)The current management is appropriate and no changes
are currently necessary.
D)Administer 50 mEq sodium bicarbonate IV push
followed by a bicarbonate infusion until pH is at least 7.30.
E)Effect airway control with intubation and mechanical
ventilation to buffer the acidosis.
• Correct Answer: C
• Rationale
Bicarbonate therapy for pH over 7.2 does not improve
hemodynamics or tissue oxygenation, even in patients
with lactic acidosis.1,2 This patient appears to be
improving with her current therapy. Bicarbonate therapy
is therefore not indicated and neither is mechanical
ventilation.
• Question 136:
• A 22-year-old, gravida 3, para 3 woman is admitted to
the ICU 5 days after a cesarean delivery following
protracted labor and prolonged rupture of membranes.
On the third postoperative day, she develops a fever and
is placed on piperacillin/tazobactam. Five days
postdelivery she looks and feels well. She currently has a
BP of 115/78 mm Hg, HR of 112/min, temperature of
38.4°C (101.1°F), RR of 22/min, and WBC count of
13,900/µL. Her wound appears to be healing well and
findings of a CT of the pelvis, blood cultures, and other
laboratory testing are all negative.
The most appropriate next step is to:

A)Perform an emergent hysterectomy

B)Obtain a pelvic MRI/MR angiogram/MR venogram

C)Start therapeutic anticoagulation with heparin

D)Add fluconazole to her antimicrobial regimen

E)Perform a transvaginal ultrasonography


• Correct Answer: C
• Rationale
This patient most likely has septic pelvic thrombophlebitis.
While some patients can present several weeks
postpartum, most patients present 3-5 days postpartum
with a persistent fever despite initial antibiotics. The
clinical exam is usually benign and the patient does not
appear severely ill. Cesarean section, arrested descent,
and prolonged rupture of the membranes are significant
risk factors for this condition. The treatment of choice is
antibiotic administration along with systemic
anticoagulation. The diagnosis can be difficult. While most
patients have an elevated white blood cell count, blood
culture results are frequently negative. However, if
positive they can assist in antibiotic selection. Without a
positive culture result, antibiotics should be directed
toward gram-positive cocci and enteric organisms
including anaerobes, which are the most common
pathogens.
There is no gold standard for the diagnosis of septic
pelvic thrombophlebitis, especially septic
thrombophlebitis of the deep veins. While CT and MRI
have a better sensitivity then ultrasonography, a
negative study does not rule out the presence of a
septic pelvic thrombophlebitis; venography will not
visualize the uterine veins and neither will transvaginal
ultrasonography.

There is no evidence that the patient has a systemic


fungal infection and with only 2 days of antibiotic
therapy, a fungal infection is unlikely. Removal of the
uterus is rarely indicated for septic pelvic
thrombophlebitis and in this patient is not a first
choice for management.
• Question 137:
• Which of the following is an effective, evidence-based strategy
for reducing central venous catheter–related bloodstream
infections in the ICU?

A)Strict adherence to hand hygiene and maintenance of full


barrier precautions during catheter insertion
B)Replacing transparent dressings at catheter sites every 3
days to minimize entry site bacterial colonization
C)Preferential placement at the internal jugular site, using
ultrasound guidance to minimize number of cannulation
attempts
D)Routine replacement of central venous catheters to a new
site if still needed after 14 days
E)Using a povidone-iodine alcohol-based solution for insertion
site skin preparation and site maintenance
• Correct Answer: A
• Rationale
Central venous catheters are commonly utilized in the
ICU. Although estimates vary, approximately 80,000
catheter-related bloodstream infections are thought to
occur annually among ICU patients in the United States,
and carry with them significant morbidity as well as
financial burden. In recent years, studies have shown
that infection risk can be substantially reduced with
adoption of a comprehensive unit-based safety program
that includes staff education and training, use of
checklists during catheter insertion, appropriate
catheter maintenance, and utilization of preventive
strategies to minimize infection risk.
At the time of catheter insertion, appropriate hand
hygiene, as well as full-barrier precautions using sterile
gloves and gown, mask, cap, and large sterile drapes that
entirely cover the patient during catheter insertion has
been shown to be an essential first step to preventing
catheter-related bloodstream infections. Therefore, choice
A is o e t. “ki p epa atio ith a . % chlorhexidine
solution has been shown to be superior to aqueous
povidone-iodine solution.

While it is true that ultrasound-guided placement of an


internal jugular line may decrease the rate of catheter-
related bloodstream infections by decreasing the average
number of cannulation attempts, the subclavian site has
been associated with a lower risk of infection—provided
placement can be done safely. Thus, option C is incorrect.
Routine replacement of central venous catheters as a
strategy to reduce infection is discouraged. Rather,
emphasis should be placed on a comprehensive strategy
that includes prompt removal of unnecessary catheters,
site selection that minimizes risk (in general, subclavian
preferred over femoral or internal jugular placement as
above, especially if longer term use is expected), and
conversion to a tunneled catheter if prolonged need for
central access is identified. Therefore, option D is
incorrect.
With regard to site maintenance, daily inspection and
immediate replacement of soiled, damp, or loose
transparent dressings are especially important. Although
the optimal frequency of dressing changes is unknown,
studies have shown that scheduled changes could be
safely decreased to 7-day intervals provided the above
precautions are followed. Therefore, option B is incorrect.
Some centers additionally utilize chlorhexidine-
impregnated sponge dressings to minimize regrowth of
cutaneous flora around the entry site.
• Question 138:
• A 32-year-old man is admitted to the ICU with a clinical
diagnosis of meningococcemia and septic shock. He was
intubated in the emergency department for airway
protection. Results of a lumbar puncture are compatible
with bacterial meningitis. IV penicillin is initiated. The
chest radiograph shows no abnormalities. His temperature
is 39°C (102.2°F), with an HR of 115/min. His mean arterial
pressure is 60 mm Hg with the administration of IV
norepinephrine. He is sedated and mechanically
ventilated. Pulse oximetry is 95% with a FIO2 of 0.5 and a
positive end-expiratory pressure of 5 cm H2O. Since
presenting to the hospital 4 hours ago, he has received
crystalloid solutions totalling 2.5 L. His urine output has
decreased and is currently 0.25 mL/kg/h over the past 2
hours. He has a central venous pressure of 6 mm Hg and
an ScvO2 of 60%. WBC count is 23,500/µL, hematocrit is
28%, and platelet count is 42,000/µL (65,000/µL at the
time of the lumbar puncture).
Which of the following therapeutic interventions is
most appropriate at this time?

A)Transfuse 5 U of pooled platelets.

B)Titrate norepinephrine to mean arterial pressure >65


mm Hg.

C)Initiate dobutamine at 5 µg/kg/min.

D)Transfuse 2 U packed red blood cells.

E)Administer a plasma volume expansion bolus.


• Correct Answer: E
• Rationale
Hemodynamic support is the cornerstone of the
management of patients with severe sepsis and septic
shock. The shock profile associated with sepsis is
complex and involves several abnormalities in the
cardiovascular system. Abnormalities associated with
severe sepsis and septic shock include the following:

• I t a as ula olu e depletio se o da to i eased


capillary leakage with inflammation;
• H pote sio ith p ofou d vasodilation; and
• De eased a dia o t a tilit , hi h i the ea l
phases of sepsis is associated with a decreased cardiac
output.
In the early phases of sepsis, hemodynamic support is
aimed at correcting these abnormalities. Goals of
hemodynamic support in the first 6 hours include:

• Co e tio of i t a as ula olu e as easu ed a


central venous pressure (CVP) of 8-12 mm Hg;
• Mai te a e of ade uate tissue pe fusio as easu ed
by a mean arterial pressure (MAP) Hg; a d
• Mai te a e of ade uate a dia output as easu ed
by an ScvO2 >70%.

In the case given in the question, there are several


hemodynamic end points that have not been met. The
CVP is 6 mm Hg, MAP is 64 mm Hg, and the ScvO2 is 60%.
The first goal must be restoration of intravascular volume,
as measured by a CVP of 8-12 mm Hg. The correct answer
at this point is to give more fluids.
• Question 139:
• An 85-year-old woman is admitted to the ICU after
sustaining 7 rib fractures in a motor vehicle crash. Her
sequential organ failure assessment score is 13. A central
venous catheter is urgently inserted in the right common
femoral vein in the trauma bay due to hypotension. A right
radial artery catheter is inserted to monitor mean arterial
pressure. Paravertebral blocks are placed to help manage
her pain and improve pulmonary toilet. Two days later, she
becomes febrile to 39°C (102.1°F) and hypotensive,
requiring plasma volume expansion and pressors. Physical
examination shows no evidence of chest or abdominal
pathology. Empiric antibiotics are started. Urinalysis is
negative, and chest radiography demonstrates pulmonary
contusions. Blood cultures grow methicillin-
resistantStaphylococcus aureus infection.
Which of the following is the most likely source of the
septic shock?

A)Paravertebral block catheters

B)Central venous catheter

C)Pulmonary abscess

D)Sinusitis and sinus abscess


• Correct Answer: B
• Rationale
Approximately 15 million central venous catheter (CVC)
days are used each year in the US; the incidence of CVC-
related bloodstream infection is 80,000 cases per year.
(1) Arterial lines are frequently inserted in conjunction
with CVC. A multicenter ICU trial assessing the incidence
of arterial line and CVC infections reported a similar
infection rate for CVC and arterial lines. (2) This may be
due to the increased access of arterial lines for blood.
(3) Risk factors for arterial line colonization include
femoral site of insertion, chronic heart failure, and
chronic respiratory failure.
The risk of CVC colonization increases with septic shock,
trauma, antibiotic treatment at CVC insertion, and use
of sites other than the subclavian vein. Hence, in this
case the patient is at increased risk of central line
infection given the location and trauma history.

Sinusitis and abscess are uncommon on hospital day 2


and more commonly occur in those with endotracheal
intubation, an element that is not present in this
patient. Paravertebral block catheters are uncommonly
infected compared to urgently placed femoral venous
catheters, rendering option A much less likely than
option B.
• Question 140:
• You are on a medical mission in Africa when your camp
is enveloped in a severe storm and lightning. Several
individuals are struck by lightning.
• Which of the following statements is the best guide to
treatment of lightning injury?

A)With lightning injury, patients who appear dead should


be evaluated last.
B)Direct renal injury is a common occurrence with truncal
voltage injuries.
C)Ileus and gastric stress ulceration are the most common
complications.
D)Loss of consciousness <5 minutes does not require
inpatient admission for monitoring.
E)All patients with lightning injury should be evaluated for
24 hours for rhabdomyolysis.
• Correct Answer: E
• Rationale
Vascular injuries from electrical current can present either
immediately or as a delayed complication. Medial wall
damage can result in delayed thrombosis. Direct renal
injury rarely occurs during electrical injuries. Acute renal
failure, however, may result secondary to rhabdomyolysis.
Gastrointestinal injuries are rare. Paralytic ileus and stress
ulceration of gastric mucosa are the most frequently seen
gastrointestinal complications. Admissions for monitoring
are required if any of the following is present: (1) history
of loss of consciousness or cardiac arrest, (2) cardiac
arrhythmia in the field or in the emergency department,
and (3) abnormal ECG. Unlike virtually all other
circumstances, victims of lightning strikes may appear
dead due to a cardiac dysrhythmia that is manageable
with a precordial thump or countershock and should be
assessed and treated before all others.
• Question 141:
• A 45-year-old man is brought to the emergency
department after a motor vehicle crash in which he was
an unrestrained passenger. He presents with Glasgow
Coma Scale score of 6 and is promptly intubated for
airway protection. He was placed in a cervical collar at
the crash site. Head CT reveals petechial hemorrhage
surrounding diffuse edema and 5-mm midline shift. No
fractures or dislocations are noted on cervical spine
imaging.
Which of the following treatment options is without
outcome benefit?

A)Fosphenytoin load and daily treatment with phenytoin


for 7-10 days
B)Intracerebral pressure monitoring with placement of
ventricular drain
C)IV methylprednisolone at 1 mg/kg in divided doses for
48 hours
D)Hypertonic saline bolus and infusion for elevated
intracranial pressure
E)Airway control and minute ventilation management to
PCO2 of 35-40 mm Hg
• Correct Answer: C
• Rationale
This patient has severe traumatic brain injury (TBI). His
Glasgow Coma Scale (GCS) score before intubation
was 6 and declining, and he has abnormal findings on
his head imaging. The Brain Trauma Foundation
recommends monitoring of intracerebral pressure in
patients with a GCS score of 8 or less and abnormal
findings on head imaging. An intraventricular drain is
preferred as, in addition to allowing monitoring of
intracranial pressure (ICP), the draining of ventricular
fluid can treat the raised pressure. The goals are to keep
ICP less than 20 mm Hg and cerebral perfusion pressure
greater than 60 mm Hg.
Episodes of raised ICP of greater than 20 mm Hg lasting
more than 5-10 minutes need to be treated aggressively.
Adequate sedation and analgesia is needed. Draining
ventricular fluid, mannitol, and hypertonic saline are the
first-line treatment options. ICP refractory to these
measures can be treated with second-line therapies such
as induced hypothermia or phenobarbital coma.
Decompressive craniectomy has been associated with
more poor outcomes in survivors and is not
recommended.
The risk of seizures following TBI is greater than 10%.
Seizures create additional metabolic demands and
adversely impact management. Prophylactic antiseizure
medication has been shown to decrease the incidence of
early-onset seizures but has no effect on late-onset
seizures. The recommendations currently are for
prophylactic medications for 7-10 days.
Use of IV steroids after TBI has been shown to result in
increased morbidity and mortality and is not
recommended. Thus, option C is correct and steroids
should not be used after TBI. Airway control, on the
other hand, is associated with improved outcome in
those with Glasgow Coma Scale scores below 8 when
coupled with appropriate PaCO2management.
• Question 142:
• A 60-year-old man with a history of hypertension,
chronic kidney disease, and type 2 diabetes mellitus
is involved in a high-speed motor vehicle collision.
Injuries include a closed head injury (subarachnoid
hemorrhage, subdural hematoma, and
intraparenchymal hemorrhage) and a grade 4
actively bleeding liver laceration that is emergently
coagulated and packed. Because of the near
exsanguination and significant visceral edema due to
massive transfusion, the surgical team pursues a
damage control operation and leaves the abdomen
open with a temporary abdominal wall closure. The
team requests paralysis of the patient to manage
intracranial hypertension (intracranial pressure of 22
mm Hg) with a holiday for daily neurologic
examinations.
Which of the following neuromuscular blockers (in
addition to an analgesic and a sedative) is best suited for
this patient?

A)Pancuronium

B)Cisatracurium

C)Vecuronium

D)Pipecuronium

E)Mivacurium
• Correct Answer: B
• Rationale
Pancuronium, vecuronium, and pipecuronium are
steroidal neuromuscular blockers that are metabolized
in the liver into compounds that also have significant
neuromuscular blocking activity and are cleared by the
kidney. In patients with renal insufficiency, paralysis may
persist well after the neuromuscular blocker has been
stopped.
Mivacurium is a short-acting neuromuscular blocker. Its
action is stopped by plasma pseudocholinesterase and is
not prolonged in hepatic and renal failure. In the ICU it
is used mostly for neuromuscular blockade to facilitate
brief procedures such as bronchoscopy and central line
placement. Mivacurium is not currently available in the
United States. The action of cisatracurium, an
intermediate neuromuscular blocker, is terminated at
physiologic pH and temperature by Hofmann
degradation to inactive products.

Cisatracurium does not accumulate with renal


impairment and does not have active metabolites that
will prolong neuromuscular blockade.
• Question 143:
• Three days ago, a 38-year-old man had a splenectomy
for blunt abdominal trauma and a grade 4 splenic
laceration sustained in a motor vehicle crash. He is
otherwise healthy. His hemoglobin level immediately
after surgery was 8.8 g/dL; today it is 7.6 g/dL. His
resting HR is 110/min and he reports light-headedness
when he stands up. He weighs 85 kg (188.9 lbs).
The best initial course of action is to:

A)Transfuse 2 U of packed red blood cells

B)Administer recombinant human erythropoietin,


25,000 IU

C)Administer IV iron sucrose, 200 mg, and luminal iron


sulfate

D)Administer furosemide, 20 mg IV, and change to keep-


vein-open fluids

E)Administer IV crystalloid, 1,000 mL as a fluid bolus


• Correct Answer: E
• Rationale
A large, prospective, randomized trial demonstrated that
results of a transfusion trigger at a hemoglobin level of 7
g/dL were equivalent to those obtained with a trigger of 9
g/dL. Subgroup analysis showed decreased mortality
among patients who were less severely ill (Acute
Ph siolog a d Ch o i Health E aluatio [APACHE II]
and patients younger than 55 years.1 These results have
also been seen in retrospective studies of various groups,
including children and patients with traumatic brain
injuries. The administration of erythropoietin and/or iron
in postoperative patients has either shown no change in
hemoglobin levels2 or minimally increased hemoglobin
levels and no effect on outcomes.3 Resting tachycardia and
light-headedness are more likely symptoms of
hypovolemia than of anemia; therefore, volume infusion
with crystalloid is the best initial course of action.
• Question 144:
• A 22-year-old man who weighs 70 kg (156 lbs) is burned in
a house fire. He sustains second and third degree burns
over the entirety of his back, and the entire circumference
of 1 arm and 1 leg. Both extremity burns are
circumferential. What should be the initial IV fluid
resuscitation rate?

A)285 mL/h
B)585 mL/h
C)785 mL/h
D)1,085 mL/h
E)1,385 mL/h
• Correct Answer: C
• Rationale
The patient has suffered second and third degree burns on
45% of his body surface area (BSA). The percent of the BSA
that is i ol ed i the u a e esti ated the ule
of i es see Figu e elo . This patie t has
circumferential (both anterior and posterior) burns to one
arm and one leg, plus his entire back. Therefore, the
percent BSA equals 9% + 18% + 18% = 45%. he Parkland
formula for calculating the volume of crystalloid
resuscitation required over the first 24 hours is as follows:
Crystalloid Volume = %BSA × 4 × Weight (kg). This formula
indicates that 12,600 mL of fluid will be required over the
first 24 hours. Half of this fluid should be administered
over the first 8 hours. This translates to 6.3 L of crystalloid
over the first 8 hours, or 785 mL/h.
A o e is ette app oa h to esus itatio is ot
necessarily prudent. Although patients with severe
burns may require massive intravascular volume
resuscitation, administering fluids in excess of what the
patient actually requires may lead to complications. The
most notable complications of over-resuscitating a burn
patient would be compartment syndromes, either
abdominal compartment syndrome, or perhaps in this
case an extremity compartment syndrome. Given the
circumferential burns that this patient has in his
extremities, administering fluids in excess of what the
patient needs will contribute to edema and could
increase his risk of developing an extremity
compartment syndrome.
• Question 145:
• A 25-year-old man weighing 75 kg (167 lbs) sustains a
flame burn to 60% of his total body surface area in a
house fire. Resuscitation with lactated Ringer solution is
started, 1,125 mL/h, and titrated to maintain urine output
of 0.5 mL/kg/h. The next morning, temperature is 38.3°C
(100.9°F), HR is 120/min, BP is 130/70 mm Hg, and RR is
25/min. Starch-based colloids are administered to
minimize crystalloid fluids. His urine is clear yellow, with a
urine output of 40 mL/h. Laboratory results 24 hours after
admission demonstrate the following: WBCs, 18,000/µL;
hemoglobin, 12 g/dL; hematocrit, 35%; platelets,
65,000/µL; blood urea nitrogen, 10 mg/dL; creatinine, 1.2
mg/dL; sodium, 140 mEq/L; chloride, 100 mEq/L; and
potassium, 4.2 mEq/L. Arterial blood gas demonstrates a
pH of 7.42, PaCO2of 38 mm Hg, and PaO2 of 100 mm Hg
on an FIO2 of 30%.
Which of the following is the most likely cause of his
tachycardia?

A)Wound sepsis

B)Hypervolemia

C)Acute kidney injury

D)Hypermetabolism

E)Missed injury
• Correct Answer: D
• Rationale
Severe burn injury results in the massive release of
catecholamines, which doubles the metabolic rate in
burns covering more than 40% total body surface area.
The hypermetabolic state is associated with an elevated
temperature, tachycardia, tachypnea, and profound
weight loss despite appropriate enteral nutrition. The
white blood cell count is elevated for several days after
injury and subsequently drops precipitously and is not an
indicator of infection. The hypermetabolic state persists
for 2 years after injury. Wound sepsis does not occur
within 24 hours of injury. The patient has appropriate
urine output, vital signs, and laboratory results, making
hypovolemia or hypervolemia unlikely. While missed
injuries do occur, they do so at a low rate of about 4%—a
rate that is much less common than the well-characterized
hypermetabolism of thermal injury.
• Question 146:
• A 73-year-old woman is admitted to the ICU after a
motor vehicle collision in which she sustained fractures
of her femur and hip. She has a history of angina and
documented 2-vessel coronary artery disease managed
medically. She has no underlying pulmonary disease.
Three days later she undergoes surgery for her
fractures, and after returning from the operating room
is noted to be hypotensive (78/50 mm Hg), cyanotic
(SaO2 of 80% on room air), and tachycardic (112/min). A
pulmonary artery catheter is inserted revealing the
pulmonary artery/pulmonary artery occlusion pressure
tracing shown in the Figure.
Cardiac output is 2.4 L/min, cardiac index is 1.5
L/min/m2, right atrial pressure is 21 mm Hg,
pulmonary artery pressure is 38/26/29 mm Hg, and
pulmonary artery occlusion pressure is 14 mm Hg.
Based on the clinical and hemodynamic data, which
of the following best explains the hypotension and
hypoxemia?
A)Acute pulmonary or fat embolism
B)Postoperative hemorrhage
C)Right ventricular infarction
D)Abdominal compartment syndrome
E)Thoracic compartment syndrome
• Correct Answer: A
• Rationale
The patient is hypotensive and has a very low cardiac
output. The key findings are a very high right atrial
pressure that is greater than the pulmonary artery
occlusion pressure (PAOP) and a large pulmonary artery
end-diastolic pressure–PAOP gradient, indicating high
pulmonary vascular resistance. This picture indicates acute
right heart failure due to increased pulmonary vascular
resistance. (The fact that there is only a modest increase in
the pulmonary artery pressure may be explained by the
very low cardiac output). Acute pulmonary
thromboembolism or fat embolism could produce this
picture. A right ventricle infarction can result in a low
cardiac output, with the right atrial pressure higher than
the PAOP, and severe hypoxemia due to a patent foramen
ovale and low SvO2. However, with a right ventricular
infarct, this marked increase in pulmonary vascular
resistance would not be expected.
Abdominal compartment syndrome may produce
hypotension, low cardiac output and index but a low
right atrial pressure from impeded venous return.
Thoracic compartment syndrome generally produces
similar findings with impeded venous return but has
equalized right atrial, pulmonary artery, and pulmonary
artery occlusion pressures due to the uniformly
distributed increase in intrathoracic pressure.
• Question 147:
• A patient is brought to the ICU after ingesting cocaine by
the nasal route. He is agitated, with BP of 190/110 mm Hg,
HR of 120/min, RR of 20/min, and temperature of 39°C
(102.1°F). Which of the following interventions would be
most appropriate for this patient?

A)IV propranolol by infusion

B)IV lorazepam

C)Activated charcoal via nasogastric tube

D)Intramuscular haloperidol

E)Clonidine by transdermal patch


• Correct Answer: B
• Rationale
The first line of treatment for agitation in acute cocaine
intoxication is benzodiazepines, which should be delivered
by an IV route. Haloperidol, which can lower seizure
threshold, should be avoided. An ECG should be obtained
to evaluate for potential myocardial ischemia. Fluids
should be administered in light of the tachycardia,
hyperthermia, and excessive muscular activity with
agitation, which predisposes to the development of
rhabdomyolysis. Cocaine is rapidly absorbed from all
mucosal surfaces and activated charcoal is unlikely to have
any benefit after the first hour following ingestion.
Significant hypertension associated with cocaine
intoxication often responds to control of agitation. For
sustained hypertension, an agent with alpha- and beta-
blocking activity (such as labetalol) is preferred. Use of a
relatively selective beta-blocker (such as metoprolol) alone
may result in unopposed alpha-adrenergic-mediated
vasoconstriction and worsening of hypertension.
• Question 148:
• A 50-year-old man is transferred to the ICU after rescue
from a house fire. On exam, he is unresponsive and
comatose. Arterial blood gas results on 10-L nasal cannula
oxygen are as follows: pH, 7.16; PaCO2, 30 mm Hg; PaO2,
214 mm Hg. The most important next step in management
is to:

A)Check the arterial blood gas carboxyhemoglobin level


B)Place the patient on a 100% nonrebreather mask
C)Transfer the patient to a hyperbaric chamber
D)Intubate and ventilate the patient with 100% FIO2
E)Treat the patient with IV sodium bicarbonate
• Correct Answer: D
• Rationale
The patient is presenting with severe carbon monoxide
poisoning. Carbon monoxide poisoning is a major cause
of death following smoke inhalation. The correct answer
is D, intubation and ventilation. The patient is comatose
and unresponsive and thus immediate intervention
must include securing the airway. Options A, C, and E
a e all p o a l i di ated i the patie t’s t eat e t pla
but should follow intubation. Option B would be the
correct initial management if the patient did not have
severe altered mental status or impending respiratory
failure requiring intubation.
• Question 149:
• A 50-year-old man is transferred to the ICU after rescue
from a house fire. On exam, he is confused and tachypneic
with RR of 38/min. Carbon monoxide poisoning is
associated with increased minute ventilation via which of
the following mechanisms?

A)Increased dead space ventilation


B)Shift of hemoglobin dissociation curve
C)Induction of systemic lactic acidosis
D)Decreased pulmonary capillary exchange of carbon
dioxide
E)Increased tissue oxygen content
• Correct Answer: C
• Rationale
Carbon monoxide rapidly diffuses across the pulmonary
capillary membrane and has more than 200 times greater
affinity for binding to hemoglobin than does oxygen. Once
bound, carbon monoxide impairs the ability of
hemoglobin to offload bound oxygen and thus impairs
tissue oxygen delivery, rather than increasing it (option E).
This impaired delivery can lead to anaerobic metabolism
and severe lactic acidosis. This metabolic acidosis triggers
compensatory hyperventilation, and thus the answer is C.
The shift of the hemoglobin dissociation curve in option B
does occur, but does not trigger significant
hyperventilation. Options A and D are incorrect as they
have nothing to do with carbon monoxide intoxication.
• Question 150:
• You are consulted by the emergency department for a
50-year-old man brought by ambulance after exposure
to an office fire. He has previous medical history of
chronic alcohol abuse, cigarette smoking, hypertension,
and diabetes mellitus. He is asymptomatic and physical
exam reveals no signs of burn injury. Arterial blood gas
(ABG) results on room air show pH of 7.37, PaCO2 of 50
mm Hg, and a PaO2 of 78 mm Hg. Carboxyhemoglobin
level is 10%.
• Which of the following is the most appropriate next step
in management?

A)Discharge the patient to home after completion of


emergency department evaluation.
B)Admit to a floor bed on 100% nonrebreather with
repeat carboxyhemoglobin in 24 hours.
C)Admit to the ICU on a 100% nonrebreather with serial
carboxyhemoglobin levels.
D)Intubate and oxygenate with 100% oxygen; admit to ICU
with serial carboxyhemoglobin levels.
E)Intubate and ventilate the patient with 100% oxygen;
transfer urgently for hyperbaric oxygen therapy.
• Correct Answer: A
• Rationale
The patient is being evaluated for possible carbon
monoxide poisoning given exposure to smoke
inhalation. Carboxyhemoglobin level is 10% on arterial
blood gas studies. A normal carboxyhemoglobin level in
a nonsmoker is less than 5%; however, cigarette
smokers can have a baseline carboxyhemoglobin level of
10%- %. Gi e the patie t’s histo of s oki g a d
negative workup, he can be safely discharged home
after a thorough evaluation.

You might also like